The Flat Earth Society

Flat Earth Discussion Boards => Flat Earth Theory => Topic started by: Iactuallycanthink on March 30, 2020, 12:38:35 PM

Title: The Math for universal Acceleration IS INCORRECT
Post by: Iactuallycanthink on March 30, 2020, 12:38:35 PM
STATEMENTS MADE by Flat Earth Theory Wiki
Gravity does not exist ... that Equivalancy can produce the same effect
That Universal acceleration of the earth does not mean the earth will accelerate past the speed of light

I have shown simply below that  Universal acceleration using the equations that were use to prove that  the earth would not accelerate past the speed of light would mean this  acceleration MUST  reach near 0 making it impossible for  euqivalancy to be a possible explanation for  gravitational effect after  the speed of the earth reachs speeds near speed of light

I have NEVER MENTIONED FRAME OF REFERENCE FOR A REASON.
A ) YOU CANNOT DETERMINE THE MOTION OF THE FRAME OF REFERENCE FROM WITHIN THE FRAME OF REFERENCE
B)  i DONT CARE WHAT THE FRAME OF REFERENCE IS  .. i AM NOT THE ONE SAYING THE EARTH  TRAVELS AT A CONSTANT ACCELERATION FOREVER..

c )i AM NOT STUPID  ..  MY TOTAL  ARGUEMENT IS YOU  cannot have a constant acceleration  greater than 0 of the frame of the EARTH  with  NO SPEED CHANGE AND NO DIRECTION CHANGE .. THIS IS BASIC  DEFINITION OF  ACCELERATION..

UNFORTUNATELY   ALL THE PEOPLE GO ON AND ON ABOUT FRAME OF REFERENCE  sorry you dont know  what that means and i am not going into detail about it .. with you as  just the SIMPLE  TOPIC  THAT AN EQUATION  GIVEN AS PROOF  that the earth would  not  go above   the speed of light   cant be comprehended  by  closed minded people

some tenants of  RELATIVITY  for those who THINK  they can tell me they have to teach me things
The maximum speed of anything  with mass is the just under the speed of light  ..this is a given
with this as a given it means that  the object that was accelerating  when it reaches near c MUST be slowing  its acceleration  to 0 WHEN IT  reaches the maximum speed  TheREALDAVE  said there was nothing in relativity that said something cannot accelerate forever    HAVING A MAX SPEED  means you cannot exceed said speed and therefore your acceleration MUST drop to 0

for all those fixated on picking a FRAME OF REFERENCE TELL ME WHAT FRAME  of reference allows the earth TO   accelerate  and have constant speed at the  SAME TIME

simply put 

as i stated before  FET  equation for   acceleration is     UA  = g / L^3

where L = sqrt (1 - v^2/c^2)  being the lorentz equation  .. im sorry you cant understand  .. that  there is no need to reference frames   

this equation cant be simplified  to  UA = g * (1 - v^2/c^2)^(3/2)

the  FET  stipulated on this website says  that    the max velocity of the earth  is  c   and that  if the earth is accelerating at  1 g for  a limited time  will eventually  reach   the speed of  c

just before  it reaches c  say    c - 0.0000000000000000000000000000000000000000000000000001

the UA will be   = g * (1 - (c - 0.00000000000000000000000000000000000000000000001)^2/c^2)^(3/2)

which  you can see will be  approx  by   g* (1 - c^2/c^2)^(3/2) with any  normal accuracy

notice how   UA = g* 0^(3/2)

notice HOW   UA = 0 actually so close it wont even matter it will be like   UA = 0.000000000000000000000000000000000000001

WHEN  the  speed  reaches near to  c

I appologize if you dont get math ..its   not my job to   teach everyone when something applies  and when it doesnt..

there is  NO FRAME OF REFERENCE VARIATION   that you can come up with where there can be  a  MEASURED ACCELERATION OF  NEAR   g

or even  1/10 g  .. when the  velocity is NEAR  C   of any object    period  thank you for  your  endliess  bs  about  FOR  = frames of reference 

they do not apply to this   

ps  SAYING   frames of reference  affects it and NOT GIVING A SINGLE EXAMPLE    .. of course i am ignoring  TheRealDaves .. bogus  example where   the earth  can miraculously  accelerate   while it keeps a  velocity of  0 as FICTION  not fact



The purpose of this post is to show the lack of knowledge regarding  acceleration and velocity

FET on this webpage asserts that  an object can accelerate forever because the limit in speed is  c the speed of light

this is INCORRECT .. because as it gets closer to the speed of light the acceleration SLOWS and thus  eventually in the limit as I have shown in the equations below drops to 0 if UA drops to 0 when  the speed of the earth is  near C then universal accleration cant be a reason for the feeling of gravity

one thing follows another

unlike  DAVE the Donkey   I know about frames of reference ..i know that if an object is travelling at the maximum speed it can its acceleration is 0 and that  any  AND I DO MEAN  ANY ...

AFFECTS of that acceleration  like the feeling of being pulled down  disappear

any OBJECT moving at a constant speed in space   without some other force acting on it  the person inside that object or on that object will experience 0 effects

It would be nice if the website  runners actually  fix the FALSE math  regarding  UA but that is  wishful thinking as  they have defenders LIKE  DAVE THE DONKEY ..  who has provided  0   ACTUAL work  ... he just says  my assertion is incorrect

damn    but he wants me to show my WORK WHICH I DID  AT LEAST THREE TIMES

DAVE SAYS without actually showing proof that the earth can accelerate in its own frame of reference

a BOX ..  can move with respect to itself

thats is what MOTION IS

IF   E is here

then   later  E is here

with respect to itself  it has not MOVED SO ITS ACCELERATION IS 0

WASNT IT  you DAVE  the DONKEY who said with respect to what  trying to discredit me  with your astute statement



dv/dt = acceleration

your explanation that  with relativity you are limited to a speed of  c  is  very simple minded

if the velocity at one moment is  c and the velocity a second later is  c

dv/dt = (c - c)/1 =  0/1 = 0

this means that when the object gets close to c it starts to not ACCELERATE SO FAST

and eventually STOPS  accelerating..
this can be shown by your equation 

dv/dt = g/L^3

where  L is lorentz factor

the reciprical of the lorentz factor   gets closer and closer to 0

since 1/L gets close to 0

1/L^3 must get really close to 0



Title: Re: The Math for universal Acceleration IS INCORRECT
Post by: Clyde Frog on March 30, 2020, 02:09:03 PM
this means that when the object gets close to c it starts to slow down

and eventually STOPS  accelerating..
Slows down and stops accelerating with respect to what?
Title: Re: The Math for universal Acceleration IS INCORRECT
Post by: Iactuallycanthink on March 30, 2020, 03:43:48 PM
this means that when the object gets close to c it starts to slow down

and eventually STOPS  accelerating..
Slows down and stops accelerating with respect to what?

the statement that  FLAT EARTH PEOPLE  say is that the earth  ACCELERATES  forever .. but  that  it NEVER EXCEEDS   the speed of light due to their missapplying the  equations of relativity

I am just showing that when the   speed of  the earth  nears c which they say is the limit that   

by the same equations they give the  earth MUST SLOW  its acceleration and  cannot keep acceleration at   10 m/s

they say the earth is  MOVING  ACCELERATING ALWAYS  WITH RESPECT TO THE REST OF THE UNIVERSE
Title: Re: The Math for universal Acceleration IS INCORRECT
Post by: Groit on March 30, 2020, 07:07:24 PM
this means that when the object gets close to c it starts to slow down

and eventually STOPS  accelerating..
Slows down and stops accelerating with respect to what?

With respect to the 'comoving' Galaxy. All comoving objects move with the expansion of space, not through space. An accelerating Earth would be moving through space relative to the comoving Galaxy, and matter cannot accelerate through space to the speed of light, the energy needed would go to infinity.

Title: Re: The Math for universal Acceleration IS INCORRECT
Post by: Clyde Frog on March 30, 2020, 09:51:41 PM
this means that when the object gets close to c it starts to slow down

and eventually STOPS  accelerating..
Slows down and stops accelerating with respect to what?

the statement that  FLAT EARTH PEOPLE  say is that the earth  ACCELERATES  forever .. but  that  it NEVER EXCEEDS   the speed of light due to their missapplying the  equations of relativity

I am just showing that when the   speed of  the earth  nears c which they say is the limit that   

by the same equations they give the  earth MUST SLOW  its acceleration and  cannot keep acceleration at   10 m/s

they say the earth is  MOVING  ACCELERATING ALWAYS  WITH RESPECT TO THE REST OF THE UNIVERSE
You're asserting that there is a preferred frame of reference. That is in disagreement with modern physics where the consensus is that there is no preferred FoR. How would you back up your position?

If you accept there is no preferred FoR, then there really isn't an issue with something undergoing constant acceleration indefinitely. This is quite literally the worst argument against UA there could be.
Title: Re: The Math for universal Acceleration IS INCORRECT
Post by: pricelesspearl on March 30, 2020, 10:34:20 PM
Quote
You're asserting that there is a preferred frame of reference. That is in disagreement with modern physics where the consensus is that there is no preferred FoR. How would you back up your position?

If you accept there is no preferred FoR, then there really isn't an issue with something undergoing constant acceleration indefinitely. This is quite literally the worst argument against UA there could be.

Constant acceleration with respect to what ?  You can’t have it both ways.  If an earth slowing its rate of acceleration assumes a preferred FOR, so does an earth that is increasing its rate.
Title: Re: The Math for universal Acceleration IS INCORRECT
Post by: Clyde Frog on March 30, 2020, 10:47:45 PM
It's astounding how poorly you understand the thing you are arguing about. The earth can be constantly accelerating in its own frame of reference. Which is to say, an object situated on its surface could experience a constant 9.8m/s2 downward force perpetually.

Why is this the hill so many people choose to die on? There is nothing in Relativity that suggests any object could not undergo constant acceleration forever. If you disagree, please show your work.
Title: Re: The Math for universal Acceleration IS INCORRECT
Post by: pricelesspearl on March 30, 2020, 11:30:42 PM
It's astounding how poorly you understand the thing you are arguing about. The earth can be constantly accelerating in its own frame of reference. Which is to say, an object situated on its surface could experience a constant 9.8m/s2 downward force perpetually.

Why is this the hill so many people choose to die on? There is nothing in Relativity that suggests any object could not undergo constant acceleration forever. If you disagree, please show your work.

You are arguing what you said went against physics...that the Earth's frame of reference is preferred.

If the earth is constantly accelerating in it's own frame of reference resulting in objects situated on its surface experiencing a constant 9.8m/s2 downward force , then according to the math Icanactuallythink already showed you, in it's own frame of reference, the earth should have reached the point at which acceleration slows and objects on its surface would begin experience something less than 9.8m/s2 downward force.

I don't think that is happening.

Title: Re: The Math for universal Acceleration IS INCORRECT
Post by: Iactuallycanthink on March 31, 2020, 12:08:03 AM
It's astounding how poorly you understand the thing you are arguing about. The earth can be constantly accelerating in its own frame of reference. Which is to say, an object situated on its surface could experience a constant 9.8m/s2 downward force perpetually.

Why is this the hill so many people choose to die on? There is nothing in Relativity that suggests any object could not undergo constant acceleration forever. If you disagree, please show your work.

REALLY.. SO IF I AM IN A ROCKET SHIP  and it accelerates at 10 m/s^2  does  IT CAN  keep on going fore ever

that MEANS THAT THE EQUATION IS FALSE 
MAN YOU PEOPLE  just dont get it

you say that  the limit of speed is  c
this ONLY happens if the acceleration slows as the speed of  light is reached

dv/dt = g * (1/L)^3

since L = 1/sqrt(1- v^2/c^2)

then  dv/dt = g * sqrt(1- v^2/c^2)^3

in the limit as v > c

1 - v^2/c^2 = 0

when you want to tell ME  or any scientist that  something can accelerate forever you are a moron

after  1 year  without  using  relativity the speed  reaches c
and therefore the acceleration reaches 0


You are arguing what you said went against physics...that the Earth's frame of reference is preferred.

If the earth is constantly accelerating in it's own frame of reference resulting in objects situated on its surface experiencing a constant 9.8m/s2 downward force , then according to the math Icanactuallythink already showed you, in it's own frame of reference, the earth should have reached the point at which acceleration slows and objects on its surface would begin experience something less than 9.8m/s2 downward force.

I don't think that is happening.
Title: Re: The Math for universal Acceleration IS INCORRECT
Post by: Clyde Frog on March 31, 2020, 12:10:05 AM
It's astounding how poorly you understand the thing you are arguing about. The earth can be constantly accelerating in its own frame of reference. Which is to say, an object situated on its surface could experience a constant 9.8m/s2 downward force perpetually.

Why is this the hill so many people choose to die on? There is nothing in Relativity that suggests any object could not undergo constant acceleration forever. If you disagree, please show your work.

You are arguing what you said went against physics...that the Earth's frame of reference is preferred.
No, I'm not. Maybe you need to study the subject a little more. This is why FoR matters. If you are standing on the Earth and it's a disc traveling upwards at g, the Earth's velocity in your FoR is always 0 m/s, but you feel the force of g holding you down. If you are floating in space at t=0 and see the Earth fly by you at 9.8m/s/s the moment it passes you and it continues accelerating as it gets further away, you see it slow down as it approaches c, but GUESS WHAT? For the person on the Earth, they can still keep experiencing g standing on the surface. And 1 year after the Earth has passed the first person, another person floating in space (moving quite fast as compared to the first person we enountered) sees the Earth fly by them at 9.8m/m/s and continue accelerating as it gets further away. They see the Earth slow down as it continues approaching c from their FoR. But the first person sees the Earth moving much slower than the second. Now extrapolate over n observers as n approaches infinity. The n-1 observer still just sees the Earth fly by them at 9.8m/s/s.

I'm not even FE. You are just poorly informed.
Title: Re: The Math for universal Acceleration IS INCORRECT
Post by: Iactuallycanthink on March 31, 2020, 12:18:51 AM
It's astounding how poorly you understand the thing you are arguing about. The earth can be constantly accelerating in its own frame of reference. Which is to say, an object situated on its surface could experience a constant 9.8m/s2 downward force perpetually.

Why is this the hill so many people choose to die on? There is nothing in Relativity that suggests any object could not undergo constant acceleration forever. If you disagree, please show your work.

ive shown the work you HAVE NOT

WITH RESPECT TO THE SECOND PERSON FLOATING IN SPACE

dv/dt of the earth after ONE YEAR WOULD BE  0

THAT MEANS THE earth would be  CONSTANTLY MOVING AT C

IT IS  you that cant understand the relationship between acceleration and velocity
if there is no speed change ore direction change there is no velocity

thats why  scientists who want a gravitation environment forever in space use a  ROTATIONAL SYSTEM  because rotation can go on forever

You are arguing what you said went against physics...that the Earth's frame of reference is preferred.
No, I'm not. Maybe you need to study the subject a little more. This is why FoR matters. If you are standing on the Earth and it's a disc traveling upwards at g, the Earth's velocity in your FoR is always 0 m/s, but you feel the force of g holding you down. If you are floating in space at t=0 and see the Earth fly by you at 9.8m/s/s the moment it passes you and it continues accelerating as it gets further away, you see it slow down as it approaches c, but GUESS WHAT? For the person on the Earth, they can still keep experiencing g standing on the surface. And 1 year after the Earth has passed the first person, another person floating in space (moving quite fast as compared to the first person we enountered) sees the Earth fly by them at 9.8m/m/s and continue accelerating as it gets further away. They see the Earth slow down as it continues approaching c from their FoR. But the first person sees the Earth moving much slower than the second. Now extrapolate over n observers as n approaches infinity. The n-1 observer still just sees the Earth fly by them at 9.8m/s/s.

I'm not even FE. You are just poorly informed.
Title: Re: The Math for universal Acceleration IS INCORRECT
Post by: Iactuallycanthink on March 31, 2020, 12:34:41 AM
this means that when the object gets close to c it starts to slow down

and eventually STOPS  accelerating..
Slows down and stops accelerating with respect to what?

the statement that  FLAT EARTH PEOPLE  say is that the earth  ACCELERATES  forever .. but  that  it NEVER EXCEEDS   the speed of light due to their missapplying the  equations of relativity

I am just showing that when the   speed of  the earth  nears c which they say is the limit that   

by the same equations they give the  earth MUST SLOW  its acceleration and  cannot keep acceleration at   10 m/s

they say the earth is  MOVING  ACCELERATING ALWAYS  WITH RESPECT TO THE REST OF THE UNIVERSE
You're asserting that there is a preferred frame of reference. That is in disagreement with modern physics where the consensus is that there is no preferred FoR. How would you back up your position?

If you accept there is no preferred FoR, then there really isn't an issue with something undergoing constant acceleration indefinitely. This is quite literally the worst argument against UA there could be.

if you are a NON FLAT EARTHER   you  are not in science

first of all I NEVER STATE THE EARTH IS MOVING .. i say if the  earth is ACCELERATING IN A STRAIGHT LINE AS FLAT EARTHERS SAY

AND  that  the equations  THEY USE  to prove that  the earth  WOULD not exceed  the speed of C  also show  that the  ACCELERATION  the earth experiences  will eventually  be 0

Dont assume what you cant understand ..

i know no object with mass can ever move faster with respecty to any other object  faster than the speed of light

the argument  that flat earthers says

the earth is moving with respect to the rest of space

doesnt matter   any frame of reference CANNOT   keep moving at a high acceleration before eventually reaching the speed of C

TWO

THEY SAY THAT EVENTUALLY THE EARTH  REACHES THE SPEED OF C

THEN  by the basic definition of   ACCELERATION   dv/dt   if the velocity  DOES NOT CHANGE   the  acceleration is  0

YOU PICKED A PROPER SYMBOL FOR YOUR FACE
Title: Re: The Math for universal Acceleration IS INCORRECT
Post by: Iactuallycanthink on March 31, 2020, 12:36:15 AM
this means that when the object gets close to c it starts to slow down

and eventually STOPS  accelerating..
Slows down and stops accelerating with respect to what?



With respect to the 'comoving' Galaxy. All comoving objects move with the expansion of space, not through space. An accelerating Earth would be moving through space relative to the comoving Galaxy, and matter cannot accelerate through space to the speed of light, the energy needed would go to infinity.


IF SOMETHING CANT EXCEED A SPEED IT   reaches what is its acceleration
dv/dt = (c - c)/1 = 0/1 = 0

this is  SIMPLE RELATION
Title: Re: The Math for universal Acceleration IS INCORRECT
Post by: Iactuallycanthink on March 31, 2020, 12:48:32 AM
It's astounding how poorly you understand the thing you are arguing about. The earth can be constantly accelerating in its own frame of reference. Which is to say, an object situated on its surface could experience a constant 9.8m/s2 downward force perpetually.

Why is this the hill so many people choose to die on? There is nothing in Relativity that suggests any object could not undergo constant acceleration forever. If you disagree, please show your work.

You are arguing what you said went against physics...that the Earth's frame of reference is preferred.
No, I'm not. Maybe you need to study the subject a little more. This is why FoR matters. If you are standing on the Earth and it's a disc traveling upwards at g, the Earth's velocity in your FoR is always 0 m/s, but you feel the force of g holding you down. If you are floating in space at t=0 and see the Earth fly by you at 9.8m/s/s the moment it passes you and it continues accelerating as it gets further away, you see it slow down as it approaches c, but GUESS WHAT? For the person on the Earth, they can still keep experiencing g standing on the surface. And 1 year after the Earth has passed the first person, another person floating in space (moving quite fast as compared to the first person we enountered) sees the Earth fly by them at 9.8m/m/s and continue accelerating as it gets further away. They see the Earth slow down as it continues approaching c from their FoR. But the first person sees the Earth moving much slower than the second. Now extrapolate over n observers as n approaches infinity. The n-1 observer still just sees the Earth fly by them at 9.8m/s/s.

I'm not even FE. You are just poorly informed.


IF a object is in space and  the earth approaches at near the speed of light

1 second later the speed of light cant change because this is the limit of speed of an object with mass  according to RELATIVITY

I understand more than you can imagine  DAVE

THE BASIC TENANT OF RELATIVITY IS there is no absolute  frame of reference

if someone ON THE EARTH  without gravity

AND THIS EARTH IS GOING AT A CONSTANT LINEAR  VELOCITY

it will experience 0 acceleration

F = ma = m*0 = 0 force

so a person on the earth that is moving at a constant velocity of  C  will experience  0 force and feel no pull down toward the earth from NON EXISTANT UNIVERSAL ACCELERATION
Title: Re: The Math for universal Acceleration IS INCORRECT
Post by: Clyde Frog on March 31, 2020, 01:22:16 AM
I can't hold your hand through understanding Relativity. Clearly you aren't grasping one of the pillars of the theory, which is that in any FoR, an observer in said frame measures the same value for c. Even if that observer is accelerating with respect to another observer. And there is no law that prevents something from undergoing constant acceleration indefinitely. The hypothetical object that is constantly accelerating never exceeds c from its own frame. It is only from outside frames that relativistic effects on its observed velocity would be observed. You need to come to terms with how this all works before you start an argument that is nonsense.
Title: Re: The Math for universal Acceleration IS INCORRECT
Post by: Iactuallycanthink on March 31, 2020, 01:25:25 AM
It's astounding how poorly you understand the thing you are arguing about. The earth can be constantly accelerating in its own frame of reference. Which is to say, an object situated on its surface could experience a constant 9.8m/s2 downward force perpetually.

Why is this the hill so many people choose to die on? There is nothing in Relativity that suggests any object could not undergo constant acceleration forever. If you disagree, please show your work.

FALSE

i am not stipulating any frame of reference

the FE model states that   scientists are incorrect when it is stated that UNIVERSAL ACCELERATION  will exceed the speed of light

i have simply shown with the same equations that  the  FE'S  use THAT THE earths acceleration drops to 0  WITH RESPECT TO ANY OBJECT  or frame of reference  and when it drops to 0  the  force felt by someone on the earth therefore has to drop to 0

I have MADE NO ASSUMPTIONS 
Title: Re: The Math for universal Acceleration IS INCORRECT
Post by: Clyde Frog on March 31, 2020, 01:31:50 AM
You aren't accounting for FoR though, which is where your argument fails. Because there is no preferred FoR, you can't say anything about how fast the Earth is moving unless you are also stating from which FoR you are measuring from. And that matters IMMENSELY. In UA, the Earth is always moving at 0 m/s at any moment from its own reference frame so it can't possibly exceed c. And from an outside frame, it hardly matters, because the person feeling the effects of gravity on Earth is obviously not in said outside frame.
Title: Re: The Math for universal Acceleration IS INCORRECT
Post by: Iactuallycanthink on March 31, 2020, 01:41:32 AM
I can't hold your hand through understanding Relativity. Clearly you aren't grasping one of the pillars of the theory, which is that in any FoR, an observer in said frame measures the same value for c. Even if that observer is accelerating with respect to another observer. And there is no law that prevents something from undergoing constant acceleration indefinitely. The hypothetical object that is constantly accelerating never exceeds c from its own frame. It is only from outside frames that relativistic effects on its observed velocity would be observed. You need to come to terms with how this all works before you start an argument that is nonsense.


I dont need HAND HOLDING

you  DONT UNDERSTAND RELATIVITY

p1      E        p2

if E is accelerating always toward P2

then P1 sees E moving away at a constant acceleration of  10 m/s^2

but  E cant move faster than C  THIS IS  basic tenant of relativity .. i dont need to argue it it is 

so when the E reaches  the speed of C it is essentially NOT ACCELERATING

no ACCELERATION
 MEANS   no  FORCE DOWN

just moving at C does not create a force down sorrry

the reason  YOU FEEL A FORCE DOWN in an elevator is  BECAUSE THERE IS  gravity  which UNIVERSAL acceleration is supposed to  REPLACE

I HAVE ALSO SHOWN  that  with the equations  from RELATITIVITY THAT THE  FE'S  USE  to prove the validity of UA

that  dv/dt = g (1 - v^2/c^2)^(3/2)

notice that as  v approaches c  that   the  portion under the exponent is  near 0

a number near 0 to any power is a smaller number near 0

is that enough  bloody HAND HOLDING FOR YOU  MR DAVE THE DONKEY
Title: Re: The Math for universal Acceleration IS INCORRECT
Post by: juner on March 31, 2020, 02:52:24 AM
I can't hold your hand through understanding Relativity. Clearly you aren't grasping one of the pillars of the theory, which is that in any FoR, an observer in said frame measures the same value for c. Even if that observer is accelerating with respect to another observer. And there is no law that prevents something from undergoing constant acceleration indefinitely. The hypothetical object that is constantly accelerating never exceeds c from its own frame. It is only from outside frames that relativistic effects on its observed velocity would be observed. You need to come to terms with how this all works before you start an argument that is nonsense.


I dont need HAND HOLDING

you  DONT UNDERSTAND RELATIVITY

p1      E        p2

if E is accelerating always toward P2

then P1 sees E moving away at a constant acceleration of  10 m/s^2

but  E cant move faster than C  THIS IS  basic tenant of relativity .. i dont need to argue it it is 

so when the E reaches  the speed of C it is essentially NOT ACCELERATING

no ACCELERATION
 MEANS   no  FORCE DOWN

just moving at C does not create a force down sorrry

the reason  YOU FEEL A FORCE DOWN in an elevator is  BECAUSE THERE IS  gravity  which UNIVERSAL acceleration is supposed to  REPLACE

I HAVE ALSO SHOWN  that  with the equations  from RELATITIVITY THAT THE  FE'S  USE  to prove the validity of UA

that  dv/dt = g (1 - v^2/c^2)^(3/2)

notice that as  v approaches c  that   the  portion under the exponent is  near 0

a number near 0 to any power is a smaller number near 0

is that enough  bloody HAND HOLDING FOR YOU  MR DAVE THE DONKEY

Keep your whining rants in the Angry Ranting forum. See the forum rules if you need further details. Warned.

Also, if you are going to claim to understand relativity, in this case SR, then maybe you should try using the proper equation instead of sticking with classical mechanics. That, and learning how a FoR works, should help keep you from being as proudly ignorant as you are here. It is embarrassing to both you and your fellow RErs.

Title: Re: The Math for universal Acceleration IS INCORRECT
Post by: Iactuallycanthink on March 31, 2020, 03:02:29 AM
You aren't accounting for FoR though, which is where your argument fails. Because there is no preferred FoR, you can't say anything about how fast the Earth is moving unless you are also stating from which FoR you are measuring from. And that matters IMMENSELY. In UA, the Earth is always moving at 0 m/s at any moment from its own reference frame so it can't possibly exceed c. And from an outside frame, it hardly matters, because the person feeling the effects of gravity on Earth is obviously not in said outside frame.


EP

THIS is a Earth with a person on it

later ................   EP

with if as per your stipulation  DAVE THE EXPERT ON FOR

E VELOCITY AT THE beginning is 0
E velocity at the second point is  0

change in velocity  = 0 - 0 = 0

change in time    something

change in velocity / change in time  = 0 /something = 0 

damn  why do you figure that an object  can  have an acceleration inside its own frame of reference when its velocity never changes

and  two   the premise of relativity is that  there is NO WAY TO MEASURE  your  motion inside YOUR OWN FOR


Title: Re: The Math for universal Acceleration IS INCORRECT
Post by: Iactuallycanthink on March 31, 2020, 04:53:04 AM
Quote
You're asserting that there is a preferred frame of reference. That is in disagreement with modern physics where the consensus is that there is no preferred FoR. How would you back up your position?

If you accept there is no preferred FoR, then there really isn't an issue with something undergoing constant acceleration indefinitely. This is quite literally the worst argument against UA there could be.

Constant acceleration with respect to what ?  You can’t have it both ways.  If an earth slowing its rate of acceleration assumes a preferred FOR, so does an earth that is increasing its rate.

No i did not argue or stipulate any frame of reference ..

I simply used the equations given by FET  for Universal Acceleration.. and showed that at the same time it reaches the maximum speed of  C  it must also  UA must diminish to a limit of  0

that is ALL I DID .. i dont care what frames you pick
Title: Re: The Math for universal Acceleration IS INCORRECT
Post by: Iactuallycanthink on March 31, 2020, 04:58:38 AM
I can't hold your hand through understanding Relativity. Clearly you aren't grasping one of the pillars of the theory, which is that in any FoR, an observer in said frame measures the same value for c. Even if that observer is accelerating with respect to another observer. And there is no law that prevents something from undergoing constant acceleration indefinitely. The hypothetical object that is constantly accelerating never exceeds c from its own frame. It is only from outside frames that relativistic effects on its observed velocity would be observed. You need to come to terms with how this all works before you start an argument that is nonsense.


I dont need HAND HOLDING

you  DONT UNDERSTAND RELATIVITY

p1      E        p2

if E is accelerating always toward P2

then P1 sees E moving away at a constant acceleration of  10 m/s^2

but  E cant move faster than C  THIS IS  basic tenant of relativity .. i dont need to argue it it is 

so when the E reaches  the speed of C it is essentially NOT ACCELERATING

no ACCELERATION
 MEANS   no  FORCE DOWN

just moving at C does not create a force down sorrry

the reason  YOU FEEL A FORCE DOWN in an elevator is  BECAUSE THERE IS  gravity  which UNIVERSAL acceleration is supposed to  REPLACE

I HAVE ALSO SHOWN  that  with the equations  from RELATITIVITY THAT THE  FE'S  USE  to prove the validity of UA

that  dv/dt = g (1 - v^2/c^2)^(3/2)

notice that as  v approaches c  that   the  portion under the exponent is  near 0

a number near 0 to any power is a smaller number near 0

is that enough  bloody HAND HOLDING FOR YOU  MR DAVE THE DONKEY

Keep your whining rants in the Angry Ranting forum. See the forum rules if you need further details. Warned.

Also, if you are going to claim to understand relativity, in this case SR, then maybe you should try using the proper equation instead of sticking with classical mechanics. That, and learning how a FoR works, should help keep you from being as proudly ignorant as you are here. It is embarrassing to both you and your fellow RErs.

i was responding to an person who has NO KNOWLEDGE OF  complex let alone simple 

Title: Re: The Math for universal Acceleration IS INCORRECT
Post by: JSS on March 31, 2020, 11:22:32 AM
so when the E reaches  the speed of C it is essentially NOT ACCELERATING

The above quote shows you are not grasping the basic theory of relativity.  It's not surprising as it really is a crazy concept to wrap your head around, but the math for Universal Acceleration is quite correct.

But there are tons of great videos and tutorials out there that explain it.  Here is one, but you should look at a bunch until you get the concept. It took me a while before it all clicked, and only a shallow understanding. Don't ask me to quote formulas, they make my head hurt.

https://www.sciencealert.com/watch-the-famous-twin-paradox-of-special-relativity-explained

Nothing about UA breaks any of the rules of relativity, you certainly can accelerate at 1G forever and never reach C.  You just have to remember that time AND space is relative. An outside observer will see you get closer and closer to C but never reach it, and the Earth will experience time dilation and see that observer moving slower and slower. Objects undergoing acceleration change both how they experience moving through time as well as space.

If you want to argue against UA, there are plenty of other options.  Going 99.99999999% of the speed of light means the rest of the universe better be a complete void, because if you hit a single atom at that speed the explosion will vaporize the whole planet instantly. I can't even express how huge the numbers get after experiencing only 6,000 years of 1G acceleration. An outside observer would need to wait 8.843e+1345 years to watch the Earth accelerate for that long.  That's 1,345 zeros.
Title: Re: The Math for universal Acceleration IS INCORRECT
Post by: Groit on March 31, 2020, 03:59:34 PM
So according to UA, any outside observers will see the earth travelling near the speed of light and also see our clocks ticking very slowly. Is that correct?
Title: Re: The Math for universal Acceleration IS INCORRECT
Post by: Groit on March 31, 2020, 04:54:14 PM
Also, when cosmic rays (protons) collide with the upper atmosphere (or atmolayer), are we travelling into them, or are they travelling towards us?
Title: Re: The Math for universal Acceleration IS INCORRECT
Post by: JSS on March 31, 2020, 05:46:54 PM
So according to UA, any outside observers will see the earth travelling near the speed of light and also see our clocks ticking very slowly. Is that correct?

If UA is following the laws of relativity then yes, an outside observer would see earth traveling at near lightspeed if it's been in flight for thousands let alone millions or billions of years.  However with such extreme speed you would likely not be able to make out much at all before it passed with all the light piled up in front, and the light coming back would be so red shifted as to be nearly undetectable.

Also, when cosmic rays (protons) collide with the upper atmosphere (or atmolayer), are we travelling into them, or are they travelling towards us?

It's all relative, pun intended.  If you were riding on one of those particles you would feel like you were standing still as a massive planet came flying at you.  The video I linked gives a good explanation and examples of how two people can both see the other as moving/shorter/slower at the same time. Frankly I wouldn't want to be anywhere near a planet sized object moving at that speed, the radiation would likely sterilize anything even close to it's flight path. Cosmic rays are insanely dangerous if you are in space, and can even kill you (cancer) down here if you get hit by one and are unlucky, and those are just single electrons or protons and not moving anywhere near as fast as a UA earth would be.

The key to understanding relativity is that there is NO SUCH THING as a universal reference frame. There is no "place" that planets or galaxies are put into. You can't ever look at two things and determine which is moving and which is standing still, every reference frame is equally valid. You just pick one and say "For this observation we are using this frame of reference as out viewpoint."  It's hard to do, as we live our whole lives with a single frame of reference, the Earth. It's easy to say this building is still and this car is moving, but what we mean is "That car is moving in reference to the surface of the earth."
Title: Re: The Math for universal Acceleration IS INCORRECT
Post by: Groit on March 31, 2020, 06:33:01 PM
Ok, so why do atomic clocks on satellites in orbit tick more slowly than the clocks on the surface of the Earth? what you're saying is that from the satellites frame, the clock on Earth would tick more slowly as well, and they would both cancel out, but that's not what we observe or measure? 
Title: Re: The Math for universal Acceleration IS INCORRECT
Post by: JSS on March 31, 2020, 09:23:28 PM
Ok, so why do atomic clocks on satellites in orbit tick more slowly than the clocks on the surface of the Earth? what you're saying is that from the satellites frame, the clock on Earth would tick more slowly as well, and they would both cancel out, but that's not what we observe or measure?

GPS satellites have an added complication.  Their speed makes time go slightly slower, but they are further away from the Earth's gravity well which makes time run slightly faster. 

But we do observe exactly what Einstein predicts, otherwise GPS wouldn't work at all.

Watch the video I posted, it explains how two observers can BOTH see the other as being slower, each measuring the other and both coming up with different results, both being correct from THEIR frame of reference.  Time does not work the way one expects once you start messing around with relativity.  In fact, it's entirely possible for two observers to observe far away events and see them happen in a different order. Who is right?  Again, both... different frames of reference (speed or gravity) will produce different observations. 

It's very weird, but has been proven over and over again in thousands of experiments.  Relativity is one of the most tested theories out there, and has never failed to work exactly as predicted.  So far.  But that's science, when someone finally does break it, we will just modify it and have an even better and more accurate theory.
Title: Re: The Math for universal Acceleration IS INCORRECT
Post by: Groit on March 31, 2020, 09:34:47 PM
Time slows down for accelerating frames and this is not what is observed when we measure the life time of Muons passing through the atmosphere. Special relativity only applies in inertial frames, Earth is considered an inertial frame, and we can measure the effects of SR with the Muons which are created from cosmic rays.

This explains it:

Quote
There is a type of fundamental particle called a muon, which in many ways resembles an electron. The main differences are the fact that muons are about two hundred times more massive than electrons, and that they decay into their less massive counterparts after a mean lifetime of a few microseconds. Muons can be created in the Earth’s upper atmosphere when high-energy protons from outer space, known as cosmic rays, collide with the atoms they encounter in the upper atmosphere. The muons so generated travel predominantly in a downwards direction, towards the Earth’s surface, at a speed close to the speed of light.

 Experiments show the following result, which at first sight is quite puzzling. At the top of a mountain the number of muons arriving per hour can be measured and recorded. At the bottom of the mountain, say a couple of thousand metres lower down, the number of muons arriving per hour can also be measured and recorded. Since the muons take several microseconds to travel from the top of the mountain to the bottom, a certain fraction of them should have decayed along the way, meaning that less will be detected at the bottom than at the top. In fact, knowing the mean lifetime of a muon (from measurements in the laboratory), one can predict how many muons should be recorded at the bottom of the mountain as a percentage of those recorded at its top.

 And here’s the puzzle: in every case when such an experiment is performed, the number of muons recorded at the bottom of the mountain is far higher than predicted. In other words, far fewer muons have decayed than might be expected. So what’s going on? The crucial fact here is that the muons are travelling close to the speed of light. Einstein’s specialtheory ofrelativity implies that, according to the observer measuring the rate of arrival of the muons, time is passing more slowly for the muons. Consequently, the journey from the top of the mountain to the bottom takes the muons less time than measured by the observer, and more of the muons survive the trip. This result is an example of one of the key concepts of Albert Einstein’s special theory of relativity. It may be paraphrased by the statement ‘moving clocks run slow’.

Title: Re: The Math for universal Acceleration IS INCORRECT
Post by: JSS on April 01, 2020, 10:50:42 AM
Time slows down for accelerating frames and this is not what is observed when we measure the life time of Muons passing through the atmosphere. Special relativity only applies in inertial frames, Earth is considered an inertial frame, and we can measure the effects of SR with the Muons which are created from cosmic rays.

This explains it:

Quote
There is a type of fundamental particle called a muon, which in many ways resembles an electron. The main differences are the fact that muons are about two hundred times more massive than electrons, and that they decay into their less massive counterparts after a mean lifetime of a few microseconds. Muons can be created in the Earth’s upper atmosphere when high-energy protons from outer space, known as cosmic rays, collide with the atoms they encounter in the upper atmosphere. The muons so generated travel predominantly in a downwards direction, towards the Earth’s surface, at a speed close to the speed of light.

 Experiments show the following result, which at first sight is quite puzzling. At the top of a mountain the number of muons arriving per hour can be measured and recorded. At the bottom of the mountain, say a couple of thousand metres lower down, the number of muons arriving per hour can also be measured and recorded. Since the muons take several microseconds to travel from the top of the mountain to the bottom, a certain fraction of them should have decayed along the way, meaning that less will be detected at the bottom than at the top. In fact, knowing the mean lifetime of a muon (from measurements in the laboratory), one can predict how many muons should be recorded at the bottom of the mountain as a percentage of those recorded at its top.

 And here’s the puzzle: in every case when such an experiment is performed, the number of muons recorded at the bottom of the mountain is far higher than predicted. In other words, far fewer muons have decayed than might be expected. So what’s going on? The crucial fact here is that the muons are travelling close to the speed of light. Einstein’s specialtheory ofrelativity implies that, according to the observer measuring the rate of arrival of the muons, time is passing more slowly for the muons. Consequently, the journey from the top of the mountain to the bottom takes the muons less time than measured by the observer, and more of the muons survive the trip. This result is an example of one of the key concepts of Albert Einstein’s special theory of relativity. It may be paraphrased by the statement ‘moving clocks run slow’.

I'm not sure what you're arguing here. Time slows down to an outside observer looking at a reference frame that is moving faster.  Since all reference frames are relative, both sides will see the other slow down, both sides think they are the ones at rest.

You said this is not what is observed but that's exactly what happens in the quote you provided. From Earth's frame of reference, the Muons are moving close to the speed of light, and time is running slower for them, letting more of them reach the surface than they should.
Title: Re: The Math for universal Acceleration IS INCORRECT
Post by: pricelesspearl on April 01, 2020, 02:37:06 PM
Quote
Watch the video I posted, it explains how two observers can BOTH see the other as being slower, each measuring the other and both coming up with different results, both being correct from THEIR frame of reference.
 

That is only true if both clocks are in an inertial reference frame, then special relativity applies. If one clock is accelerating and the other isn’t, the accelerating clock will run slower relative to the inertial clock, but both clocks will not run slower relative to one another. In the twin paradox, the traveling twin comes back younger. The traveling twin accelerates, the earth twin does not.

Quote
You said this is not what is observed but that's exactly what happens in the quote you provided. From Earth's frame of reference, the Muons are moving close to the speed of light, and time is running slower for them, letting more of them reach the surface than they should

The muons are accelerating, the earth is not.


Title: Re: The Math for universal Acceleration IS INCORRECT
Post by: Groit on April 01, 2020, 03:46:36 PM


I'm not sure what you're arguing here. Time slows down to an outside observer looking at a reference frame that is moving faster.  Since all reference frames are relative, both sides will see the other slow down, both sides think they are the ones at rest.

You said this is not what is observed but that's exactly what happens in the quote you provided. From Earth's frame of reference, the Muons are moving close to the speed of light, and time is running slower for them, letting more of them reach the surface than they should.

What I'm trying to say is that the scientists who measure the effects of time dilation for the muons use special relativity in their calculations, and special relativity only works in inertial frames of reference. So that means the Earth's frame of reference is inertial (not accelerating) and the muons frame of reference is inertial (not accelerating, the muons are travelling at a constant speed).

If the Earth was accelerating then the scientists would need to use more rigorous calculations from 'general relativity' for accelerating frames of reference. We know this is not what they're doing, therefore the Earth and the muons must be in 'inertial frames of reference', otherwise SR wouldn't work.

Also, as pricelesspearl pointed out, when one frame is accelerating and the other isn't, time always runs slower for the accelerating frame for both observers. So if the Earth was accelerating like FET suggests, then the scientist would actually measure less muons at the bottom of the mountain because the muons clock would be running faster. 
Title: Re: The Math for universal Acceleration IS INCORRECT
Post by: Groit on April 01, 2020, 03:56:08 PM

The muons are accelerating, the earth is not.

The muons are not accelerating, the Earth and the muons are in inertial frames of reference.
Title: Re: The Math for universal Acceleration IS INCORRECT
Post by: JRowe on April 01, 2020, 04:01:25 PM
The Earth is not in an inertial frame of reference under RET or the UA FET this thread is about.
Title: Re: The Math for universal Acceleration IS INCORRECT
Post by: Groit on April 01, 2020, 05:22:46 PM
The Earth is not in an inertial frame of reference under RET or the UA FET this thread is about.


An inertial frame of reference is a frame of reference in which Newton's first law is very nearly correct, and the Earth is in one of those frames.

Quote
So your earthbound personal frame of reference is not, strictly speaking, inertial, nor is any other frame of reference fixed on the Earth. However, the acceleration involved is very small compared with those arising from everyday forces, and its effect is negligible for most everyday observations. Other terrestrial and solar motions involve even smaller accelerations. The Earth’s acceleration due to its orbital motion around the Sun is about an order of magnitude smaller than that due to its axial spinning, whereas the Sun’s acceleration towards the centre of our galaxy is about 3×10^−10 m/s^2. Thus, for most purposes a frame of reference fixed on the Earth provides a very good approximation to an inertial frame. Although a truly inertial frame is something of an idealisation, the non-existence of an ideal inertial frame is not really a problem because there are plenty of frames of reference that are very nearly inertial (i.e. there are plenty of frames of reference in which Newton’s first law is very nearly correct).
Title: Re: The Math for universal Acceleration IS INCORRECT
Post by: pricelesspearl on April 01, 2020, 05:31:28 PM
The Earth is not in an inertial frame of reference under RET or the UA FET this thread is about.

In RE, earth is considered inertial as a practical matter.  Any inertial effects on the surface are minor enough not to be considered and the center of the earth would be considered inertial in GR because it is in freefall.

If the earth is accelerating in FET, then by definition, it is not inertial.
Title: Re: The Math for universal Acceleration IS INCORRECT
Post by: pricelesspearl on April 01, 2020, 05:33:05 PM

The muons are accelerating, the earth is not.

The muons are not accelerating, the Earth and the muons are in inertial frames of reference.

Yes, you are correct...got my self turned around.
Title: Re: The Math for universal Acceleration IS INCORRECT
Post by: ImAnEngineerToo on April 02, 2020, 01:38:09 AM
I’m going to attempt to rectify your broken rhetoric, icanactuallythink...

I think what you are saying is that if the reference frame is to be held at a constant velocity (inertial change of zero), and an acceleration occurs, then the body under some constant or varying acceleration in one direction will not have the same velocity in two different points, as the UA theory seems to indicate?

I believe a FE response, if I understand UA well enough hopefully, is that the reference frame is also accelerating with the earth in time, allowing the earth to net zero inertial energy by accelerating within the accelerating reference frame. Pete, correct me if I‘m wrong.
Title: Re: The Math for universal Acceleration IS INCORRECT
Post by: pricelesspearl on April 03, 2020, 05:15:30 AM
I’m going to attempt to rectify your broken rhetoric, icanactuallythink...

I think what you are saying is that if the reference frame is to be held at a constant velocity (inertial change of zero), and an acceleration occurs, then the body under some constant or varying acceleration in one direction will not have the same velocity in two different points, as the UA theory seems to indicate?

I believe a FE response, if I understand UA well enough hopefully, is that the reference frame is also accelerating with the earth in time, allowing the earth to net zero inertial energy by accelerating within the accelerating reference frame. Pete, correct me if I‘m wrong.

All this talk about frame of reference is irrelevant. Yes, in SR you can accelerate ad infinitum without ever reaching c…but it has nothing to do with frame of reference.  Yes, an outside observer in an inertial FoR will see the object slow and contract.   But the object will never reach c whether there is an inertial outside observer or not. It won’t reach c in the accelerating frame of reference either.  In no frame of reference will anybody ever see anything exceed c...at least with current technology.
Title: Re: The Math for universal Acceleration IS INCORRECT
Post by: Groit on April 03, 2020, 10:01:50 AM
So, in FET, is the Earth in an inertial frame or a non-inertial frame?
Title: Re: The Math for universal Acceleration IS INCORRECT
Post by: Pete Svarrior on April 03, 2020, 02:35:49 PM
So, in FET, is the Earth in an inertial frame or a non-inertial frame?
This question is nonsensical, and reveals an elementary misunderstanding of what frames of reference are.

There is no such thing as an objective frame of reference - you can consider either, usually whichever one makes your calculations easier at the time.
Title: Re: The Math for universal Acceleration IS INCORRECT
Post by: Groit on April 03, 2020, 04:52:49 PM
You can't have it both ways Pete, it doesn't work like that. It's like saying "the shape of the Earth is round and flat"  ::)
Title: Re: The Math for universal Acceleration IS INCORRECT
Post by: pricelesspearl on April 03, 2020, 07:51:45 PM
Quote
There is no such thing as an objective frame of reference

That is only true in GR. In SR acceleration in absolute.  And even in GR, only coordinate acceleration is relative.  Proper acceleration is always absolute. It isn’t necessary to relate your acceleration to some outside reference.  An accelerometer within your own reference would show acceleration or not.
Title: Re: The Math for universal Acceleration IS INCORRECT
Post by: Pete Svarrior on April 04, 2020, 05:58:13 PM
You can't have it both ways Pete, it doesn't work like that.
It absolutely does. I strongly suggest that you familiarise yourself with how you can transform between frames of reference. It's absolutely essential knowledge for this debate, and something that was likely explained to you in high school.

Proper acceleration is always absolute.
But of course it's relative to something. There is no such thing as an objective frame of reference. Any statement that implies otherwise is a non-starter. I'll just refer you to Wikipedia (https://en.wikipedia.org/wiki/Proper_acceleration), since we already know you're an obvious troll. You only need to go in as deep as two sentences to correct your error, but I do suggest you read on.
Title: Re: The Math for universal Acceleration IS INCORRECT
Post by: Groit on April 04, 2020, 08:22:51 PM
You can't have it both ways Pete, it doesn't work like that.
It absolutely does. I strongly suggest that you familiarise yourself with how you can transform between frames of reference. It's absolutely essential knowledge for this debate, and something that was likely explained to you in high school.

I'm sorry Pete, but if the Earth is accelerating in a straight line like FET states, then it's in a non-inertial frame. In order to make the frame inertial then you would need to account for the 'Fictitious force' which would be f = -ma.

And since the scientists don't need to compensate for any 'Fictitious forces' when measuring the muons that reach the surface of the Earth, then we know that the Earth is not accelerating and our frame of reference is inertial.

https://en.wikipedia.org/wiki/Fictitious_force

Quote
Detection of non-inertial reference frame

Observers inside a closed box that is moving with a constant velocity cannot detect their own motion; however, observers within an accelerating reference frame can detect that they are in a non-inertial reference frame from the fictitious forces that arise. For example, for straight-line acceleration Vladimir Arnold presents the following theorem:[12]

In a coordinate system K which moves by translation relative to an inertial system k, the motion of a mechanical system takes place as if the coordinate system were inertial, but on every point of mass m an additional "inertial force" acted: F = −ma, where a is the acceleration of the system K.
Title: Re: The Math for universal Acceleration IS INCORRECT
Post by: Pete Svarrior on April 04, 2020, 08:27:58 PM
I'm sorry Pete, but if the Earth is accelerating in a straight line like FET states, then it's in a non-inertial frame.
You can be sorry all you want, but this statement continues to be nonsensical, to the point where you can't call it true or false. You can consider any physical scenario in any frame of reference. That's pretty much the point of relativity. Now, it may be more intuitive for you to consider it in a non-inertial FoR (and I'd be inclined to agree), but you're immediately going to run into issues with fellas like pricesspearl who will then mix up multiple frames to make their point.

You might also mean that the Earth's own FoR is a non-intertial FoR, or that it is stationary with regard to a non-inertial FoR rather than that it is in one. Those statements make some sense, and are true, but is largely useless.

I can't help you here. You're gonna have to brush up on basic physics. This is not a FE vs RE issue.
Title: Re: The Math for universal Acceleration IS INCORRECT
Post by: pricelesspearl on April 04, 2020, 10:14:56 PM
Quote
But of course it's relative to something. There is no such thing as an objective frame of reference. Any statement that implies otherwise is a non-starter. I'll just refer you to Wikipedia (https://en.wikipedia.org/wiki/Proper_acceleration), since we already know you're an obvious troll. You only need to go in as deep as two sentences to correct your error, but I do suggest you read on.

Wikipedia says “Proper acceleration contrasts with coordinate acceleration, which is dependent on choice of coordinate systems and thus upon choice of observers.”
I’ll translate for you.  Proper acceleration is different than coordinate acceleration.  Coordinate acceleration is relative, proper acceleration is not.

Don’t really see how that is different from “only coordinate acceleration is relative.  Proper acceleration is always absolute”, which is what I said.

Quote
The reason for this is that an observer can always tell what their acceleration is without referring to anything external. All you have to do is drop something and see what happens. If you're not accelerating the dropped object will just hover next to you. If you are accelerating then the dropped object will accelerate away from you

The acceleration measured in this way is called the proper acceleration and is an important concept in general relativity. All observers everywhere will agree on the value of your proper acceleration, making it an absolute not a relative quantity.

https://physics.stackexchange.com/questions/366301/is-acceleration-relative-in-classical-physics

Quote
In his special theory of relativity, the relativity of motion is implemented only for inertial motions. It does not extend to accelerated motions. They are motions that change their speed or direction or both. We can still say that something accelerates without adding a further qualification "with respect to...". Thus, acceleration is an absolute for Einstein's special theory.

http://www.pitt.edu/~jdnorton/teaching/HPS_0410/chapters/Special_relativity_principles/

Quote
This chapter focuses on acceleration, which, unlike velocity, can be detected by a body in motion and measured within the moving system. Acceleration is also more fundamental; it is absolute, whereas velocity is relative and depends on the frame of reference chosen

http://physics.gu.se/~f3aamp/edu/acceleration.html

In FET, in earth’s frame of reference, acceleration creates “gravity”.   We can objectively determine within our own frame that earth is accelerating because “gravity” exists.  What some other person, in some other frame of reference perceives doesn’t change our perception that the earth is accelerating because “gravity” exists.  Therefore, earth's frame of reference is objective with respect to its acceleration.

Or are you arguing that in FET “gravity” only exists on earth if some other person, in some other frame of reference perceives the earth to be accelerating?
Title: Re: The Math for universal Acceleration IS INCORRECT
Post by: Pete Svarrior on April 05, 2020, 09:12:58 AM
Don’t really see how that is different from “only coordinate acceleration is relative.  Proper acceleration is always absolute”, which is what I said.
Yes, that is the problem, as usual. You took an article which explains that proper acceleration is measured relative to a well-defined FoR and chose to "translate" it into "well uhh duh it agrees with me". You then did your all-time classic of spamming questionable sources that you think agree with you because they used a word you like while ignoring their meaning.

But we already know you're trolling, so we won't be wasting time here. Your error has been corrected. Better luck next time.
Title: Re: The Math for universal Acceleration IS INCORRECT
Post by: pricelesspearl on April 05, 2020, 09:55:40 PM
Quote
Yes, that is the problem, as usual. You took an article which explains that proper acceleration is measured relative to a well-defined FoR and chose to "translate" it into "well uhh duh it agrees with me". You then did your all-time classic of spamming questionable sources that you think agree with you because they used a word you like while ignoring their meaning.

“ It is thus acceleration relative to a free-fall, or inertial, observer who is momentarily at rest relative to the object being measured”…your “gotcha” is nothing more than a different way of stating my exact point.

If an inertial observer were to see the earth accelerating upwards in FET…what rate of acceleration would he perceive?  What rate of acceleration would the people on earth perceive? They both would perceive 9.8 m/s2.

And since the whole point of SR is that any experiment performed in any inertial frame of reference will have the same results….in every single inertial frame of reference, the earth’s acceleration will always be measured as 9.8 m/s2…and it will always measure the same as within the earth’s frame.  That is what makes it absolute.  It doesn’t change dependent on the FoR.

In SR, acceleration of earth will always equal 9.8 m/s2 in any inertial frame.  In SR, Acceleration of earth with it's own frame will always equal 9.8 m/s2.  Therefore in, SR, the acceleration of earth is not frame dependent.  It would behoove you to think through logical inferences and use some critical thinking skills before you jump to conclusions about the point I am making.  Then I wouldn’t have to translate.

I’m not sure that a paper published on a respected university’s website, written by a Distinguished Professor of that University, should be considered “questionable”…but whatever.
Title: Re: The Math for universal Acceleration IS INCORRECT
Post by: Pete Svarrior on April 06, 2020, 10:23:12 AM
your “gotcha” is nothing more than a different way of stating my exact point.
You describe simple physics as "gotchas" and "your exact point" is that something stops being relative when it's relative to a specific FoR. Nobody is going to fall for it.

If an inertial observer were to see the earth accelerating upwards in FET…what rate of acceleration would he perceive?  What rate of acceleration would the people on earth perceive? They both would perceive 9.8 m/s2.
This continues to be a completely incorrect assumption. We've explained this to you many times. I do not believe for a moment that you've failed to understand it, given that your main schtick here is pretending not to understand English.

Your failure, and one you could have once again solved by reading Wikipedia (https://en.m.wikipedia.org/wiki/Acceleration_(special_relativity)#Proper_acceleration), is that you assume that there is one inertial frame of reference with regard to which you can consider FE's acceleration for an extended period of time. This is incorrect, and was pointed out to you time and time again by FE'ers and RE'ers alike. At any point in time, you can identify an inertial FoR with regard to which the acceleration will be 9.81ms^-2 for an infinitesimal length of time. That is to say, the Earth would not be immediately moving relative to that frame. A moment later, this would no longer hold. Yet your objection continues to assume that it would.

The problem with trying to rely solely on academic papers is that it requires the reader to be able to follow what's being said. You can't (or, rather, pretend not to be able to) follow the basics of how a spirit level works, so you desperately cling to keywords which you think help your case.

Stop trying to derail this thread with some of the most transparent trolling this forum has seen in its decades of operation. This is the last time I will ask so politely.
Title: Re: The Math for universal Acceleration IS INCORRECT
Post by: pricelesspearl on April 06, 2020, 01:40:46 PM
[quote[Your failure, and one you could have once again solved by reading Wikipedia, is that you assume that there is one inertial frame of reference with regard to which you can consider FE's acceleration for an extended period of time. This is incorrect, and was pointed out to you time and time again by FE'ers and RE'ers alike. At any point in time, you can identify an inertial FoR with regard to which the acceleration will be 9.81ms^-2 for an infinitesimal length of time. That is to say, the Earth would not be immediately moving relative to that frame. A moment later, this would no longer hold. Yet your objection continues to assume that it would.[/quote]

And in every single one of those inertial frames, for any length of time, the acceleration will be the same.  Proper acceleration is Lorentz-Invariant...the same in all inertial frames of reference.

You are confusing and velocity and acceleration.  That same confusion is evident in your wiki.

 
Quote
Relative to an inertial observer in the universe, however, the Earth's acceleration decreases as the its velocity approaches c.

It isn't the acceleration that decreases, it is the velocity.  Somebody needs to learn how to add relativistic velocities, then they would understand that in SR, every interval of acceleration increases the velocity a little less. The rate of acceleration remains the same, but the velocity will never reach c. The conclusion on the wiki is correct, but the reasoning behind it is wrong. 

According to FET, gravity and acceleration are the same thing.  We have objectively quantified gravity...so by FET's own definition, we have objectively quantified its rate of acceleration.
Title: Re: The Math for universal Acceleration IS INCORRECT
Post by: Pete Svarrior on April 06, 2020, 09:27:11 PM
It isn't the acceleration that decreases, it is the velocity.
And what do we call a decrease in velocity?

every interval of acceleration increases the velocity a little less
lol, it's almost as if your trolling got even more transparent.

And in every single one of those inertial frames, for any length of time, the acceleration will be the same.
That is the opposite of what "being true for only an infitniesimal length of time" means. In other words, that is the opposite of what mainstream physics states. As with your previous failures, your disagreement is fundamentally with the model you're supposed to champion.

Quote
Relative to an inertial observer in the universe, however, the Earth's acceleration decreases as the its velocity approaches c.

It isn't the acceleration that decreases, it is the velocity.
Ah, of course. As the velocity approaches c, the velocity decreases. And how does it approach c while decreasing?
Title: Re: The Math for universal Acceleration IS INCORRECT
Post by: pricelesspearl on April 07, 2020, 03:20:40 AM
Quote
In other words, that is the opposite of what mainstream physics states

Mainstream physics says that proper acceleration is Lorentz Invariant (says it right on the wiki page you linked), which by definition means it is the same in any and every inertial frame of reference.

Quote
Ah, of course. As the velocity approaches c, the velocity decreases. And how does it approach c while decreasing?

The short answer is that relativistic velocities can’t be added in the classical sense.

Quote
What this shows is that the principle of relativity prohibits us adding velocities in the usual way. We cannot add velocities by the ordinary rule 100,000 + 100,000 = 200,000. More generally, the classical rule for the composition of velocities fails…

In its place we need a new rule for the composition of velocities. It ought to look like the ordinary rule as long as velocities are small--we do know that the ordinary rule works for slow moving things like cars on freeways and trains. But it must look very different at high speeds. If we use it to add two velocities close to light, we must get a resultant that is still less than the velocity of light. Einstein found that the principle of relativity forces a particular rule. For the case of velocities oriented in the same direction in space, the relativistic rule for composition of velocities is: velocity of A with respect to C =Velocity of A with respect to B+Velocity of B with respect to C / reduction factor. Click here to see the complete formula.

The full explanation can be found here https://www.pitt.edu/~jdnorton/teaching/HPS_0410/chapters/Special_relativity_adding/index.html, but the conclusion is

Quote
The total speed of the last boosted machine increases as we proceed along the sequence "I,""II," etc. But the increases become smaller and smaller…. No matter how often we add 100,000 miles per second, we never get past the speed of light--here set at exactly 186,000 miles per second. We get closer and closer to it. But never past it.

IOW, when the proper formula is used to add relativistic velocities, the rate of acceleration can remain constant, but the increases in velocity gets smaller.

Hopefully, a book written by a Distinguished Professor at the University of Pittsburg, who is considered an authority on Einstein and highly regarded in his field is mainstream enough for you. If not, there is a good, but more technical explanation, starting on page 106 of the standard text Space Time Physics, cowritten by Archibald Wheeler and Edwin Taylor. If their credentials aren’t good enough for you…there’s not much more I can say. You can download the book here.  I can’t cut and paste excerpts from it and it is simply too much information to try and condense.
http://www.eftaylor.com/spacetimephysics/0000_spacetime_physics.pdf

Title: Re: The Math for universal Acceleration IS INCORRECT
Post by: Pete Svarrior on April 07, 2020, 10:47:31 AM
Mainstream physics says that proper acceleration is Lorentz Invariant (says it right on the wiki page you linked), which by definition means it is the same in any and every inertial frame of reference.
Well, no, it doesn't. It says that "In infinitesimal small durations there is always one inertial frame, which momentarily has the same velocity as the accelerated body, and in which the Lorentz transformation holds."

Infinitesimal durations. There is always [at least] one. This directly contradicts your claim that it holds at any time in any inertial frame.

The short answer is that relativistic velocities can’t be added in the classical sense.
Oh, not at all. That would explain why the rate of change of velocity (what do we call that?) is decreasing. You claim that it is velocity that's decreasing as it approaches c.

Let me spell this out for you. Consider an object whose velocity is less than c in a FoR of your choice. We know that the body's velocity is approaching c. Please explain how you can claim that its velocity is decreasing, as you just did.

Once again, it doesn't matter how many authorities you appeal to when you use them to say things that make no sense. You (pretend to) struggle to understand the text you're reading, and your interpretation of it is not the same as what these authorities are telling you.

Finally, could you outline your objection to our wiki? The transformations are clearly written out for three-acceleration, not proper acceleration, yet for some reason you're only focusing on butchering the latter. It sounds to me like you've made a lot of assumptions about what's being proposed. Again.
Title: Re: The Math for universal Acceleration IS INCORRECT
Post by: JSS on April 07, 2020, 11:07:59 AM
I'm not going to get into the argument over velocity vs acceleration but did wand to point out a thought I had.

One of the sad things about Einsteins theories is it seems to make FTL travel impossible, mostly due to time-travel as a side effect no matter how you travel.

But Flat Earth + Universal Acceleration has an advantage here, in that you can simply set the Earth as a preferred frame of reference which solves all kinds of problems and limitations, including allowing FTL.

Nothing in general or special relativity prohibits having a preferred frame of reference, there just isn't any reason to pick one point in the universe over another in mainstream physics.

But if the Earth is the center of the universe and special in FE theory, then it seems like an obvious choice to make it the preferred reference frame too.

Seems like a good fit to me, and thought I'd make the suggestion.
Title: Re: The Math for universal Acceleration IS INCORRECT
Post by: Pete Svarrior on April 07, 2020, 11:12:10 AM
That wouldn't make it FTL in any FoR, still...
Title: Re: The Math for universal Acceleration IS INCORRECT
Post by: Groit on April 07, 2020, 11:47:19 AM
Well, no, it doesn't. It says that "In infinitesimal small durations there is always one inertial frame, which momentarily has the same velocity as the accelerated body, and in which the Lorentz transformation holds."

The length of time for the muons to decay is approx 2.2 x 10^-6 s (or 2.2 us). Are you saying that during this short time period, In FET, the Earth is in an inertial frame where SR holds?
Title: Re: The Math for universal Acceleration IS INCORRECT
Post by: Pete Svarrior on April 07, 2020, 01:00:58 PM
Are you saying that during this short time period, In FET, the Earth is in an inertial frame where SR holds?
No.

Once again, you need to find out what frames of reference are. The Earth isn't "in" one - that would imply the existence of an objective FoR. You can frame any physical situation in any frame of reference.

You should also not attribute Wikipedians words to me, unless I specifically wrote them. It's a disservice to their contributors.
Title: Re: The Math for universal Acceleration IS INCORRECT
Post by: JSS on April 07, 2020, 03:08:32 PM
That wouldn't make it FTL in any FoR, still...

It wouldn't directly allow it, no.  You would still need wormholes, warp drive, hyperspace or some other unknown method to get to FTL in the first place. All of which are still very theoretical (if not outright based on some questionable math) anyway.

Having a special reference frame would however allow you to eliminate any relativity based time travel paradoxes that any FTL method would introduce, as you could resolve them by that preferred reference frame acting as a resolver of any conflicts.
Title: Re: The Math for universal Acceleration IS INCORRECT
Post by: Pete Svarrior on April 08, 2020, 12:52:58 PM
Having a special reference frame would however allow you to [do stuff]
That is absolutely not how frames of reference work. You cannot create a FoR in which the laws of physics are miraculously broken. A FoR is just a frame from which you observe a [hypothetical] scenario. It does not change the scenario.
Title: Re: The Math for universal Acceleration IS INCORRECT
Post by: JSS on April 08, 2020, 02:39:10 PM
Having a special reference frame would however allow you to [do stuff]
That is absolutely not how frames of reference work. You cannot create a FoR in which the laws of physics are miraculously broken. A FoR is just a frame from which you observe a [hypothetical] scenario. It does not change the scenario.

Agreed, let me clarify.

It's not that a special reference frame lets you do stuff, it solves one of many problems preventing it.  Without a special or preferred frame, any kind of FTL travel no matter how you do it will allow time travel paradoxes. Among other advantages, a preferred frame solves the time travel problem by providing a single reference point to order events by and eliminating the possibility of sending information backwards in time.

How you perform your FTL travel is another subject entirely, and it still may not be possible (which is either very bad or very good for our future as a species), but at least with a preferred frame you eliminate one of the major issues with it.
Title: Re: The Math for universal Acceleration IS INCORRECT
Post by: pricelesspearl on April 08, 2020, 03:32:55 PM
Quote
Well, no, it doesn't. It says that "In infinitesimal small durations there is always one inertial frame, which momentarily has the same velocity as the accelerated body, and in which the Lorentz transformation holds."

Saying that velocity is momentarily the same is not the same thing as saying that the rate of acceleration is momentarily the same.  Two cars can both be going 20mph at any given moment when one is accelerating at 10mph and the other at 5mph.

What’s more …an inertial observer always has a proper acceleration of zero.  Are you suggesting that there is some infinitesimal small duration of time, in some inertial frame where Earth’s acceleration is zero?

Quote
You claim that it is velocity that's decreasing as it approaches c.

You’re right, I should have been more precise and said that its velocity is decreasing relative the rate of acceleration.

Quote
It sounds to me like you've made a lot of assumptions about what's being proposed.

I’m just reading the wiki text and the way I read it says that the earth can have a proper acceleration of 1g forever and not reach c because from an inertial frame of reference the proper acceleration will appear to decrease at increasing rates and therefore an inertial observer will never see earth’s velocity reach the speed of light.

Is that a correct interpretation?  If it is, that explanation is wrong.  The reason an inertial observer never sees earth’s velocity reach the speed of light is because it never reaches the speed of light in the accelerating frame, even though it continues to accelerate at the same rate.  How and why that can be true is clearly explained, even simplistically, in the sources I cited. Because the formula for adding velocities must take time dilation into account, an object can accelerate ad infinitum without the total velocity ever exceeding c.  IOW, accelerating 100,000 mph will add less than 100,000 mph to the total velocity. Check the formula for yourself and see how it works out if you doubt it.

If that’s the correct interpretation, it also means that even though the proper acceleration will appear to decrease from an initial observer, it doesn’t decrease within the earth’s frame of reference, as measured by an accelerometer, and within its own frame, Earth would reach c.
Title: Re: The Math for universal Acceleration IS INCORRECT
Post by: Pete Svarrior on April 09, 2020, 08:03:15 PM
Saying that velocity is momentarily the same is not the same thing as saying that the rate of acceleration is momentarily the same.
Indeed. I take it you've realised your error, then?
Title: Re: The Math for universal Acceleration IS INCORRECT
Post by: pricelesspearl on April 10, 2020, 05:56:09 PM
Quote
Indeed. I take it you've realised your error, then?

This as is as simple as I can make it…if you don’t understand this, there is not any more I can say.

An accelerometer is a device that measures proper acceleration. If the earth always, at every single moment, for every infinitesimal duration, has a proper acceleration of 9.81 m/s2, an accelerometer will read 9.81 m/s2 at every single moment for every infinitesimal duration.

An inertial frame always has a proper acceleration of zero at every single moment, for every infinitesimal duration, so an accelerometer will always read zero at every single moment, for every infinitesimal duration.

Therefore the “relative” proper acceleration between the earth and an inertial observer will always be 9.81 m/s2 at every single moment, for every infinitesimal duration and an inertial observer will observe it accelerating at 9.81 m/s2  for every single, moment, for every infinitesimal duration.

The Earth’s proper acceleration is absolute, it doesn’t change as perceived from an inertial frame. Earth’s acceleration is objective.  It can be determined from within its own frame of reference, all you need to do is look at an accelerometer.

Earth is always in accelerating frame of reference, according to FET. 
Title: Re: The Math for universal Acceleration IS INCORRECT
Post by: Pete Svarrior on April 10, 2020, 11:12:25 PM
This as is as simple as I can make it…if you don’t understand this, there is not any more I can say.
You can't say much in general. You continue to change your mind on things and contradict yourself. You know, things like "as the Earth's velocity increases, the Earth's velocity decreases. Also, those changes in velocity are not acceleration." All I'm asking for you to do is stop doing that in the upper, since I do not for a moment believe you're arguing sincerely.

In short: the problem is not that I don't understand what you're saying. I do. You just end up contradicting simple physics every time you open your mouth. This is why you get thoroughly mocked by RE'ers and FE'ers alike.

The Earth’s proper acceleration
A friendly reminder that no part of the Wiki refers to proper acceleration. Several people pointed this out to you at different points in time. Your fixation here is not useful.

Earth is always in accelerating frame of reference, according to FET.
This continues not to be the case. You can consider any situation from any FoR. There is no such thing as an objective FoR, and it is nonsensical to claim that something "is in" a FoR. Every time you use this phrasing, you remind us that you (are pretending to) have no idea what you're doing.
Title: Re: The Math for universal Acceleration IS INCORRECT
Post by: pricelesspearl on April 11, 2020, 12:43:34 AM
Quote
You can consider any situation from any FoR.

That’s true, but from whatever inertial frame of reference you perceive the earth, its relative acceleration will always be 9.81 m/s2  .  The earth’s acceleration will always be 9.81 m/s2, from its own FoR.  Acceleration in any inertial frame will always be zero from it’s own FoR.  From any inertial frame, earth will always be accelerating at 9.81 m/s2  .  And from earth, an inertial frame will always be accelerating at zero.

Quote
A friendly reminder that no part of the Wiki refers to proper acceleration.

If FET did not consider earth’s acceleration to be proper acceleration, there could not be any “gravity” caused by UA. Proper acceleration is the “physical acceleration (i.e., measurable acceleration as by an accelerometer) experienced by an object.”   If the earth wasn’t physically experiencing proper acceleration of 9.81 m/s2  , there  could not the physical effect of “gravity” caused by the acceleration. 
Title: Re: The Math for universal Acceleration IS INCORRECT
Post by: Pete Svarrior on April 11, 2020, 11:01:54 AM
That’s true, but from whatever inertial frame of reference you perceive the earth, its relative acceleration will always be 9.81 m/s2
This directly contradicts your prior statement, in which there is one inertial FoR in which that applies for a given moment. This is the usual problem with your claims - you keep disagreeing with yourself.

Also, "relative acceleration"? Relative to what?

If FET did not consider earth’s acceleration to be proper acceleration
You are once again (pretending to be) hopelessly confused. The Earth's acceleration cannot be "considered to be proper acceleration" or not. It can be expressed as one or the other.

Proper acceleration is acceleration expressed within a specific frame of reference (or, rather, a set of FoR's). It just so happens that it's more useful and intuitive to express it as three-acceleration for the purpose of this explanation. You could express the same problem in terms of four-acceleration and get an equivalent result, just one that's harder to digest. Same goes for proper acceleration, if you really want to make your life difficult (and, given your proficiency, I'd seriously advise against artificially making your analysis more difficult).

This is just a restatement of your confusion as to which FoR the Earth "is in".
Title: Re: The Math for universal Acceleration IS INCORRECT
Post by: Groit on April 11, 2020, 01:13:27 PM
That’s true, but from whatever inertial frame of reference you perceive the earth, its relative acceleration will always be 9.81 m/s2
This directly contradicts your prior statement, in which there is one inertial FoR in which that applies for a given moment. This is the usual problem with your claims - you keep disagreeing with yourself.

Also, "relative acceleration"? Relative to what?

Relative to an observer in free fall.
In FET, if i was to jump from a high rise building, the moment i step off the building my acceleration would be zero. And from my inertial FoR, the Earth will be accelerating at -9.81 m/s^2 (towards me) so the Earth is accelerating relative to all inertial frames.
Title: Re: The Math for universal Acceleration IS INCORRECT
Post by: Pete Svarrior on April 11, 2020, 10:15:01 PM
Relative to an observer in free fall.
How long has the observer been in free-fall for? This may seem trivial, but is the very core of PP's objection.

so the Earth is accelerating relative to all inertial frames.
This is entirely incorrect, for reasons even PP accepted. Let's not waste time repeating ourselves - you can catch up with the thread you're contributing to in your own time.
Title: Re: The Math for universal Acceleration IS INCORRECT
Post by: pricelesspearl on April 12, 2020, 04:33:24 AM
Quote
This directly contradicts your prior statement, in which there is one inertial FoR in which that applies for a given moment. This is the usual problem with your claims - you keep disagreeing with yourself.

No, it doesn’t contradict.  What I said that for any given moment, for any infinitesimal duration, the “relative” acceleration would be 9.81 m/s2.   That means it is always 9.81 m/s2.  Pick whatever moment you want in any inertial frame you want…its always going to be 9.81 m/s2.   One inertial frame may apply for that moment, but in the next moment, in the next inertial frame and in every moment and ever inertial frame after that it will 9.81 m/s2.

Quote
You are once again (pretending to be) hopelessly confused. The Earth's acceleration cannot be "considered to be proper acceleration" or not. It can be expressed as one or the other.

It isn’t an either/or proposition.  Proper acceleration can be expressed as relative…but doing that doesn’t change proper acceleration into relative acceleration or mean that proper acceleration no longer applies.

Proper acceleration by definition cannot be relative.  It is “the physical acceleration experienced by an object”.  Relative means “considered in relation or in proportion to something else”.  What is being physically experienced isn’t considered in relation to something else, whether its acceleration or pain. You don’t have to ask anybody else if you have a toothache. It doesn't matter if someone else doesn't believe your tooth is hurting. You know “objectively” if your tooth hurts or not.  There is nothing subjective about it.

If you are accelerating in a car at 100mph, you will experience physical effects from that.  You will feel it. Without ever looking outside,  you can “objectively” know you are accelerating. It doesn’t matter if your “relative” acceleration is 5mph to the guy on the sidewalk.  If you hit a brick wall you will still die. His perception of how fast you were going doesn’t mean squat.  Do you deny that is true? Do you not understand that what is physically perceived from one frame doesn't effect what is physically occurring in another?

You can’t have it both ways.  Either the earth is constantly, physically accelerating at 9.81 m/s2 or not.  For the sake of argument, imagine someone else in an inertial frame in a galaxy far far away perceives earth’s “relative” acceleration as 7.2 m/s2.  Does that mean “gravity” is weaker on earth? What if someone else in another inertial frame perceives it at 12 m/s2? Is" gravity" stronger?

If there is an infinite number of inertial frames and the earth’s proper acceleration can be perceived at an infinite number of different rates…how can FET say it is accelerating at any specific one if it can't be objectively determined?  For that matter, how can FET even say that it is accelerating at all, if you can’t determine it objectively?  Has any flat earther ever observed the earth from an external inertial frame?
Title: Re: The Math for universal Acceleration IS INCORRECT
Post by: Pete Svarrior on April 12, 2020, 11:36:05 AM
No, it doesn’t contradict.  What I said that for any given moment, for any infinitesimal duration, the “relative” acceleration would be 9.81 m/s2.
Oh, so you just disagree with the definition of proper acceleration. Amazing. Perhaps stop using that term and define your own, then?

Proper acceleration by definition cannot be relative.
This continues to be factually incorrect, and you were provided with an explanation as to why. You're gonna have to address it, instead of simply restating the same error over and over.

For the sake of argument, imagine someone else in an inertial frame in a galaxy far far away perceives earth’s “relative” acceleration as 7.2 m/s2.  Does that mean “gravity” is weaker on earth? What if someone else in another inertial frame perceives it at 12 m/s2? Is" gravity" stronger?
Keeping in mind what time dilation is (and that you failed to account for it) - yes, that is more or less correct. If you somehow managed to observe the Earth from one of the FoR's you described, your observation would be drastically different from that of a local observer on the Earth.

If you are accelerating in a car at 100mph, you will experience physical effects from that.  You will feel it.
Applying classical mechanics to a discussion on special relativity is a schoolboy error. Don't waste our time with that.

You can’t have it both ways.  Either the earth is constantly, physically accelerating at 9.81 m/s2 or not.
Welcome to the amazing world of relativity, where the things you consider obvious and intuitive are completely wrong. Enjoy your stay, and do some reading before you humiliate yourself again.

If there is an infinite number of inertial frames and the earth’s proper acceleration can be perceived at an infinite number of different rates…how can FET say it is accelerating at any specific one if it can't be objectively determined?  For that matter, how can FET even say that it is accelerating at all, if you can’t determine it objectively?  Has any flat earther ever observed the earth from an external inertial frame?
The very core of relativity is that there is no objective frame of reference. It's not that FET can't determine these things objectively - it's that your idea of being "objective" doesn't exist in physics. The core point here is that a frame of reference in which the Earth exceeds the speed of light cannot be defined. We focus on two reasonably observable FoR's - a local observer, and an arbitrary external inertial observer. Those are the "important" ones to explain, and we've covered them both.

We can say that the Earth is accelerating relative to a local observer in free-fall (as we do), because we're local observers, and we can easily verify this. This remains true in FET and RET alike. You can jump off a chair (please do so more competently than most other things you do - be careful not to hurt yourself), and, taking your body as the frame of reference, you will observe the Earth accelerating towards you to meet you.
Title: Re: The Math for universal Acceleration IS INCORRECT
Post by: Groit on April 12, 2020, 02:36:10 PM
We can say that the Earth is accelerating relative to a local observer in free-fall (as we do), because we're local observers, and we can easily verify this. This remains true in FET and RET alike. You can jump off a chair (please do so more competently than most other things you do - be careful not to hurt yourself), and, taking your body as the frame of reference, you will observe the Earth accelerating towards you to meet you.

Pete, hypothetically, what if we have two local observers, one is standing on a chair with a height of 1 m and the other is standing on top of a skyscraper that stands 1000 km above sea level. According to FET both observers will be accelerating with the Earth (since they're both attached to the surface and in the same FoR). So, if both observers jumped at the same time they should both see the Earth accelerating towards them at 9.81 m/s^2. However, we know that this is not the case, the observer on the skyscraper would experience an acceleration of 7.3 m/s^2 and the one on the chair 9.81 m/s^2. How does UA account for this change in acceleration? 
Title: Re: The Math for universal Acceleration IS INCORRECT
Post by: Pete Svarrior on April 12, 2020, 02:58:47 PM
How does UA account for this change in acceleration?
It doesn't. UA is not the only component of gravity.

Please familiarise yourself with the basics, and please avoid derailing threads with unrelated questions.
Title: Re: The Math for universal Acceleration IS INCORRECT
Post by: pricelesspearl on April 14, 2020, 12:30:20 AM
Quote
The very core of relativity is that there is no objective frame of reference. It's not that FET can't determine these things objectively - it's that your idea of being "objective" doesn't exist in physics.

If you have a problem with the idea of being able to determine your state of motion objectively, you can take it up with Edwin Taylor and John Archibald Wheeler .  They are both on Wikipedia if you doubt credentials. 


Page 31 Spacetime Physics @ http://www.eftaylor.com/spacetimephysics/

Quote
A reference frame is said to an “inertial’ or “free-float” or “Lorentz” reference frame in a certain region of space and time when throughout that region of spacetime-and within some specified accuracy-every free test particle initially at rest with respect to that  frame remains at rest, and every free test particle initially in motion with respect to that frame continues it motion  without change in speed or direction.

Wonder of wonders! This test can be carried out entirely within the free-float frame.  The observer need not look out of the room or refer to any measurements made external to the room. A free-float- frame is “local” in the sense that it is limited in space and time-and also “local in the sense that its free-float character can be determined from within, locally.

Does the FET earth meet the definition in the first paragraph?


Quote
The core point here is that a frame of reference in which the Earth exceeds the speed of light cannot be defined

The core point is why it can't be defined. 


Title: Re: The Math for universal Acceleration IS INCORRECT
Post by: Pete Svarrior on April 14, 2020, 10:58:09 AM
If you have a problem with the idea of being able to determine your state of motion objectively, you can take it up with Edwin Taylor and John Archibald Wheeler .   
I can't help you with this. Taking your misconceptions and trying to shield yourself with big names who didn't say what you think they said is a common strategy or yours, but it just doesn't work.

Until you've understood the very basics of the subject you're trying to argue about, this thread is doomed to go the way of the magical spirit level.

Does the FET earth meet the definition in the first paragraph?
The Earth is not a frame of reference. It is a body.
Title: Re: The Math for universal Acceleration IS INCORRECT
Post by: pricelesspearl on April 14, 2020, 11:27:07 PM
Quote
I can't help you with this. Taking your misconceptions and trying to shield yourself with big names who didn't say what you think they said is a common strategy or yours, but it just doesn't work.

So are you suggesting that the quote does not say that you can determine your state of motion objectively? If so, what does it mean? What you call "shielding" myself, most others call providing support for your position.  Your objection is disingenuous considering your claim that my position does not "exist in physics".  When I provide support that it does...just responding "that's not what it means" is not very persuasive or intellectually honest.

Quote
The Earth is not a frame of reference. It is a body.


Fair enough...let me rephrase...does earth exist in a certain region of space and time when throughout that region of spacetime-and within some specified accuracy-every free test particle initially at rest with respect to that  frame remains at rest, and every free test particle initially in motion with respect to that frame continues it motion  without change in speed or direction?
Title: Re: The Math for universal Acceleration IS INCORRECT
Post by: Clyde Frog on April 15, 2020, 12:56:46 AM
You are currently traveling at 0 m/s relative to the couch you are sitting upon. And simultaneously, you are traveling at .996c relative to a really energetic cosmic ray flying in your direction. Both of those frames are equally valid. If you were on a rocket, hurtling directly toward that cosmic ray at, let's say for fun, .5c relative to your couch that you were on moments ago. What do you think your perception is regarding how fast that cosmic ray is now flying at you? And what do you think the cosmic ray, if it had eyes, would perceive your velocity to be? If your answer to both is less than c, I really don't know why you are still posting in this thread. And if your answer is greater than c, then I think you'll understand before you even reply that you need to read some things.
Title: Re: The Math for universal Acceleration IS INCORRECT
Post by: Pete Svarrior on April 15, 2020, 09:29:50 AM
So are you suggesting that the quote does not say that you can determine your state of motion objectively?
What I am suggesting is that you should learn to walk before you can run. Special relativity is complicated and unintuitive, and I'm doing my best to select sources you should be able to easily digest. You're going out of your way to find sources which disagree, but you don't understand the context or implications of what's being said in them, and I'm not a good enough educator to wade through it all for you (nor do I care for your strategy of quote-mining papers as a substitute for argumentation). I don't care if you find me persuasive or not - I won't be able to help you so long as you're unwilling to be helped.

Fair enough...let me rephrase...does earth exist in a certain region of space and time when throughout that region of spacetime-and within some specified accuracy-every free test particle initially at rest with respect to that  frame remains at rest, and every free test particle initially in motion with respect to that frame continues it motion  without change in speed or direction?
I answered that question a long time ago:

At any point in time, you can identify an inertial FoR with regard to which the acceleration will be 9.81ms^-2 for an infinitesimal length of time. That is to say, the Earth would not be immediately moving relative to that frame. A moment later, this would no longer hold.

Try reading TheRealDave's post. I think it might help you.
Title: Re: The Math for universal Acceleration IS INCORRECT
Post by: Groit on April 15, 2020, 12:07:45 PM
You are currently traveling at 0 m/s relative to the couch you are sitting upon. And simultaneously, you are traveling at .996c relative to a really energetic cosmic ray flying in your direction. Both of those frames are equally valid. If you were on a rocket, hurtling directly toward that cosmic ray at, let's say for fun, .5c relative to your couch that you were on moments ago. What do you think your perception is regarding how fast that cosmic ray is now flying at you? And what do you think the cosmic ray, if it had eyes, would perceive your velocity to be? If your answer to both is less than c, I really don't know why you are still posting in this thread. And if your answer is greater than c, then I think you'll understand before you even reply that you need to read some things.

Yes, the velocities will never exceed c.
According to UA the Earth doesn't have constant velocity, it has acceleration, and when a cosmic rays approach Earth with constant velocity it takes a certain amount of time to get from the upper atmoshpere (atmolayer) to the ground. During this time the Earth accelerates at 9.81 m/s^2 with respect to the cosmic ray. The Earth's change in velocity during this time is very small but it's still there, do you agree?   
Title: Re: The Math for universal Acceleration IS INCORRECT
Post by: Clyde Frog on April 15, 2020, 02:28:53 PM
During this time the Earth accelerates at 9.81 m/s^2 with respect to the cosmic ray.
It most certainly does not. Relativistic effects would make for pretty pronounced differences in the perception of the Earth's acceleration when observed from the cosmic ray's side of things versus the observation someone would make on the surface of the Earth. The cosmic ray would "see" the Earth accelerating at a much smaller rate. Smaller with each passing moment. Yet the observer on the Earth would observe no change in the Earth's acceleration at all.
Title: Re: The Math for universal Acceleration IS INCORRECT
Post by: Groit on April 15, 2020, 03:09:48 PM
During this time the Earth accelerates at 9.81 m/s^2 with respect to the cosmic ray.
It most certainly does not. Relativistic effects would make for pretty pronounced differences in the perception of the Earth's acceleration when observed from the cosmic ray's side of things versus the observation someone would make on the surface of the Earth. The cosmic ray would "see" the Earth accelerating at a much smaller rate. Smaller with each passing moment. Yet the observer on the Earth would observe no change in the Earth's acceleration at all.

Ok, so observers on the surface of Earth are in an accelerating frame (non-inertial)?
Title: Re: The Math for universal Acceleration IS INCORRECT
Post by: JSS on April 15, 2020, 03:21:23 PM
Can someone fill me in about what's being argued here now? I've been reading this since it started and I'm kinda lost now. :)

The OP was saying that UA can't be true because it would push the Earth past the speed of light which is clearly wrong. UA is fine in those regards, 1G acceleration forever doesn't break laws. I think everyone can agree the OP's argument was wrong.

Now it seems too be focused on arguing if Earth is an non-inertial reference frame, which to me seems pretty clear that it is. Doesn't matter if it's Earths gravity accelerating us down or some UA force accelerating us up, either way a person on the surface is undergoing acceleration.

What exactly is being argued here at this point? I could use some help.
Title: Re: The Math for universal Acceleration IS INCORRECT
Post by: Groit on April 15, 2020, 03:45:54 PM
Now it seems too be focused on arguing if Earth is an non-inertial reference frame, which to me seems pretty clear that it is. Doesn't matter if it's Earths gravity accelerating us down or some UA force accelerating us up, either way a person on the surface is undergoing acceleration.

What exactly is being argued here at this point? I could use some help.

But those accelerations are not the same. UA is due to a force from beneath the Earth, and gravity is due to the curvature of spacetime and is not really a force, when you are accelerating towards the surface of the Earth you don't feel any forces acting on you.

Anyway my argument was towards the way we observe muons passing through the atmosphere.
JSS, maybe you can help, if the Earth was accelerating towards the muons would we measure them any differently to how we observe them in RET?
Title: Re: The Math for universal Acceleration IS INCORRECT
Post by: JSS on April 15, 2020, 04:20:39 PM
JSS, maybe you can help, if the Earth was accelerating towards the muons would we measure them any differently to how we observe them in RET?

I don't know enough about relativistic math to quote any supporting equations. But that muon would accelerate faster as it neared the Earth either due to our planets gravity, or due to it accelerating forward due to UA.

My suspicion is if there were any difference, it would be incredibly hard to measure. Possibly beyond the accuracy of anything we can currently rig up.

Lets take the rising elevator example as a starting point.  If you are in a metal box, you can not tell the difference between being on the surface of a planet at 1g, or being accelerated in space by a rocket at 1g. There is just no way in relativity theory to tell if you are accelerating or being pulled by gravity, they are literally the same thing.

So inside that box you just can't tell.

Now lets say the top of the box is open, and you have a muon detector with you.

Will muons behave differently?  I don't think so, but really, none of us have a PhD in Relativistic Physics in this so it's all kind of guesswork. I haven't drawn space-time diagrams to trace light cones through different frames of reference since college so I'm a little rusty.

Title: Re: The Math for universal Acceleration IS INCORRECT
Post by: Groit on April 15, 2020, 05:49:24 PM

I don't know enough about relativistic math to quote any supporting equations. But that muon would accelerate faster as it neared the Earth either due to our planets gravity, or due to it accelerating forward due to UA.

My suspicion is if there were any difference, it would be incredibly hard to measure. Possibly beyond the accuracy of anything we can currently rig up.

Lets take the rising elevator example as a starting point.  If you are in a metal box, you can not tell the difference between being on the surface of a planet at 1g, or being accelerated in space by a rocket at 1g. There is just no way in relativity theory to tell if you are accelerating or being pulled by gravity, they are literally the same thing.

So inside that box you just can't tell.

Now lets say the top of the box is open, and you have a muon detector with you.

Will muons behave differently?  I don't think so, but really, none of us have a PhD in Relativistic Physics in this so it's all kind of guesswork. I haven't drawn space-time diagrams to trace light cones through different frames of reference since college so I'm a little rusty.

Thanks for the explanation.
I would say that UA is probably the best evidence for FE'rs to back up their theory, as its very hard to disprove. Even Einstein said " It's as though the Earth is accelerating upwards".

Let's try another approach:
During some of my studies we observed natural radio waves emitted from gas clouds through the plane of our Galaxy. In doing this we could determine the distance and speeds of the spiral arms of the galaxy by measuring the change in wavelength of the radio waves. To get accurate measurements, we used some sophisticated software to account for the Earth's motion within the Galaxy, these being; Earth's axial spin, the orbit around the sun and the suns orbit around the Galaxy.
There were no formulas used to account for the Earth accelerating at 9.81 m/s^2.

Also, I do believe that when cosmologist are measuring the subtle differences in the Cosmic Microwave Backround Radiation (CMBR) to map the early universe, then they too need to account for the same motions and also the motion of the Galaxy around its centre of mass. They do not account for any acceleration due to UA.

If we were in fact accelerating as UA suggest, then we could actually measure the effects it would have on the CMBR. In the direction of acceleration the CMBR's would be greatly blueshifted and in the opposite direction it would be significantly redshifted, This is not what's being observed by the cosmologists. I think the CMBR can be used as reference frame in many aspects of physics/cosmology especially when travelling at relativistic speeds through acceleration.   
Title: Re: The Math for universal Acceleration IS INCORRECT
Post by: JSS on April 15, 2020, 06:10:20 PM
Thanks for the explanation.
I would say that UA is probably the best evidence for FE'rs to back up their theory, as its very hard to disprove. Even Einstein said " It's as though the Earth is accelerating upwards".

Let's try another approach:
During some of my studies we observed natural radio waves emitted from gas clouds through the plane of our Galaxy. In doing this we could determine the distance and speeds of the spiral arms of the galaxy by measuring the change in wavelength of the radio waves. To get accurate measurements, we used some sophisticated software to account for the Earth's motion within the Galaxy, these being; Earth's axial spin, the orbit around the sun and the suns orbit around the Galaxy.
There were no formulas used to account for the Earth accelerating at 9.81 m/s^2.

Also, I do believe that when cosmologist are measuring the subtle differences in the Cosmic Microwave Backround Radiation (CMBR) to map the early universe, then they too need to account for the same motions and also the motion of the Galaxy around its centre of mass. They do not account for any acceleration due to UA.

If we were in fact accelerating as UA suggest, then we could actually measure the effects it would have on the CMBR. In the direction of acceleration the CMBR's would be greatly blueshifted and in the opposite direction it would be significantly redshifted, This is not what's being observed by the cosmologists. I think the CMBR can be used as reference frame in many aspects of physics/cosmology especially when travelling at relativistic speeds through acceleration.

Yes that's exactly right, we would absolutely see everything blue shifted in the sky, and red shifted if we drilled a hole through the Earth's disk and looked through it.

It's actually way worse than not detecting any blueshift.  As stated in another thread, if we have been accelerating at 1g for a mere 6000 years, the outside universe would experience 8.843e+1345 years of time. That's an insanely long amount of time. There wouldn't BE a universe outside at this point, which would be lucky because if there was, even a single PHOTON hitting the earth would vaporize it. I can't even calculate how much energy incoming light would have going that close to light speed. So the universe would have to be 100% empty. Everything we see would have to be, well, something else. It would mean that 100% of everything we know about the universe is wrong. All of it. That's a big stretch.

The basic issue here is that to make UA work it has to exist in a very different universe than our own. Planets, the stars, the sun, cosmic background radiation, gravity... none of it works. Literally everything not attached to the Earth has to have some alternate explanation.  Maybe some illusion or projection so the likely answer to your question is "CBR isn't real" or "CBR is just static leaking from the holo-emitters that create the sky."

UA breaks everything.
Title: Re: The Math for universal Acceleration IS INCORRECT
Post by: MathJunkie1 on April 15, 2020, 07:42:44 PM
I assume Iactuallycanthink believes the earth is round, which is great! I think so too. If you use the conventional model of our Earth orbiting a celestial star, the Sun, it is accelerating and maintaining a constant speed. For the sake of argument, though, has anyone ever suggested that a flat earth could simply do the same? If it were facing inward, centrifugal force could plausibly create a gravitational force similar to our planet’s own.
Please get back with me if I sound crazy. This is the first time I put that thought out and I’m NOT a flat earther I swear!
Title: Re: The Math for universal Acceleration IS INCORRECT
Post by: JSS on April 15, 2020, 08:18:10 PM
I assume Iactuallycanthink believes the earth is round, which is great! I think so too. If you use the conventional model of our Earth orbiting a celestial star, the Sun, it is accelerating and maintaining a constant speed. For the sake of argument, though, has anyone ever suggested that a flat earth could simply do the same? If it were facing inward, centrifugal force could plausibly create a gravitational force similar to our planet’s own.
Please get back with me if I sound crazy. This is the first time I put that thought out and I’m NOT a flat earther I swear!

That's actually a great idea. There is nothing at all wrong with using your imagination, now you've given people something interesting to think about.

If the Earth were a disk, attached to the sun with a a rope and spun around it fast enough we would indeed experience centrifugal force, and it would be very similar to gravity.

The difference would be the Coriolis force for anything thrown or launched into the air, making it curve.  On something as big as a planet this effect would be small but absolutely measurable and we would have seen it. Artillery shells for instance would need to be corrected for it. Long distance battleship guns in fact have to take into account the curve and rotation of the Earth, so on a flat Earth being spun we would need different corrections.

Math time!  I used to design make believe rotating space stations in my spare time when I was younger, so I remember the formula for centrifugal force. (With help from Wikipedia)

g = v^2 / r

So g is the gravitational acceleration felt in meters per second, v is the radial velocity in meters per second, r is the radius in meters.

We can rearrange that to this to solve for the radial velocity.

v = (g * r)^0.5

We want 1g for the liner acceleration so that is 9.8m/s

Lets set the radius at 150,000,000,000m which is the distance to the Sun.

So we get...

( 150,000,000,000 * 9.8 ) ^ 0.5 = 1,212,435m/s

So that's pretty fast.  The speed of light is 299,792,458m/s so that's nearly 0.5 percent of the speed of light.  Whoo.

That speed would make one year about 9 days long. 

If you fell off the edge, you would go flying off into space away from the sun at 0.5c for free.  This would make launching deep space probes super easy, just push it off the edge. Getting them back would be a problem.

Launching anything sunward would be interesting.  If you weren't careful it would slam back into the disk as it was being slung around.

This would make an awesome setting for a sci-fi story, actually.  Finding some crazy flat planet tethered to a star that aliens built.  I'd read that!

( Edit: Well duh, that's basically Niven's Ringworld stories but a disk instead of a ring. Same distance and speed, hah. The sci-fi comment made me remember it. )
Title: Re: The Math for universal Acceleration IS INCORRECT
Post by: Groit on April 18, 2020, 03:48:48 PM
During this time the Earth accelerates at 9.81 m/s^2 with respect to the cosmic ray.
It most certainly does not. Relativistic effects would make for pretty pronounced differences in the perception of the Earth's acceleration when observed from the cosmic ray's side of things versus the observation someone would make on the surface of the Earth. The cosmic ray would "see" the Earth accelerating at a much smaller rate. Smaller with each passing moment. Yet the observer on the Earth would observe no change in the Earth's acceleration at all.

So from the muons FoR the Earth's acceleration would be very small and we can say its negligible. Then the Earth and the muons are travelling towards each other at close to speed of light, the muons at 0.98c and lets say the Earth is also travelling at 0.98c (although it would probably be higher since its been accelerating for thousands of years).

So then using the velocity transformation to find the relative velocity:
v%26%23039%3B_%7Bx%7D%3D%5Cfrac%7Bv_%7Bx%7D-V%7D%7B1-%5Cfrac%7BVv_%7Bx%7D%7D%7Bc%5E%7B2%7D%7D%7D


v%26%23039%3B_%7Bx%7D is the relative velocity
v_%7Bx%7D is the muons velocity
V is the Earth's velocity
c%3D1 using 1 for the speed of light

we have:

v%26%23039%3B_%7Bx%7D%3D%5Cfrac%7B-0.98c-0.98c%7D%7B1%2B%5Cfrac%7B0.98c%5Ctimes%200.98c%7D%7B1%5E%7B2%7D%7D%7D

%3D-0.9998c

Now, the rest halflife time for the muons to decay is 2.2%5Ctimes%2010%5E%7B-6%7Ds

Using the time dilation equation:
T%3D%5Cfrac%7BT_%7B0%7D%7D%7B%5Csqrt%7B1-%5Cfrac%7Bv%26%23039%3B%5E%7B2%7D%7D%7Bc%5E%7B2%7D%7D%7D%7D

T is Earth's clock
T_%7B0%7D is the muons clock

So then the time measured on Earth would be:
T%3D%5Cfrac%7B2.2%5Ctimes%2010%5E%7B-6%7D%7D%7B%5Csqrt%7B1-%5Cfrac%7B0.9998%5E%7B2%7D%7D%7B1%5E%7B2%7D%7D%7D%7D

%3D1.1%5Ctimes%2010%5E%7B-4%7Ds

This is not the time being measured by the scientists. The actual time measured is %3D1.1%5Ctimes%2010%5E%7B-5%7Ds
And if we use the relative velocity between the Earth and the muons of 0.98c then this matches the observations made by the scientists:

T%3D%5Cfrac%7B2.2%5Ctimes%2010%5E%7B-6%7Ds%7D%7B%5Csqrt%7B1-%5Cfrac%7B0.98%5E%7B2%7D%7D%7B1%5E%7B2%7D%7D%7D%7D

%3D1.1%5Ctimes%2010%5E%7B-5%7Ds

If the Earth was hurtling towards the muons at near c then the muons would take 10 times longer to decay.





Title: Re: The Math for universal Acceleration IS INCORRECT
Post by: Clyde Frog on April 18, 2020, 06:20:44 PM
Now you want to talk about muons? I guess have fun continuing to shift the conversation.
Title: Re: The Math for universal Acceleration IS INCORRECT
Post by: Groit on April 18, 2020, 08:36:44 PM
Now you want to talk about muons? I guess have fun continuing to shift the conversation.

It's slightly off topic to the OP, but I've been talking about the 'muon measurements' since page 2 of the thread, but we couldn't seem to decide on a reference frame for the Earth.
Pete Svarrior said "take note of TheRealDave's posts" as you seem to know a bit about UA, and your last post suggested that the Earth is travelling close to the speed of light and the acceleration would be very small from the cosmic rays rest frame. Since muons are created by cosmic rays, i took your advice and used some SR to calculate the effects of time dilation when the Earth and the muons are travelling towards each other.
Title: Re: The Math for universal Acceleration IS INCORRECT
Post by: xasop on April 30, 2020, 10:09:26 AM
If the Earth was hurtling towards the muons at near c then the muons would take 10 times longer to decay.

Your reasoning here is completely backwards.

You start with the calculated speed of these muons based on observations combined with the RE model. You then plug this RE-derived speed into the FE model (using a frame of reference corresponding to the Earth's velocity hundreds of years ago, for some reason) and conclude that the prediction of their decay time doesn't match observations. No shit?

That's not how science works. The only variable in this entire calculation that has been directly measured is the muons' decay time, which tells us their speed relative to the Earth. Any and all calculation of other variables must be done using a consistent model.
Title: Re: The Math for universal Acceleration IS INCORRECT
Post by: BRrollin on April 30, 2020, 02:53:47 PM
If the Earth was hurtling towards the muons at near c then the muons would take 10 times longer to decay.

Your reasoning here is completely backwards.

You start with the calculated speed of these muons based on observations combined with the RE model. You then plug this RE-derived speed into the FE model (using a frame of reference corresponding to the Earth's velocity hundreds of years ago, for some reason) and conclude that the prediction of their decay time doesn't match observations. No shit?

That's not how science works. The only variable in this entire calculation that has been directly measured is the muons' decay time, which tells us their speed relative to the Earth. Any and all calculation of other variables must be done using a consistent model.

That is not quite correct either, because the muon lifetime has been pinned down using other collision experiments. Hence, We also can identify the inertial frame to be the Earth.

If the Earth was accelerating upwards, we would indeed measure a longer lifetime for the muon, rather than a shorter one.

The muon decay is experimental evidence that stands against UA.
Title: Re: The Math for universal Acceleration IS INCORRECT
Post by: JSS on April 30, 2020, 10:18:46 PM
So from the muons FoR the Earth's acceleration would be very small and we can say its negligible. Then the Earth and the muons are travelling towards each other at close to speed of light, the muons at 0.98c and lets say the Earth is also travelling at 0.98c (although it would probably be higher since its been accelerating for thousands of years).

Higher is an understatement. :)

I've tried to calculate what percentage of the speed of light a Flat Earth would be going after 6000 years of 1G acceleration and have yet to find a way of calculating it with the precision needed to get anything other than 100%.  It's going to be 99.99999% something, but with a LOT of nines.  I'm still trying to get an answer for my own curiosity.

I think I'll try again tonight to enter the equations into an infinite precision calculator of some sort and see if I can get a number.  It's a fun challenge.

It would be fast enough that a single muon would likely be a literal earth shattering event.
Title: Re: The Math for universal Acceleration IS INCORRECT
Post by: Pete Svarrior on April 30, 2020, 10:24:41 PM
Most of these objections continue to assume that the Earth "has been accelerating for thousands of years", relative to some mystical univeral frame of reference.

So, for the guys at the back: there is no such thing as a universal frame of reference. Similarly, saying that the Earth is moving at a certain percentage of c without defining the FoR is not just wrong, it's meaningless.
Title: Re: The Math for universal Acceleration IS INCORRECT
Post by: Tom Bishop on April 30, 2020, 10:51:30 PM
Not all physicists agree (http://dispatchesfromturtleisland.blogspot.com/2018/02/muon-g-2-anomaly-fully-explained-with.html) that the muon anomaly represents new physics or a contradiction of the equivalence principle.
Title: Re: The Math for universal Acceleration IS INCORRECT
Post by: iamcpc on April 30, 2020, 11:26:50 PM
Most of these objections continue to assume that the Earth "has been accelerating for thousands of years", relative to some mystical univeral frame of reference.

So, for the guys at the back: there is no such thing as a universal frame of reference. Similarly, saying that the Earth is moving at a certain percentage of c without defining the FoR is not just wrong, it's meaningless.

Pete the issue here is that there has been some force pulling things down toward the earth for as long as we have history of. There are hieroglyphs from ancient Egypt depicting moving stone blocks along the ground because of some force which is pulling those blocks twoard the earth. That was literally thousands of years ago.

In UA this force is caused by the acceleration of the earth. Because we know this force existed for thousands of years, and the force is caused by acceleration, then the acceleration must have been happening for thousands of years.
Title: Re: The Math for universal Acceleration IS INCORRECT
Post by: BRrollin on April 30, 2020, 11:45:43 PM
Not all physicists agree (http://dispatchesfromturtleisland.blogspot.com/2018/02/muon-g-2-anomaly-fully-explained-with.html) that the muon anomaly represents new physics or a contradiction of the equivalence principle.

The g-2 anomalous magnetic moment of the muon is an entirely different subject, and has to do with quantum electrodynamics. It has nothing to do with relativistic time dilation, and instead involves the deviation from a point like structure as found through higher order loop corrections of Feynman diagrams.

The only connection between this topic and the current discussion is the word “muon.”

Perhaps you wish to start a new thread? I would be happy to talk more about it there, including detailing the mathematics of the renormalization process of the spacetime integrals. But it is not on-topic here.

Nevertheless, it is always important, if you wish to cite disagreement among physicists, to link published articles in peer reviewed journals. Because that is where the dialogues of disagreement takes place. Doing so also ensures one is not inadvertently linking to pseudoscientific tangents that seek to derail legitimate scientific discourse.

Scientists disagree on all sorts of things. This is one power of science :)
Title: Re: The Math for universal Acceleration IS INCORRECT
Post by: JSS on April 30, 2020, 11:53:17 PM
Most of these objections continue to assume that the Earth "has been accelerating for thousands of years", relative to some mystical univeral frame of reference.

So, for the guys at the back: there is no such thing as a universal frame of reference. Similarly, saying that the Earth is moving at a certain percentage of c without defining the FoR is not just wrong, it's meaningless.

I guess I'm confused. If the Earth is accelerating at 1G now, and is still accelerating at 1G in ten minutes, that's 10 minutes of acceleration.

So if you go back 1000 years, that's 1000 years of acceleration at 1G.
Title: Re: The Math for universal Acceleration IS INCORRECT
Post by: Tom Bishop on May 01, 2020, 12:23:15 AM
Not all physicists agree (http://dispatchesfromturtleisland.blogspot.com/2018/02/muon-g-2-anomaly-fully-explained-with.html) that the muon anomaly represents new physics or a contradiction of the equivalence principle.

The g-2 anomalous magnetic moment of the muon is an entirely different subject, and has to do with quantum electrodynamics. It has nothing to do with relativistic time dilation, and instead involves the deviation from a point like structure as found through higher order loop corrections of Feynman diagrams.

The only connection between this topic and the current discussion is the word “muon.”


It is talking about how Einstein's Equivalence Principle may have or have not been properly implemented: "It is my melancholy duty to report that these articles are fundamentally flawed in that they fail to correctly implement the Einstein equivalence principle of general relativity."

The article links to a blog by physicist Luboš Motl which also talks about the equivalence principle for this:

https://motls.blogspot.com/2018/02/experiments-may-only-measure-gauge.html

Quote
For example, the equivalence principle says that if you perform an experiment inside a small enough and freely falling lab which has no windows, the results don't allow you to figure out whether you're in a gravitational field or not. If the ratio of the electron's and muon's magnetic moments depended on your being near Earth, you could say whether you're near the Earth inside that lab, and the equivalence principle would be violated. That's it.

Again, this description from physicist Luboš Motl is describing about how this muon anomaly may violate the equivalence principle. You are incorrect to claim that this does not have anything to do with relativistic effects.

If not this muon-equivalence-principle anomaly, what muon-equivalence-principle anomaly are you referring to?

Quote from: BRrollin
Nevertheless, it is always important, if you wish to cite disagreement among physicists, to link published articles in peer reviewed journals. Because that is where the dialogues of disagreement takes place. Doing so also ensures one is not inadvertently linking to pseudoscientific tangents that seek to derail legitimate scientific discourse.

The blog links to the work of physcists, and their disagreements. It shows that it is controversial. I can't see that you have linked us to anything.

This physicist sure seems to think that the anomalous muon magnetic moment is related to a test of relativistic time dilation:

https://inspirehep.net/literature/133026

Quote
Final Report on the CERN Muon Storage Ring Including the Anomalous Magnetic Moment and the Electric Dipole Moment of the Muon, and a Direct Test of Relativistic Time Dilation

Abstract: A comprehensive description of the muon storage ring and its operation is given, and the final results of the experiment are presented and discussed. The anomalous magnetic moments of positive and negative muons are found to be a μ + = 1165911(11) × 10 −9 and a μ − = 1165937(12) × 10 −9 giving an average value for muons of a μ = 1165924(8.5) × 10 −9 . The electric dipole moments were also measured with the results D μ += (8.6 ± 4.5) × 10 −9 e · cm and D μ − = (0.8 ± 4.3) × 10 −19 e · cm. Under the assumption of the CPT theorem these yield a weighted average of D μ = (3.7 ± 3.4) × 10 −19 e · cm. Finally the time transformation of special relativity is shown to be valid to (0.8 ± 0.7) × 10 −3 at γ ≅ 29.3. All the errors quoted here are one standard deviation and contain both statistical and systematic effects.

So again, what are you talking about when you claim that the anomalous magnetic moment has nothing to do with relativistic time dilation or the equivalence principle?

Maybe you should link to and list out these anomalous muon time dilation experiments for us, so we can see what they actually are.
Title: Re: The Math for universal Acceleration IS INCORRECT
Post by: BRrollin on May 01, 2020, 01:25:42 AM
Not all physicists agree (http://dispatchesfromturtleisland.blogspot.com/2018/02/muon-g-2-anomaly-fully-explained-with.html) that the muon anomaly represents new physics or a contradiction of the equivalence principle.

The g-2 anomalous magnetic moment of the muon is an entirely different subject, and has to do with quantum electrodynamics. It has nothing to do with relativistic time dilation, and instead involves the deviation from a point like structure as found through higher order loop corrections of Feynman diagrams.

The only connection between this topic and the current discussion is the word “muon.”

Perhaps you wish to start a new thread? I would be happy to talk more about it there, including detailing the mathematics of the renormalization process of the spacetime integrals. But it is not on-topic here.

Nevertheless, it is always important, if you wish to cite disagreement among physicists, to link published articles in peer reviewed journals. Because that is where the dialogues of disagreement takes place. Doing so also ensures one is not inadvertently linking to pseudoscientific tangents that seek to derail legitimate scientific discourse.

Scientists disagree on all sorts of things. This is one power of science :)

It is talking about how Einstein's Equivalence Principle may have or have not been properly implemented: "It is my melancholy duty to report that these articles are fundamentally flawed in that they fail to correctly implement the Einstein equivalence principle of general relativity."

The article links to a blog by physicist Luboš Motl which also talks about the equivalence principle for this:

https://motls.blogspot.com/2018/02/experiments-may-only-measure-gauge.html

Quote
For example, the equivalence principle says that if you perform an experiment inside a small enough and freely falling lab which has no windows, the results don't allow you to figure out whether you're in a gravitational field or not. If the ratio of the electron's and muon's magnetic moments depended on your being near Earth, you could say whether you're near the Earth inside that lab, and the equivalence principle would be violated. That's it.

Again, this description from physicist Luboš Motl is describing about how this muon anomaly may violate the equivalence principle. You are incorrect to claim that this does not have anything to do with relativistic effects.

If not this muon-equivalence-principle anomaly, what muon-equivalence-principle anomaly are you referring to?

Quote from: BRrollin
Nevertheless, it is always important, if you wish to cite disagreement among physicists, to link published articles in peer reviewed journals. Because that is where the dialogues of disagreement takes place. Doing so also ensures one is not inadvertently linking to pseudoscientific tangents that seek to derail legitimate scientific discourse.

The blog links to the work of physcists, and their disagreements. I can't see that you have linked us to anything.

True, but the blogs are...blogs. Scientific discourse is not conducted in blogs. See here:

https://arxiv.org/abs/hep-ph/0010194

Or here:

https://res.mdpi.com/d_attachment/atoms/atoms-07-00028/article_deploy/atoms-07-00028-v2.pdf

Or one of the seminal papers here:

https://journals.aps.org/pr/abstract/10.1103/PhysRev.140.B397

Even entire books have been written:

https://link.springer.com/book/10.1007/978-3-540-72634-0

Which all support the known understanding of what g-2 is about.

You could also look in just about any graduate text on quantum field theory, or quantum electrodynamics...since this is what they teach. See a University course’s PowerPoint:

https://www.asc.ohio-state.edu/gan.1/teaching/winter10/Chapter3.pdf

It seems as though you may have confused two topics. The anomalous moment of the muon (g-2) is different than your source’s claimed “anomalous equivalence principle” issue. It would probably be a good start to find a published article on this claimed effect - which should exist if it is legitimate.

I’m not sure how to best help you. Instead of discussing some very interesting implications of particle behavior, we are getting stuck on the basic definitions.

For example, Quantum relativity and relativity are entirely different fields. One is the proper context to discuss time dilation, while the other is not.

I’ll try to help the best I can, but recognize that I won’t be able to teach you all of quantum mechanics and relativity in a forum.

But I do encourage you to learn it somehow. I think it will allow you to distinguish between valid scientific claims and opinions.
Title: Re: The Math for universal Acceleration IS INCORRECT
Post by: Tom Bishop on May 01, 2020, 01:30:15 AM
I did link you to a source which was using the anomalous muon magnetic moment as a test of relativistic time dilation:

https://inspirehep.net/literature/133026

Quote
Final Report on the CERN Muon Storage Ring Including the Anomalous Magnetic Moment and the Electric Dipole Moment of the Muon, and a Direct Test of Relativistic Time Dilation

Abstract: A comprehensive description of the muon storage ring and its operation is given, and the final results of the experiment are presented and discussed. The anomalous magnetic moments of positive and negative muons are found to be a μ + = 1165911(11) × 10 −9 and a μ − = 1165937(12) × 10 −9 giving an average value for muons of a μ = 1165924(8.5) × 10 −9 . The electric dipole moments were also measured with the results D μ += (8.6 ± 4.5) × 10 −9 e · cm and D μ − = (0.8 ± 4.3) × 10 −19 e · cm. Under the assumption of the CPT theorem these yield a weighted average of D μ = (3.7 ± 3.4) × 10 −19 e · cm. Finally the time transformation of special relativity is shown to be valid to (0.8 ± 0.7) × 10 −3 at γ ≅ 29.3. All the errors quoted here are one standard deviation and contain both statistical and systematic effects.

Yet you say:

Quote
anomalous magnetic moment of the muon is an entirely different subject, and has to do with quantum electrodynamics. It has nothing to do with relativistic time dilation

Quote
The only connection between this topic and the current discussion is the word “muon.”

This can be used as a test of time dilation and the equivalence principle.

Now show us how the time dilation muon anomalies are tested with whatever experiments you think that you are talking about. You have not linked us to anything on these time dilation anomalies, only your own opinion.

Quote
True, but the blogs are...blogs. Scientific discourse is not conducted in blogs.

Actually Luboš Motl is a respected theoretical physicist, who conducts his scientific discourse in a blog.
Title: Re: The Math for universal Acceleration IS INCORRECT
Post by: BRrollin on May 01, 2020, 03:00:10 AM
I did link you to a source which was using the anomalous muon magnetic moment as a test of relativistic time dilation:

https://inspirehep.net/literature/133026

Quote
Final Report on the CERN Muon Storage Ring Including the Anomalous Magnetic Moment and the Electric Dipole Moment of the Muon, and a Direct Test of Relativistic Time Dilation

Abstract: A comprehensive description of the muon storage ring and its operation is given, and the final results of the experiment are presented and discussed. The anomalous magnetic moments of positive and negative muons are found to be a μ + = 1165911(11) × 10 −9 and a μ − = 1165937(12) × 10 −9 giving an average value for muons of a μ = 1165924(8.5) × 10 −9 . The electric dipole moments were also measured with the results D μ += (8.6 ± 4.5) × 10 −9 e · cm and D μ − = (0.8 ± 4.3) × 10 −19 e · cm. Under the assumption of the CPT theorem these yield a weighted average of D μ = (3.7 ± 3.4) × 10 −19 e · cm. Finally the time transformation of special relativity is shown to be valid to (0.8 ± 0.7) × 10 −3 at γ ≅ 29.3. All the errors quoted here are one standard deviation and contain both statistical and systematic effects.

Yet you say:

Quote
anomalous magnetic moment of the muon is an entirely different subject, and has to do with quantum electrodynamics. It has nothing to do with relativistic time dilation

Quote
The only connection between this topic and the current discussion is the word “muon.”

This can be used as a test of time dilation and the equivalence principle.

Now show us how the time dilation muon anomalies are tested with whatever experiments you think that you are talking about. You have not linked us to anything on these time dilation anomalies, only your own opinion.

Quote
True, but the blogs are...blogs. Scientific discourse is not conducted in blogs.

Actually Luboš Motl is a respected theoretical physicist, who conducts his scientific discourse in a blog.

Ahh yes, I am familiar with that study. You see, what they intended to do is experimentally constrain the moment. The results they provide are consistent with quantum field theory calculations.

As a bonus, they also use the experiment to constrain relativistic time dilation. They do this by being very clever about their experiment, so that they can extract as much science as possible. This is common.

I can see why this confused you. Both the moment and time dilation are in the same study, so it makes sense that you would conclude that one depends on the other somehow.

Unfortunately, you chose probably the worst possible study to plead your case, because they confirm Lorentz dilation to a gamma factor of 28. You don’t want that, because it SUPPORTS time dilation of the current theory.

Also, I really don’t understand what you mean by:

“Now show us how the time dilation muon anomalies are tested with whatever experiments you think that you are talking about. You have not linked us to anything on these time dilation anomalies, only your own opinion.“

There are no time dilation muon anomalies that I have heard of. This is confirmed in the results of the very study you just linked. It’s what the gamma factor means...

YOU are claiming this world renown physicist thinks there is. Okay, then link one of his publications - in a peer reviewed journal. That would be fun to discuss.

No respected physicist does science in a blog. They publish their science. Folks who can’t get published push their unpublishable ideas in blogs....

Not saying this guy is one of those. Many physicists HAVE blogs....but they publish too.

So? Let’s read some publications from this famous theorist on anomalous muon time dilation equivalence principle thing.

But I gotta tell ya, just the phrase “anomalous muon time dilation equivalence principle” just sounds like cobbled together physics words, Deepak Chopra style.

But I look forward to being corrected :)
Title: Re: The Math for universal Acceleration IS INCORRECT
Post by: pricelesspearl on May 01, 2020, 05:05:35 AM
Quote
You are currently traveling at 0 m/s relative to the couch you are sitting upon. And simultaneously, you are traveling at .996c relative to a really energetic cosmic ray flying in your direction. Both of those frames are equally valid. If you were on a rocket, hurtling directly toward that cosmic ray at, let's say for fun, .5c relative to your couch that you were on moments ago. What do you think your perception is regarding how fast that cosmic ray is now flying at you? And what do you think the cosmic ray, if it had eyes, would perceive your velocity to be? If your answer to both is less than c, I really don't know why you are still posting in this thread. And if your answer is greater than c, then I think you'll understand before you even reply that you need to read some things.

The discussion wasn't about relative velocity...it was about "relative" vs. "proper" acceleration.  Those are two different things. if I am accelerating at a constant 9.8 m/s2 and I am holding an accelerometer...that's what it will read no matter how fast anything is hurtling towards me.
 
Title: Re: The Math for universal Acceleration IS INCORRECT
Post by: xasop on May 01, 2020, 06:13:05 PM
That is not quite correct either, because the muon lifetime has been pinned down using other collision experiments. Hence, We also can identify the inertial frame to be the Earth.

Have you published a paper on this? I'm sure modern science would be fascinated to know how you concluded that (the surface of) the Earth is an inertial frame of reference, given that it would discredit the past century of modern physics.
Title: Re: The Math for universal Acceleration IS INCORRECT
Post by: BRrollin on May 01, 2020, 06:38:46 PM
That is not quite correct either, because the muon lifetime has been pinned down using other collision experiments. Hence, We also can identify the inertial frame to be the Earth.

Have you published a paper on this? I'm sure modern science would be fascinated to know how you concluded that (the surface of) the Earth is an inertial frame of reference, given that it would discredit the past century of modern physics.

Are the goal posts being moved all of a sudden? Why this call for first author publications for ideas expressed on this forum? I don’t mind actually, as long as everyone is held to the same standard ;)

Or is this a straw man? Changing the debate topic to hide a weakness? I don’t think my publications are the topic here.

Tell ya what: I’ll give you the benefit of the doubt, and explain the physics.

What is an inertial frame? What absolute standard of rest do we reference? The center of the Milky Way? Whoops, that’s a noninertial frame with respect to the local group. The local group then? Nope. Cause it’s part of the Virgo cluster.

Eh? How do we proceed? Well, it behooves you to consider relevant time and distance scales. Inertial frames reside within this framework, but if you leave it and reference, for example, the CMB, then nothing is an inertial frame.

So how come I can do simple incline experiments on the Earth, under the assumption that we’re in an inertial frame, and have the results agree with theory?

Because the experiments occur on time and distance scales much different than the CMB (or other noninertial frames of reference).

Does this also work on the time and distance scales in the muon dilation experiments? Why yes, yes it does! Just about every physics undergraduate does this problem. It’s classic!

As with all science, there is quite a bit more to it than one can intuit from a few minutes of scrolling Wikipedia.
Title: Re: The Math for universal Acceleration IS INCORRECT
Post by: xasop on May 01, 2020, 09:37:14 PM
What is an inertial frame? What absolute standard of rest do we reference?

There is no "absolute standard of rest". This has been generally accepted in the context of Einsteinian relativity for over a century, and Newtonian relativity for far longer. Once again, I invite you to publish if you have evidence to the contrary.

To answer your question, an inertial frame of reference is one without a proper acceleration.
Title: Re: The Math for universal Acceleration IS INCORRECT
Post by: BRrollin on May 01, 2020, 10:14:00 PM
What is an inertial frame? What absolute standard of rest do we reference?

There is no "absolute standard of rest". This has been generally accepted in the context of Einsteinian relativity for over a century, and Newtonian relativity for far longer. Once again, I invite you to publish if you have evidence to the contrary.

To answer your question, an inertial frame of reference is one without a proper acceleration.

Yeah, that’s what my previous reply said...glad we agree.

Actually, not to be nit-picky, but the CMB itself is used as an absolute standard of rest in Cosmology. This is an approximation, of course, but is used by many.

Nice answer to the first question though! Textbook answer.

But none of this matters, as the majority of my previous post addressed the muon issue. That is the topic.

Also, how would I publish something that is common, undergraduate knowledge? I am confused by your request.
Title: Re: The Math for universal Acceleration IS INCORRECT
Post by: xasop on May 01, 2020, 10:43:19 PM
Yeah, that’s what my previous reply said...glad we agree.

No, your previous reply went off on a tangent about the CMB, as though that is some sort of absolute frame of reference. I ignored everything after the point at which you made that error, because any argument based on a flawed assertion is also flawed.
Title: Re: The Math for universal Acceleration IS INCORRECT
Post by: BRrollin on May 01, 2020, 11:04:39 PM
Yeah, that’s what my previous reply said...glad we agree.

No, your previous reply went off on a tangent about the CMB, as though that is some sort of absolute frame of reference. I ignored everything after the point at which you made that error, because any argument based on a flawed assertion is also flawed.

Your reply makes sense then: you didn’t read the previous entry!

Actually the reply AFTER that detailed the CMB as an approximate standard used in Cosmology. The reply before - which is what you are referencing, did not make that claim.

Well, lemme know if you’d like to reengage. The CMB description was to help you understand how inertial frames are used and defined. Since you don’t seem to believe this assertion, it sounds like the information would be useful to you.

It would probably help you understand how inertial frames are used in the muon problem.

At any rate, nice talking with ya. Toodles.
Title: Re: The Math for universal Acceleration IS INCORRECT
Post by: xasop on May 01, 2020, 11:44:05 PM
The reply before - which is what you are referencing, did not make that claim.

Not explicitly, no, but you did imply it:

Inertial frames reside within this framework, but if you leave it and reference, for example, the CMB, then nothing is an inertial frame.

The CMB has absolutely nothing to do with whether something is an inertial frame of reference or not. Indeed, since inertiality is not a relative property, no other frame of reference is relevant. I'm not even sure why you brought it up.
Title: Re: The Math for universal Acceleration IS INCORRECT
Post by: BRrollin on May 02, 2020, 12:36:46 AM
The reply before - which is what you are referencing, did not make that claim.

Not explicitly, no, but you did imply it:

Inertial frames reside within this framework, but if you leave it and reference, for example, the CMB, then nothing is an inertial frame.

The CMB has absolutely nothing to do with whether something is an inertial frame of reference or not. Indeed, since inertiality is not a relative property, no other frame of reference is relevant. I'm not even sure why you brought it up.

Well sure it does! You probably aren’t sure why I brought it up because you didn’t read my post ;). You then lack the proper reference frame when I expand upon it (see what I did there. Like 3 puns in that sentence. That one’s just for you, Perceval /edit: Parsifal. Sorry bout that).

Anyway, back to your point. I think you may have some confusion between inertial frames versus rest frames. I tried to help delineate this for you in my post (that you said you didn’t read). I’m not really interested in wasting my time if you’re just going to ignore content, and then express frustration at not knowing information that content addressed.

Lemme know if you’d like to start fresh. We can renormalize to the crux of the matter, which is the distance and time scales where taking the Earth to be an inertial frame is appropriate and when it is not.
Title: Re: The Math for universal Acceleration IS INCORRECT
Post by: xasop on May 02, 2020, 05:58:41 AM
Anyway, back to your point. I think you may have some confusion between inertial frames versus rest frames. I tried to help delineate this for you in my post (that you said you didn’t read).

I never said that. You're the one who, for some reason, keeps claiming I didn't read a post that I responded to, not me. Given that the post in question is utterly wrong, and contains such absurd phrases as "a noninertial frame with respect to", I don't have much regard for what you think I might be confused about.

Lemme know if you’d like to start fresh. We can renormalize to the crux of the matter, which is the distance and time scales where taking the Earth to be an inertial frame is appropriate and when it is not.

Sorry, what? You don't "take" a frame of reference to be inertial, it either is inertial or it isn't. Furthermore, the Earth is not a frame of reference, it is an object consisting of mostly rock and water. Objects are not frames of reference.
Title: Re: The Math for universal Acceleration IS INCORRECT
Post by: JSS on May 02, 2020, 12:05:31 PM
Sorry, what? You don't "take" a frame of reference to be inertial, it either is inertial or it isn't. Furthermore, the Earth is not a frame of reference, it is an object consisting of mostly rock and water. Objects are not frames of reference.

I think there is some confusing and mixing up between strict scientific definitions, and what they can be used for experimentally.

A point on the Earth is orbiting the sun, orbiting the center of the milky way, rotating, and standing on the surface means you are not in free-fall.

All of these things mean that a location on the Earth is not an inertial reference frame.

However, if you are just trying to calculate the interaction between two balls you are bouncing off each other, you can consider a point on or near the earth to be an approximate reference frame. Depending on the scale of the experiment, you can consider a point on the Earth stationary.

I think a lot of this argument is about semantics.

'Short' Answer: Earth is not X but a location on it can be considered an approximation of X for specific experiments.
Title: Re: The Math for universal Acceleration IS INCORRECT
Post by: pricelesspearl on May 02, 2020, 02:06:29 PM
Quote
Sorry, what? You don't "take" a frame of reference to be inertial, it either is inertial or it isn't.

So are you suggesting that proper acceleration is not relative? 
Title: Re: The Math for universal Acceleration IS INCORRECT
Post by: Pete Svarrior on May 02, 2020, 02:58:20 PM
So are you suggesting that proper acceleration is not relative?
You understood nothing that's been said to you. Instead of trying to trip people up with "clever" gotchas, please read up on the subject and try to develop an understanding.
Title: Re: The Math for universal Acceleration IS INCORRECT
Post by: BRrollin on May 02, 2020, 04:03:32 PM
Anyway, back to your point. I think you may have some confusion between inertial frames versus rest frames. I tried to help delineate this for you in my post (that you said you didn’t read).

I never said that. You're the one who, for some reason, keeps claiming I didn't read a post that I responded to, not me. Given that the post in question is utterly wrong, and contains such absurd phrases as "a noninertial frame with respect to", I don't have much regard for what you think I might be confused about.

Lemme know if you’d like to start fresh. We can renormalize to the crux of the matter, which is the distance and time scales where taking the Earth to be an inertial frame is appropriate and when it is not.

Sorry, what? You don't "take" a frame of reference to be inertial, it either is inertial or it isn't. Furthermore, the Earth is not a frame of reference, it is an object consisting of mostly rock and water. Objects are not frames of reference.

Oh dear, well it looks like this discussion has reached an impasse. Not sure where all the anger is coming from.

When you say that you stopped reading a reply, I take that to mean you didn’t read the rest of it...certainly you could be lying and I wouldn’t know. But why would you do that?

“The Earth is not a frame of reference.”

Yeah...this stuff is covered in like the first week of physics I. The earth is taken as a reference frame all the time. Look in any physics text. Literally ANY.

So it is clear that there is misunderstanding of the basics, yet you are accusing me of being wrong about them - and being rather rude about it too.

Hey - if you’re interested in learning, I’d be happy to teach you. Really sorry that there is such confusion.

But if you just want to hurl insults every time it is pointed out that you are mistaken, well cousin, to you I say good day sir! Some humility would be in order, especially given the obvious lack of understanding fundamental principles.
Title: Re: The Math for universal Acceleration IS INCORRECT
Post by: xasop on May 02, 2020, 05:25:46 PM
A point on the Earth is orbiting the sun, orbiting the center of the milky way, rotating, and standing on the surface means you are not in free-fall.

All of these things mean that a location on the Earth is not an inertial reference frame.

Actually, only the last one means you are not in free-fall. An orbit is a free-fall.

I think a lot of this argument is about semantics.

It is not. The point, which has apparently been missed by all involved, is that a frame of reference using a point on the Earth's surface as a fixed point approximates a non-inertial frame of reference precisely as well in FET as in RET. In both cases, there is a proper acceleration of 9.8 m s-2.

Oh dear, well it looks like this discussion has reached an impasse. Not sure where all the anger is coming from.

There is no anger, but I agree we seem to have reached an impasse, as you continue trying to "explain" incorrect physics to me rather than respond to my points. Teaching other people generally will not get you anywhere if they do not recognise you as more knowledgeable than they are.

“The Earth is not a frame of reference.”

Yeah...this stuff is covered in like the first week of physics I. The earth is taken as a reference frame all the time. Look in any physics text. Literally ANY.

Look, this is not correct, and I have studied physics at university level. The primary reason the Earth cannot be a frame of reference is that, particularly in RET, different parts of the Earth are moving at different velocities as it rotates. That is, different places on Earth are in different frames of reference, even in Newtonian physics.

Now, it is common in introductory physics textbooks to use one fixed point somewhere on the Earth's surface to define a frame of reference. This is very, very far from using the Earth itself as a frame of reference, which simply makes no sense at all.

The reason this is significant in this particular discussion is that it is possible in RET to select a point on the Earth—namely, its centre of mass—to define an inertial frame of reference, given modelling assumptions that ignore external forces such as the solar wind and meteor showers. But unless you have access to a clock measuring muon decay times located in the inner core, this is entirely irrelevant.

So it is clear that there is misunderstanding of the basics, yet you are accusing me of being wrong about them - and being rather rude about it too.

At the risk of devolving this discussion into personal attacks, might I point out that you have been accusing me of not understanding basics as well, even though your grasp of them is very obviously lacking? I don't appreciate the double standard of being called rude for returning the courtesy.

I would appreciate it if we could focus on the subject matter rather than you avoiding responding to my points by just telling me I don't understand basics.
Title: Re: The Math for universal Acceleration IS INCORRECT
Post by: BRrollin on May 02, 2020, 05:52:41 PM
A point on the Earth is orbiting the sun, orbiting the center of the milky way, rotating, and standing on the surface means you are not in free-fall.

All of these things mean that a location on the Earth is not an inertial reference frame.

Actually, only the last one means you are not in free-fall. An orbit is a free-fall.

I think a lot of this argument is about semantics.

It is not. The point, which has apparently been missed by all involved, is that a frame of reference using a point on the Earth's surface as a fixed point approximates a non-inertial frame of reference precisely as well in FET as in RET. In both cases, there is a proper acceleration of 9.8 m s-2.

Oh dear, well it looks like this discussion has reached an impasse. Not sure where all the anger is coming from.

There is no anger, but I agree we seem to have reached an impasse, as you continue trying to "explain" incorrect physics to me rather than respond to my points. Teaching other people generally will not get you anywhere if they do not recognise you as more knowledgeable than they are.

“The Earth is not a frame of reference.”

Yeah...this stuff is covered in like the first week of physics I. The earth is taken as a reference frame all the time. Look in any physics text. Literally ANY.

Look, this is not correct, and I have studied physics at university level. The primary reason the Earth cannot be a frame of reference is that, particularly in RET, different parts of the Earth are moving at different velocities as it rotates. That is, different places on Earth are in different frames of reference, even in Newtonian physics.

Now, it is common in introductory physics textbooks to use one fixed point somewhere on the Earth's surface to define a frame of reference. This is very, very far from using the Earth itself as a frame of reference, which simply makes no sense at all.

The reason this is significant in this particular discussion is that it is possible in RET to select a point on the Earth—namely, its centre of mass—to define an inertial frame of reference, given modelling assumptions that ignore external forces such as the solar wind and meteor showers. But unless you have access to a clock measuring muon decay times located in the inner core, this is entirely irrelevant.

So it is clear that there is misunderstanding of the basics, yet you are accusing me of being wrong about them - and being rather rude about it too.

At the risk of devolving this discussion into personal attacks, might I point out that you have been accusing me of not understanding basics as well, even though your grasp of them is very obviously lacking? I don't appreciate the double standard of being called rude for returning the courtesy.

I would appreciate it if we could focus on the subject matter rather than you avoiding responding to my points by just telling me I don't understand basics.

Alright, it looks like you’ve done some kind of an effort to extend an olive branch. I’ll meet you half way, even if my half is more like 3/4 ;)

I’ll believe you that you’ve studied physics in university. So then I’ll advance the conversation a bit to include your prior knowledge.

In astrophysics the ecliptic plane is defined in terms of the Earth’s rest frame. So this serves as an example that you absolutely CAN do this, and it happens at the introductory level. Moreover, the epicycle theory advanced by Tom uses this same reference point.

Furthermore, think back to your physics course. Did you compute a block sliding down an incline? I’m sure you did. How did you do this problem without introducing fictitious forces that arise from a noninertial frame?

You see, you’re missing my point. If you take two points on the Earth that are far away, then in the frame of one of them, the other will have fictitious forces (of course, in steradian measures they will not - so your used metric matters!). But if you have two objects close enough together (muon+detector), then the noninertial terms in your equations are small, and introduce an uncertainty into the result that is smaller than other systematic and random errors.

You do not need a clock at the inner core, because on the timescales of the experiment (really it’s the spacetime interval) the clock on the core and another on the surface will remain synchronized (enough). This is just straight forward basic relativity.

All these topic are studied in intro mechanics, upper division experimental physics courses (folks often DO the muon experiment here), and your first relativity course.

Now I don’t know how many courses you took in physics. If you tell me, then I would be able to hone my replies to better meet your understanding. Presently, I’m just guessing what you know. One on hand it seems like you know some definitions, but then those definitions are mis-applied. When I try to point this out you get defensive. I would guess you took two courses - mechanics and E&M. But let me know.

I’m here to help, Parsifal. I’m not your enemy. I’m not here to fight you.
Title: Re: The Math for universal Acceleration IS INCORRECT
Post by: xasop on May 02, 2020, 05:59:15 PM
Furthermore, think back to your physics course. Did you compute a block sliding down an incline? I’m sure you did. How did you do this problem without introducing fictitious forces that arise from a noninertial frame?

How would you do this without the fictitious force of gravity?

You do not need a clock at the inner core, because on the timescales of the experiment (really it’s the spacetime interval) the clock on the core and another on the surface will remain synchronized (enough).

I agree. But this is equally true in FET as in RET, which is my whole point. (Except, of course, that there is no inner core in FET, so replace "the core" with "an inertial frame of reference beginning at rest with respect to the clock on the surface".)
Title: Re: The Math for universal Acceleration IS INCORRECT
Post by: ShadySquid on May 02, 2020, 11:14:28 PM
There is nothing in Relativity that suggests any object could not undergo constant acceleration forever. If you disagree, please show your work.

It seems to me like it doesn't matter what frame of reference the Earth is accelerating in.  Regardless of the frame of reference, if the Earth is accelerating, it is gaining speed, and eventually, that speed will reach the speed of light, ipso facto, the speed can no longer increase and the Earth will stop accelerating.  The speed of light is the hard limit for speed in the universe, and an object under constant acceleration is going to reach it eventually.

You can think of it this way:  You have a 10 liter water bottle.  Your task is to constantly pour water into the water bottle at a rate of one liter per hour, forever, without spilling any.  This is completely impossible, because the water bottle can only hold ten liters.  No matter what you do to the water bottle, no matter where you put it, it is physically impossible to add more than 10 liters to the water bottle.  The speed of light is like that water bottle; you can only fill it with so much velocity.  If you are constantly adding velocity, no matter where you put the speed-of-light bottle, no matter what angle you look at it from, it has a hard limit to how much velocity you can add; ipso facto, infinite constant acceleration is impossible.

I know it's not a perfect analogy, because water and speed are two very different substances, but it gets the idea across.

So, am I missing something here?  Why does the frame of reference even matter?
Title: Re: The Math for universal Acceleration IS INCORRECT
Post by: BRrollin on May 03, 2020, 12:26:32 AM
Furthermore, think back to your physics course. Did you compute a block sliding down an incline? I’m sure you did. How did you do this problem without introducing fictitious forces that arise from a noninertial frame?

How would you do this without the fictitious force of gravity?

You do not need a clock at the inner core, because on the timescales of the experiment (really it’s the spacetime interval) the clock on the core and another on the surface will remain synchronized (enough).

I agree. But this is equally true in FET as in RET, which is my whole point. (Except, of course, that there is no inner core in FET, so replace "the core" with "an inertial frame of reference beginning at rest with respect to the clock on the surface".)

Fictitious forces are identified by not having a corresponding potential. The coriolis and centrifugal forces are examples. Gravity has a potential, and so derives from potential theory. Hence, by definition, gravity is not a fictitious force.

Going back to the muon then. If the Earth was accelerating with UA, then we would experience time dilation. So the muon’s lifetime would be shorter - it would “age” faster. If the muon instead was relativistic - rather than the Earth, then IT would be time dilated. We would age faster relative to it, and from our perspective, the muon would live longer.

Since the muon lifetime is measured experimentally, we can compare that value to muon production from cosmic ray cascades onto the atmosphere.

The results show that the muon lives longer. Ergo, it is relativistic. Not us. Ergo, UA fails this experimental observation.
Title: Re: The Math for universal Acceleration IS INCORRECT
Post by: xasop on May 03, 2020, 01:16:00 AM
Fictitious forces are identified by not having a corresponding potential. The coriolis and centrifugal forces are examples. Gravity has a potential, and so derives from potential theory. Hence, by definition, gravity is not a fictitious force.

If you accept general relativity and its description of gravitation, then gravity is a fictitious force. Do you accept general relativity?

Going back to the muon then. If the Earth was accelerating with UA, then we would experience time dilation.

Ditto if the Earth is round with Einsteinian gravitation.

So the muon’s lifetime would be shorter - it would “age” faster.

No, that does not follow. The muon is moving at relativistic speeds relative to us, therefore we observe its decay to take longer. Unlike the equivalence principle, this isn't even getting into general relativity, this is basic special relativity.

If the muon instead was relativistic - rather than the Earth
The results show that the muon lives longer. Ergo, it is relativistic. Not us.

Being "relativistic" is a relative property. I don't feel like this should need to be pointed out, as it is in the name, but here we go anyway. If two observers A and B are moving at 0.9c with respect to one another, then A is relativistic from B's frame of reference and B is relativistic from A's frame of reference. This is, again, basic special relativity.

Thus, a muon moving at 0.9c with respect to the Earth is the same thing as the Earth moving at 0.9c with respect to a muon. This follows directly from the idea that there is no absolute frame of reference, which you earlier agreed with.

It is difficult to respond to your points when you make such contradictory statements, because you are now throwing points we have already established agreement upon out of the window. Do we need to go back to basics?
Title: Re: The Math for universal Acceleration IS INCORRECT
Post by: BRrollin on May 03, 2020, 05:05:13 AM
Fictitious forces are identified by not having a corresponding potential. The coriolis and centrifugal forces are examples. Gravity has a potential, and so derives from potential theory. Hence, by definition, gravity is not a fictitious force.

If you accept general relativity and its description of gravitation, then gravity is a fictitious force. Do you accept general relativity?

Going back to the muon then. If the Earth was accelerating with UA, then we would experience time dilation.

Ditto if the Earth is round with Einsteinian gravitation.

So the muon’s lifetime would be shorter - it would “age” faster.

No, that does not follow. The muon is moving at relativistic speeds relative to us, therefore we observe its decay to take longer. Unlike the equivalence principle, this isn't even getting into general relativity, this is basic special relativity.

If the muon instead was relativistic - rather than the Earth
The results show that the muon lives longer. Ergo, it is relativistic. Not us.

Being "relativistic" is a relative property. I don't feel like this should need to be pointed out, as it is in the name, but here we go anyway. If two observers A and B are moving at 0.9c with respect to one another, then A is relativistic from B's frame of reference and B is relativistic from A's frame of reference. This is, again, basic special relativity.

Thus, a muon moving at 0.9c with respect to the Earth is the same thing as the Earth moving at 0.9c with respect to a muon. This follows directly from the idea that there is no absolute frame of reference, which you earlier agreed with.

It is difficult to respond to your points when you make such contradictory statements, because you are now throwing points we have already established agreement upon out of the window. Do we need to go back to basics?

So we are not at the scale of General relativity, we are in the weak field limit which reduces to Newton. But even if you appeal to GR’s construction of gravity as geometry, that does not posit gravity as a fictitious force in classical mechanics. That is just not the correct definition. I understand your argument, but GR does not somehow make gravity a fictitious force in CM. In CM, you define fictitious forces as any force which lacks a potential function in the lagrangian formulation. Gravity has this. The definitions are just more rigorous than how you are using them.

“Ditto if the Earth is round with Einsteinian gravitation.”

What is Einsteinian gravitation? You mean GR? 

“Thus, a muon moving at 0.9c with respect to the Earth is the same thing as the Earth moving at 0.9c with respect to a muon. This follows directly from the idea that there is no absolute frame of reference, which you earlier agreed with.”

This is true. You are correct. But you’re missing the whole point. I am failing at getting you to see this. The fault is mine. Let me try one more time:

In UA, we are not an inertial frame. Yes? I think that is clear.  So you can’t USE special relativity.

This is what I’ve been trying to tell you this whole time. To use special relativity in the first place, you MUST have inertial frames. It’s in the postulates!

This is why the twin paradox is not really a paradox. This puzzle is used to teach students which situations can use SR and which cannot.

Hence, if you hold that UA is correct, then you automatically lose relativity as method for investigating consequences.

UA must either:

(A) invent a new theory to replace SR that is consistent with UA and predicts the muon lifetimes and dilations,

(B) bow out of the muon debate.

(C) some other option I haven’t thought of...

But, you absolutely cannot use SR principles and apply them to UA.

I hope this is helpful. I’m not trying to “beat” you in a debate, rather it is unfortunate to see anyone spend time on a dead-end endeavor.
Title: Re: The Math for universal Acceleration IS INCORRECT
Post by: xasop on May 03, 2020, 05:25:14 AM
So we are not at the scale of General relativity, we are in the weak field limit which reduces to Newton.

Please stop with this double standard. Either you can treat a 9.8 m s-2 acceleration as negligible in both RET and UA, or neither. Given that your whole argument is based on treating UA as non-negligible, we must use general relativity in RET to compare apples to apples.

But even if you appeal to GR’s construction of gravity as geometry, that does not posit gravity as a fictitious force in classical mechanics.

Indeed, GR instead posits that classical mechanics is incorrect, albeit useful in many non-relativistic situations. For the purposes of this discussion, the distinction between classical mechanics being wrong and classical mechanics treating gravity as a fictitious force is academic.

What is Einsteinian gravitation? You mean GR?

Yes.

In UA, we are not an inertial frame. Yes? I think that is clear.  So you can’t USE special relativity.

Then we go back to my point that we are not in an inertial frame of reference according to RET, either. Your argument for why this doesn't matter was that the discrepancy between a non-inertial clock and an inertial one would be negligible in the time it takes for a muon to decay.

Either that argument holds, and special relativity applies with UA, or it doesn't hold and special relativity does not apply with RET. Once again, I would like to ask you to make up your mind.

Bottom line, I would like you to answer this one question. Is a 9.8 m s-2 proper acceleration in the observer's frame of reference negligible for the purposes of the muon experiment?
Title: Re: The Math for universal Acceleration IS INCORRECT
Post by: BRrollin on May 03, 2020, 12:10:26 PM
So we are not at the scale of General relativity, we are in the weak field limit which reduces to Newton.

Please stop with this double standard. Either you can treat a 9.8 m s-2 acceleration as negligible in both RET and UA, or neither. Given that your whole argument is based on treating UA as non-negligible, we must use general relativity in RET to compare apples to apples.

But even if you appeal to GR’s construction of gravity as geometry, that does not posit gravity as a fictitious force in classical mechanics.

Indeed, GR instead posits that classical mechanics is incorrect, albeit useful in many non-relativistic situations. For the purposes of this discussion, the distinction between classical mechanics being wrong and classical mechanics treating gravity as a fictitious force is academic.

What is Einsteinian gravitation? You mean GR?

Yes.

In UA, we are not an inertial frame. Yes? I think that is clear.  So you can’t USE special relativity.

Then we go back to my point that we are not in an inertial frame of reference according to RET, either. Your argument for why this doesn't matter was that the discrepancy between a non-inertial clock and an inertial one would be negligible in the time it takes for a muon to decay.

Either that argument holds, and special relativity applies with UA, or it doesn't hold and special relativity does not apply with RET. Once again, I would like to ask you to make up your mind.

Bottom line, I would like you to answer this one question. Is a 9.8 m s-2 proper acceleration in the observer's frame of reference negligible for the purposes of the muon experiment?

Lol, there is no double standard here. You keep participating as though one of us must “win.” Why this insistence for an adversarial exchange? 

The existence of a free-fall acceleration does not guarantee the applicability of GR.

It looks like you are now trying to pivot to GR as a comparative theory rather than SR. That’s probably a more lucrative strategy, and I am happy to discuss GR with you, but be warned, it is incredibly difficult.

Yes, I know the proper definition of fictitious forces is academic. I’ll level with ya: most of the epicyclic subtopics we have been discussing have been academic, but with the intention of being helpful, I have been patient.

“Either that argument holds, and special relativity applies with UA, or it doesn't hold and special relativity does not apply with RET. Once again, I would like to ask you to make up your mind.”

This is a false dichotomy. Special relativity does not hold in UA, but does apply in RET.

“Bottom line, I would like you to answer this one question. Is a 9.8 m s-2 proper acceleration in the observer's frame of reference negligible for the purposes of the muon experiment?”

NO! It is a dealbreaker! In UA,
that is indeed your proper acceleration, and SR is inapplicable. In RET, your proper acceleration standing on the earth’s surface is ZERO - because the Earth’s surface is in the way! If you’re in free-fall, then yes, now you’re fucked.

In UA, just by standing on the Earth, you are in a noninertial frame. In RET, standing on the Earth, you are in an inertial frame.
Title: Re: The Math for universal Acceleration IS INCORRECT
Post by: xasop on May 03, 2020, 06:29:27 PM
“Bottom line, I would like you to answer this one question. Is a 9.8 m s-2 proper acceleration in the observer's frame of reference negligible for the purposes of the muon experiment?”

NO! It is a dealbreaker! In UA,
that is indeed your proper acceleration, and SR is inapplicable. In RET, your proper acceleration standing on the earth’s surface is ZERO - because the Earth’s surface is in the way! If you’re in free-fall, then yes, now you’re fucked.

In UA, just by standing on the Earth, you are in a noninertial frame. In RET, standing on the Earth, you are in an inertial frame.

Nope. GR says that free-fall is an inertial frame of reference because you are following a geodesic in space-time. Being in contact with the surface of the Earth causes an upward proper acceleration of 9.8 m s-2 to stop you from falling, which is why gravity is a fictitious force. This is the equivalence principle.

So, given that your answer is "NO! It is a dealbreaker!", the clocks in the muon experiment cannot be considered inertial in either UA or RET with GR. As you have already indicated that you accept GR, I don't see how your case remains at all defensible.

Indeed, since you have asserted that a consequence of such a proper acceleration is that our observations would be inconsistent with reality, you must now surely conclude that neither UA nor GR can be valid, but I'll let you grapple with that one on your own.
Title: Re: The Math for universal Acceleration IS INCORRECT
Post by: BRrollin on May 03, 2020, 06:56:39 PM
“Bottom line, I would like you to answer this one question. Is a 9.8 m s-2 proper acceleration in the observer's frame of reference negligible for the purposes of the muon experiment?”

NO! It is a dealbreaker! In UA,
that is indeed your proper acceleration, and SR is inapplicable. In RET, your proper acceleration standing on the earth’s surface is ZERO - because the Earth’s surface is in the way! If you’re in free-fall, then yes, now you’re fucked.

In UA, just by standing on the Earth, you are in a noninertial frame. In RET, standing on the Earth, you are in an inertial frame.

Nope. GR says that free-fall is an inertial frame of reference because you are following a geodesic in space-time. Being in contact with the surface of the Earth causes an upward proper acceleration of 9.8 m s-2 to stop you from falling, which is why gravity is a fictitious force. This is the equivalence principle.

So, given that your answer is "NO! It is a dealbreaker!", the clocks in the muon experiment cannot be considered inertial in either UA or RET with GR. As you have already indicated that you accept GR, I don't see how your case remains at all defensible.

Indeed, since you have asserted that a consequence of such a proper acceleration is that our observations would be inconsistent with reality, you must now surely conclude that neither UA nor GR can be valid, but I'll let you grapple with that one on your own.

I think you may benefit from some additional study of inertial frames. This also serves as published evidence for my statements:

https://arxiv.org/pdf/1310.4465.pdf

The read is technical and long, but you WERE warned.

Since you seem adamant about not believing the current dictates of physics, I will let the article speak for itself. We can discuss the points within it, but I won’t further discuss issues beyond it - unless you likewise provide published evidence as context.

The discussion has devolved into:

Me: this is what physics says.

You: no, THIS is what physics says.

So we turn to published physics, and discuss what it says :)
Title: Re: The Math for universal Acceleration IS INCORRECT
Post by: xasop on May 03, 2020, 07:21:42 PM
We can discuss the points within it, but I won’t further discuss issues beyond it - unless you likewise provide published evidence as context.

That's fine by me. As far as I'm concerned, your objections to UA have been adequately refuted, and we can now all get on with more productive discussions in peace.
Title: Re: The Math for universal Acceleration IS INCORRECT
Post by: BRrollin on May 03, 2020, 08:07:42 PM
We can discuss the points within it, but I won’t further discuss issues beyond it - unless you likewise provide published evidence as context.

That's fine by me. As far as I'm concerned, your objections to UA have been adequately refuted, and we can now all get on with more productive discussions in peace.

Well, no, they haven’t, lol. The article will help you learn why you are mistaken in your understanding of inertial frames in GR. Why in UA it is a noninertial frame, and why in RET it is an inertial frame.

I have provided published evidence to support my position - that’s the whole point of linking it.

It is bizarre that you think the matter has somehow now been closed! I have evidence in support, but as far as I can tell, you have only the recalcitrant insistence that your claims are de facto correct.

It is your prerogative of course, but I am more concerned with my beliefs being true :)
Title: Re: The Math for universal Acceleration IS INCORRECT
Post by: xasop on May 03, 2020, 09:31:55 PM
It is your prerogative of course, but I am more concerned with my beliefs being true :)

And I'm fine with letting you believe what you want. Just please stop wasting people's time with poorly concocted arguments for why they shouldn't maintain the existing UA model.
Title: Re: The Math for universal Acceleration IS INCORRECT
Post by: Groit on May 04, 2020, 10:58:56 AM

Being "relativistic" is a relative property. I don't feel like this should need to be pointed out, as it is in the name, but here we go anyway. If two observers A and B are moving at 0.9c with respect to one another, then A is relativistic from B's frame of reference and B is relativistic from A's frame of reference. This is, again, basic special relativity.

Thus, a muon moving at 0.9c with respect to the Earth is the same thing as the Earth moving at 0.9c with respect to a muon. This follows directly from the idea that there is no absolute frame of reference, which you earlier agreed with.

Alright then, so we can say that the muons are travelling at some non-relativistic speed, and the Earth is accelerating towards them. When scientists perform the experiment, they find that their relative velocity is 0.9 c, that's fine, UA works and it seems reasonable...

However, one week later the scientist carry out the same experiment again, and bear in mind that for the past week the Earth has been accelerating so its velocity has increased by at
They now find that more muons are reaching the surface of the Earth and thus their relative velocity and time dilation has increased. A week later and they do another test, again different results... and so on.

UA model does not fit into the way we measure muons at the surface of the Earth. This experiment has been done time and time again over many years and the results are always the same, this is because the Earth is moving at a non-relativistic speed i.e. in its orbit around the sun etc... and the muons are travelling towards the Earth at approx 0.98 c. There is no acceleration for the Earth or muons.
Title: Re: The Math for universal Acceleration IS INCORRECT
Post by: xasop on May 04, 2020, 01:41:23 PM
However, one week later the scientist carry out the same experiment again, and bear in mind that for the past week the Earth has been accelerating so its velocity has increased by at
They now find that more muons are reaching the surface of the Earth and thus their relative velocity and time dilation has increased. A week later and they do another test, again different results... and so on.

We don't observe this. What are you basing this claim on?
Title: Re: The Math for universal Acceleration IS INCORRECT
Post by: BRrollin on May 04, 2020, 02:23:56 PM
However, one week later the scientist carry out the same experiment again, and bear in mind that for the past week the Earth has been accelerating so its velocity has increased by at
They now find that more muons are reaching the surface of the Earth and thus their relative velocity and time dilation has increased. A week later and they do another test, again different results... and so on.

We don't observe this. What are you basing this claim on?

Precisely, this change in the muon spectrum is NOT observed - but should be in UA.

Here’s an article that provides details:

https://cds.cern.ch/record/427778/files/0002052.pdf

Since an increasing muon energy would impact other observables, such as neutron production, so we should see an increase in those energies too. But we dont:

https://scholar.google.com/scholar?q=cosmic+ray+muons&hl=en&as_sdt=0&as_vis=1&oi=scholart#d=gs_qabs&u=%23p%3D7KRZ0fQ4F94J

The Particle Data Group has a nice summary article of muon observations. Note the lack of increasing muon energies with respect to time:

http://pdg.lbl.gov/2011/reviews/rpp2011-rev-cosmic-rays.pdf

I’ll finally note that these observations have been been conducted for decades. So there is a literal anthology of observations that UA doesn’t match.
Title: Re: The Math for universal Acceleration IS INCORRECT
Post by: xasop on May 04, 2020, 11:25:09 PM
Precisely, this change in the muon spectrum is NOT observed - but should be in UA.

You didn't answer my question. Why do you claim that it should be in UA?
Title: Re: The Math for universal Acceleration IS INCORRECT
Post by: BRrollin on May 05, 2020, 02:40:07 AM
Precisely, this change in the muon spectrum is NOT observed - but should be in UA.

You didn't answer my question. Why do you claim that it should be in UA?

We’ve...already discussed this. Are you messing with me? I don’t want to just be repeating myself....

Let’s go point by point. Maybe you can indicate where we are missing each other.

What follows is with respect to a muon detection event on the Earth’s surface using probably a scintillator and amplifier. The muon origin is a cosmic ray that creates a cascade upon interaction with the atmosphere.

Sorry, not trying to be pedantic, just getting super precise in order to find our mismatch. If you disagree with the situation above, please indicate. The points then follow:

1. In UA, we are in a noninertial frame. Agree?

2. In RET, we are in an inertial frame. Agree?

3. If agree to 1 above then SR is invalid to use. Agree?

4. If agree to 2 above then SR is valid to use. Agree?

We may end up stopped by this point to discuss. But in case not:

5. GR on the earth’s surface is a weak field limit. The spacetime curvature is small - that is, the Riemann tensor is perturbative in Minkowski space. Upon completing the expansion, Einstein’s field equations reduce to Newtonian mechanics.

Therefore, to appeal to GR in UA means to accept newton - it is hence self-contradictory, because UA assumes no Newton.

You have constructed a proof by contradiction against UA in using GR.

The only way to avoid this is to refuse the weak field limit. This is incorrect GR, but presumably you wish to modify it in some fashion. I assume so, otherwise why are we even discussing it?

The most direct way to modify it is to absorb the weak field expansion terms into the stress energy tensor. This preserves the equality. I assume this is what you intend...

That means objects entering that spacetime will encounter a shift in their spectrum. But what reference frame do you COMPARE this shift?

I hope you got something out of that article I sent you on GR frames.

Anyway, you simply measure two inbound muons - with some time delay between them. Since you are in a noninertial frame, the stress energy tensor must absorb more terms as time proceeds. So you compare the spectrum to what you got a week ago, or whatever.

But this is not observed.

I’m trying to help you construct a different modification to GR that mighty be better - hence the articles I sent last time.

I look forward to your reply :)
Title: Re: The Math for universal Acceleration IS INCORRECT
Post by: xasop on May 05, 2020, 07:37:02 AM
1. In UA, we are in a noninertial frame. Agree?

Agree.

2. In RET, we are in an inertial frame. Agree?

Disagree, as I have stated previously. But even if we cannot come to an agreement on this point...

3. If agree to 1 above then SR is invalid to use. Agree?

Disagree, because for the purposes of the muon experiment, the difference in observation from an inertial frame of reference is negligible.

Let's try a thought experiment. Suppose we hollow out the Burj Khalifa into a vertical hyperloop. We can now send objects through it that can take measurements as inertial observers, unaffected by the Earth's acceleration.

Now, launch a clock upwards into the Burj Khalifa, which will rise to the top and then fall back down. As it is not being accelerated by UA, it is an inertial observer which can be analysed using SR.

My assertion is that the difference between the muon decay time measured by a clock on the surface of the Earth, and that measured by our Burj hyperloop clock, will be so tiny as to be immeasurable, and in any case insignificant for the purposes of the muon experiment. Agree?
Title: Re: The Math for universal Acceleration IS INCORRECT
Post by: BRrollin on May 05, 2020, 12:29:39 PM
1. In UA, we are in a noninertial frame. Agree?

Agree.

2. In RET, we are in an inertial frame. Agree?

Disagree, as I have stated previously. But even if we cannot come to an agreement on this point...

3. If agree to 1 above then SR is invalid to use. Agree?

Disagree, because for the purposes of the muon experiment, the difference in observation from an inertial frame of reference is negligible.

Let's try a thought experiment. Suppose we hollow out the Burj Khalifa into a vertical hyperloop. We can now send objects through it that can take measurements as inertial observers, unaffected by the Earth's acceleration.

Now, launch a clock upwards into the Burj Khalifa, which will rise to the top and then fall back down. As it is not being accelerated by UA, it is an inertial observer which can be analysed using SR.

My assertion is that the difference between the muon decay time measured by a clock on the surface of the Earth, and that measured by our Burj hyperloop clock, will be so tiny as to be immeasurable, and in any case insignificant for the purposes of the muon experiment. Agree?

Good, this is useful for us.

For disagreement on 2. Yes, the statement is false. We are not in inertial frame in RET, because the Earth is spinning and in orbit. But on the timescales and distance scales of muon experiment, those accelerations are negligible. This is why we can perform table top experiments on earth and not get fictitious terms.

For the Burj example.

Absolutely disagree. The launched clock is not in an inertial frame, because it must accelerate to change from going up to going down.

We will probably be at an impasse until we thoroughly examine this disagreement.

You might want to look up the twin paradox - it’s resolution explains why your example cannot be accurate.

As for your assertion on synchronous clocks in the muon experiment. If you attempt to apply SR to this situation, you will not have unique world lines for your events. Moreover, if you assume synchronous clocks, you will identify contradictory space time events that will not be light-like.

Hence, it cannot be a correct approximation to treat UA as inertial. 

Not to be bossy, but it would probably be illuminating if you went through the computation. That way, you would “see it for yourself.” Which always seems to be more impactful than if someone else shows it.
Title: Re: The Math for universal Acceleration IS INCORRECT
Post by: xasop on May 05, 2020, 12:37:18 PM
Absolutely disagree. The launched clock is not in an inertial frame, because it must accelerate to change from going up to going down.

This just shows that you don't even understand the basic principle of UA, let alone being able to discredit it. Under UA, the Earth only accelerates up. Free-falling projectiles do not accelerate down. A projectile that goes up and then down again to an observer on Earth is not accelerating, it is the observer on Earth who accelerates.
Title: Re: The Math for universal Acceleration IS INCORRECT
Post by: BRrollin on May 05, 2020, 04:47:26 PM
Absolutely disagree. The launched clock is not in an inertial frame, because it must accelerate to change from going up to going down.

This just shows that you don't even understand the basic principle of UA, let alone being able to discredit it. Under UA, the Earth only accelerates up. Free-falling projectiles do not accelerate down. A projectile that goes up and then down again to an observer on Earth is not accelerating, it is the observer on Earth who accelerates.

Not according to the definition of acceleration. An object that changes its direction of motion accelerates. An object starting on the Earth in UA is already accelerating. As it moves up, it must accelerate even more in order to do so. It then must decelerate to stop, and the Earth to catch up.

Just draw a free body diagram for this. Or try to demonstrate your claim using kinematics equations. You will see this.
Title: Re: The Math for universal Acceleration IS INCORRECT
Post by: xasop on May 06, 2020, 04:39:41 AM
Not according to the definition of acceleration. An object that changes its direction of motion accelerates. An object starting on the Earth in UA is already accelerating. As it moves up, it must accelerate even more in order to do so. It then must decelerate to stop, and the Earth to catch up.

No. This is basic high school Newtonian mechanics. If object A moves with a constant velocity, and object B accelerates in the direction of motion of A, then B will eventually overtake A.

I'm not even going to bother demonstrating this because you've been talking down to me this whole time, claiming (or at least implying) that your understanding of relativity surpasses mine. You have just proven beyond any doubt that you are either lying or trolling, so engaging you further is pointless.
Title: Re: The Math for universal Acceleration IS INCORRECT
Post by: BRrollin on May 06, 2020, 04:52:18 AM
Not according to the definition of acceleration. An object that changes its direction of motion accelerates. An object starting on the Earth in UA is already accelerating. As it moves up, it must accelerate even more in order to do so. It then must decelerate to stop, and the Earth to catch up.

No. This is basic high school Newtonian mechanics. If object A moves with a constant velocity, and object B accelerates in the direction of motion of A, then B will eventually overtake A.

I'm not even going to bother demonstrating this because you've been talking down to me this whole time, claiming (or at least implying) that your understanding of relativity surpasses mine. You have just proven beyond any doubt that you are either lying or trolling, so engaging you further is pointless.

Hmmm. In UA, the earth accelerates up at 9.8 m/s^2. A rock stationary on the ground must also be accelerating at this value. Now I pick up the rock, and throw it upward. In doing so, it’s speed has changed relative to the earth. That means it’s acceleration is now no longer 9.8, but a value greater. That’s why the distance between the rock and the Earth changes.

They were moving at the same velocity. Now there is a displacement, which means their velocity changed.

Definition of acceleration is change in velocity over time.

I’m guessing you don’t want to show the equations because you realize your mistake. You must, since you claim you’ve taken physics, and this stuff is covered in the first week.

I’m not going to rub it in. We all make mistakes, and I don’t want to be a jerk. I’m more interested in acknowledging the truth and then continuing on.

I am enjoying our discussion :)
Title: Re: The Math for universal Acceleration IS INCORRECT
Post by: Pete Svarrior on May 07, 2020, 09:22:43 AM
I’m not going to rub it in. We all make mistakes, and I don’t want to be a jerk. I’m more interested in acknowledging the truth and then continuing on.
I'm going to have to ask you to stop. Nearly early post of yours in this thread includes a long passage about how much of a super expert you are, and other tangents which can be described as "discussing individuals, not ideas".

We don't do that in the upper. Obviously it will happen to everyone every now and then, but it doesn't look like a one-off oopsie with you.

Secondly, an object that's currently being thrown up is not free-falling, and Parsifal was pretty clear in declaring that that's what he's discussing. You're focusing on the "projectile that goes up" and pretending not to notice that that's simply the set up phase of the thought experiment. I'm not surprised Parsifal missed your little switcheroo, since, y'know, you're supposed to be talking about free-fall.

I asked you to post in good faith, and it was precisely to prevent these sort of non-arguments. Behave.
Title: Re: The Math for universal Acceleration IS INCORRECT
Post by: BRrollin on May 07, 2020, 01:19:48 PM
I’m not going to rub it in. We all make mistakes, and I don’t want to be a jerk. I’m more interested in acknowledging the truth and then continuing on.
I'm going to have to ask you to stop. Nearly early post of yours in this thread includes a long passage about how much of a super expert you are, and other tangents which can be described as "discussing individuals, not ideas".

We don't do that in the upper. Obviously it will happen to everyone every now and then, but it doesn't look like a one-off oopsie with you.

Secondly, an object that's currently being thrown up is not free-falling, and Parsifal was pretty clear in declaring that that's what he's discussing. You're focusing on the "projectile that goes up" and pretending not to notice that that's simply the set up phase of the thought experiment. I'm not surprised Parsifal missed your little switcheroo, since, y'know, you're supposed to be talking about free-fall.

I asked you to post in good faith, and it was precisely to prevent these sort of non-arguments. Behave.

I do believe I’m posting in good faith. I’m not trying to muddy the waters or create conflict. Now certainly misunderstanding will happen, but I am doing my best to identify the points of misunderstanding and clear them up.

The issue at hand, Pete, is that we are not in an inertial frame in UA hence SR is invalid. This is what Parsifal disagreed with in the earlier posts.

“Since we are not in an inertial frame in UA, SR is invalid.

Disagree”

The Burj example cited in the disagreement involves motion upwards and then downwards. It was not clear to me that he wanted only to analyse the motion downward - otherwise why include the motion upwards in the first place? Just have an object fall, no hyperloop needed.

But this is immaterial, because even with a falling object, WE are still accelerating in UA, so WE are not in an inertial frame.

Parsifal agreed with this statement initially.

I am not implying that my knowledge of SR is better than anyone else’s, I’m just trying to get to the bottom of this - just like everyone else, and posting my current understanding of the situation.

Also, from some googling, I see that “free-fall” is defined as an object near the surface of the earth that is being influenced only by earth’s gravity. In the UA situation, I suppose this translates to an object near the surface of the Earth while the Earth accelerated upwards. So if my understanding is correct, then an object can be in free-fall regardless of whether it’s instantaneous velocity is pointing up or down.

I think these discussions are useful in figuring out the details of the UA idea. And even if not all aspects are resolved, they help better mold everyone’s conception of the idea, and further lay foundations for developments.

This is my intention. Sorry it has come off as anything else.
Title: Re: The Math for universal Acceleration IS INCORRECT
Post by: totallackey on May 07, 2020, 01:55:04 PM
I asked you to post in good faith, and it was precisely to prevent these sort of non-arguments. Behave.

I do believe I’m posting in good faith. I’m not trying to muddy the waters or create conflict.

^ THIS...in the world of BRrollin...equals...

...but I am more concerned with my beliefs being true :)
^THIS

Sorry...I ain't buyin...
Title: Re: The Math for universal Acceleration IS INCORRECT
Post by: juner on May 07, 2020, 02:50:49 PM
I asked you to post in good faith, and it was precisely to prevent these sort of non-arguments. Behave.

I do believe I’m posting in good faith. I’m not trying to muddy the waters or create conflict.

^ THIS...in the world of BRrollin...equals...

...but I am more concerned with my beliefs being true :)
^THIS

Sorry...I ain't buyin...

totallackey, while I agree with you, this isn't the thread or forum for it.


QED, you can drop the act of "I only want to get to the bottom of it..." Your history and behavior are well known. It seems you are being allowed to post for now, even though your past account was permanently banned for the same nonsense. Anyway, there won't be a long process of warnings and short bans, it will just be a permaban of this alt account in accordance with rule 8.
Title: Re: The Math for universal Acceleration IS INCORRECT
Post by: BRrollin on May 07, 2020, 03:14:38 PM
I asked you to post in good faith, and it was precisely to prevent these sort of non-arguments. Behave.

I do believe I’m posting in good faith. I’m not trying to muddy the waters or create conflict.

^ THIS...in the world of BRrollin...equals...

...but I am more concerned with my beliefs being true :)
^THIS

Sorry...I ain't buyin...

totallackey, while I agree with you, this isn't the thread or forum for it.


QED, you can drop the act of "I only want to get to the bottom of it..." Your history and behavior are well known. It seems you are being allowed to post for now, even though your past account was permanently banned for the same nonsense. Anyway, there won't be a long process of warnings and short bans, it will just be a permaban of this alt account in accordance with rule 8.

What? I have never been banned from this site! But I admit I have had other accounts, but those were using other computers...

Why all this sudden hostility? There is no animosity from my end. I have been polite and cordial, even though that has not always been returned.

This seems like a witch-hunt, the mods ganging up on a poster once the admin got frustrated.

Well, if that’s the case, then that is really childish. I’ve made mistakes in my posts before. So what? Everyone does. Reacting with maturity and humility is an expected standard in collaboration.
Title: Re: The Math for universal Acceleration IS INCORRECT
Post by: Groit on May 08, 2020, 04:02:25 PM
“Bottom line, I would like you to answer this one question. Is a 9.8 m s-2 proper acceleration in the observer's frame of reference negligible for the purposes of the muon experiment?”

NO! It is a dealbreaker! In UA,
that is indeed your proper acceleration, and SR is inapplicable. In RET, your proper acceleration standing on the earth’s surface is ZERO - because the Earth’s surface is in the way! If you’re in free-fall, then yes, now you’re fucked.

In UA, just by standing on the Earth, you are in a noninertial frame. In RET, standing on the Earth, you are in an inertial frame.

Nope. GR says that free-fall is an inertial frame of reference because you are following a geodesic in space-time. Being in contact with the surface of the Earth causes an upward proper acceleration of 9.8 m s-2 to stop you from falling, which is why gravity is a fictitious force. This is the equivalence principle.

There is an equal and opposite force from the Earth, but the acceleration would be %5Cfrac%7BF%7D%7Bm%7D  where m is the mass of the Earth, so the acceleration very, very small.

 Can't believe I've just said the Earth is accelerating upwards  ;)

Title: Re: The Math for universal Acceleration IS INCORRECT
Post by: pricelesspearl on May 08, 2020, 04:34:45 PM
Quote
There is an equal and opposite force from the Earth, but the acceleration would be %5Cfrac%7BF%7D%7Bm%7D  where m is the mass of the Earth, so the acceleration very, very small.

Can't believe I've just said the Earth is accelerating upwards  ;)

The "earth" isn't accelerating.  The ground directly beneath you is accelerating you.  That's an important distinction. The acceleration is caused by the normal force, so if there is nothing in contact with the ground...there is no acceleration. 

Title: Re: The Math for universal Acceleration IS INCORRECT
Post by: BRrollin on May 08, 2020, 04:53:01 PM
Quote
There is an equal and opposite force from the Earth, but the acceleration would be %5Cfrac%7BF%7D%7Bm%7D  where m is the mass of the Earth, so the acceleration very, very small.

Can't believe I've just said the Earth is accelerating upwards  ;)

The "earth" isn't accelerating.  The ground directly beneath you is accelerating you.  That's an important distinction. The acceleration is caused by the normal force, so if there is nothing in contact with the ground...there is no acceleration.

A point of clarification on behalf of UA: from my (imperfect) readings, it could be that the air above the earth’s surface accelerated too - like everything contained in the dome undergoes UA. If true, then contact with the ground is not needed.

I guess in that case the acceleration on us is not caused by a normal force but by whatever drives UA.

I could be mistaken - I am not a UA expert, just wanted to point out a way UA could be interpreted to match observations.
Title: Re: The Math for universal Acceleration IS INCORRECT
Post by: JSS on May 08, 2020, 05:27:22 PM
Quote
There is an equal and opposite force from the Earth, but the acceleration would be %5Cfrac%7BF%7D%7Bm%7D  where m is the mass of the Earth, so the acceleration very, very small.

Can't believe I've just said the Earth is accelerating upwards  ;)

The "earth" isn't accelerating.  The ground directly beneath you is accelerating you.  That's an important distinction. The acceleration is caused by the normal force, so if there is nothing in contact with the ground...there is no acceleration.

What part of the ground is undergoing the acceleration? If the whole Earth was, then it would effectively be weightless and wouldn't stick together once you dug into it. You could pick up a rock and it would float there if it was being accelerated.

So there has to be a layer underground that is being accelerated, and pushing the rest of the Earth (dirt, rock, mountains, water) upwards.

Any idea how far down it is, and what it's made of?
Title: Re: The Math for universal Acceleration IS INCORRECT
Post by: pricelesspearl on May 08, 2020, 07:05:38 PM
Quote
What part of the ground is undergoing the acceleration? If the whole Earth was, then it would effectively be weightless and wouldn't stick together once you dug into it. You could pick up a rock and it would float there if it was being accelerated.

So there has to be a layer underground that is being accelerated, and pushing the rest of the Earth (dirt, rock, mountains, water) upwards.

Any idea how far down it is, and what it's made of?

I think you might misunderstand me.  I wasn't arguing for UA.  I was just making the point that the "acceleration" caused by the normal force isn't the same thing as "the earth accelerates" in UA.
Title: Re: The Math for universal Acceleration IS INCORRECT
Post by: JSS on May 08, 2020, 07:26:52 PM
Quote
What part of the ground is undergoing the acceleration? If the whole Earth was, then it would effectively be weightless and wouldn't stick together once you dug into it. You could pick up a rock and it would float there if it was being accelerated.

So there has to be a layer underground that is being accelerated, and pushing the rest of the Earth (dirt, rock, mountains, water) upwards.

Any idea how far down it is, and what it's made of?

I think you might misunderstand me.  I wasn't arguing for UA.  I was just making the point that the "acceleration" caused by the normal force isn't the same thing as "the earth accelerates" in UA.

Ah, too bad, I was hoping to get some answers. Perhaps someone else knows? Is there some sort of pusher-plate down there?

But I agree, under UA if you had a very tall vacuum tube, a projectile fired upwards would experience no forces as it was overtaken by the accelerating Earth and passed.
Title: Re: The Math for universal Acceleration IS INCORRECT
Post by: Groit on May 08, 2020, 08:28:36 PM
Quote
There is an equal and opposite force from the Earth, but the acceleration would be %5Cfrac%7BF%7D%7Bm%7D  where m is the mass of the Earth, so the acceleration very, very small.

Can't believe I've just said the Earth is accelerating upwards  ;)

The "earth" isn't accelerating.  The ground directly beneath you is accelerating you.  That's an important distinction. The acceleration is caused by the normal force, so if there is nothing in contact with the ground...there is no acceleration.

There's an upward gravitational force that the small mass exerts on the Earth, which is equal to its weight. Small masses accelerate towards the Earth, and so does the Earth towards the small mass, but the acceleration for the Earth is very, very small. Too small to be measured but its there.
Title: Re: The Math for universal Acceleration IS INCORRECT
Post by: Groit on May 08, 2020, 09:48:30 PM
If an apple with mass 0.1 kg falls from a tree, what is the acceleration of the Earth towards the apple?
Title: Re: The Math for universal Acceleration IS INCORRECT
Post by: BRrollin on May 09, 2020, 12:12:03 AM
If an apple with mass 0.1 kg falls from a tree, what is the acceleration of the Earth towards the apple?

Assuming the only force on the earth is from the apple:

a = Gm/r^2

Where a is the acceleration of the earth, G is Newton’s gravitational constant, m is the apple mass, and r is the distance between the center of the earth and the apple.

Easier way:

AM=am

A is acceleration of earth, M is earth mass, a is apple acceleration (9.8), m is apple mass.

First way uses Newton’s gravitational force directly. Second uses Newton’s third law pair forces.
Title: Re: The Math for universal Acceleration IS INCORRECT
Post by: pricelesspearl on May 09, 2020, 01:06:01 AM
If an apple with mass 0.1 kg falls from a tree, what is the acceleration of the Earth towards the apple?

Technically none.  Acceleration is caused by net force. I think it’s more accurate to say that the earth’s acceleration of the Apple is reduced by the amount of gravitational force of the Apple on the earth.

Just because a force is directed toward something doesn’t mean it accelerates it.
Title: Re: The Math for universal Acceleration IS INCORRECT
Post by: xasop on May 09, 2020, 08:22:54 AM
Nope.

There is....

Look, are you here to have a conversation or not? We have been talking more recently than the post you just responded to:

However, one week later the scientist carry out the same experiment again, and bear in mind that for the past week the Earth has been accelerating so its velocity has increased by at
They now find that more muons are reaching the surface of the Earth and thus their relative velocity and time dilation has increased. A week later and they do another test, again different results... and so on.

We don't observe this. What are you basing this claim on?

You never responded to this question. I'm not going to waste my time engaging you if you're going to ignore my responses and instead dig up more posts from a week ago to make "witty" remarks on.

And incidentally, responding to a post talking about general relativity with an analysis based on Newtonian gravitation is neither relevant nor constructive.
Title: Re: The Math for universal Acceleration IS INCORRECT
Post by: Groit on May 09, 2020, 02:53:57 PM
However, one week later the scientist carry out the same experiment again, and bear in mind that for the past week the Earth has been accelerating so its velocity has increased by at
They now find that more muons are reaching the surface of the Earth and thus their relative velocity and time dilation has increased. A week later and they do another test, again different results... and so on.

We don't observe this. What are you basing this claim on?

I'm not claiming that, If you read the last part of the post I wrote:

Quote
UA model does not fit into the way we measure muons at the surface of the Earth. This experiment has been done time and time again over many years and the results are always the same, this is because the Earth is moving at a non-relativistic speed i.e. in its orbit around the sun etc... and the muons are travelling towards the Earth at approx 0.98 c. There is no acceleration for the Earth or muons.

The results for the muon decay experiment are always the same over time. So i would like to know how the UA model could possibly fit into it?

(I didn't respond before because i thought you were taking the P, sorry) 

Title: Re: The Math for universal Acceleration IS INCORRECT
Post by: xasop on May 09, 2020, 04:21:12 PM
The results for the muon decay experiment are always the same over time. So i would like to know how the UA model could possibly fit into it?

And I would like to know why you assert that it wouldn't. Your claim is that UA predicts something that conflicts with observations. I want to know what your basis is for making this prediction, given that observations contradict it. That is, you are not basing your prediction on observations, but on something else.
Title: Re: The Math for universal Acceleration IS INCORRECT
Post by: pricelesspearl on May 09, 2020, 04:49:16 PM
The results for the muon decay experiment are always the same over time. So i would like to know how the UA model could possibly fit into it?

And I would like to know why you assert that it wouldn't. Your claim is that UA predicts something that conflicts with observations. I want to know what your basis is for making this prediction, given that observations contradict it. That is, you are not basing your prediction on observations, but on something else.

He is basing it on simple logic.  The “life expectancy” of the muons is dependent upon the relative velocity of the muons and earth.  If the earth is accelerating the relative velocity would change over time and the “life expectancy” of the muons would change accordingly. But that is not what we see.  The “life expectancy” of the muons stays consistent.
Title: Re: The Math for universal Acceleration IS INCORRECT
Post by: xasop on May 09, 2020, 05:09:22 PM
If the earth is accelerating the relative velocity would change over time

You're the third person to say this without providing any justification whatsoever. We clearly observe that it does not change over time, so why do you say that it would?
Title: Re: The Math for universal Acceleration IS INCORRECT
Post by: pricelesspearl on May 09, 2020, 06:14:10 PM
If the earth is accelerating the relative velocity would change over time

You're the third person to say this without providing any justification whatsoever. We clearly observe that it does not change over time, so why do you say that it would?

That’s the way relative velocities work.  If the velocity of two objects, as measured within their own frame, changes, their relative velocity changes. 

If A is moving 100 mph and B is moving 25 mph..what is their relative velocity?

If A accelerates to 110 mph and B remains at 25...what is their relative velocity now?

It’s pretty basic stuff.
Title: Re: The Math for universal Acceleration IS INCORRECT
Post by: Groit on May 09, 2020, 06:30:51 PM
The results for the muon decay experiment are always the same over time. So i would like to know how the UA model could possibly fit into it?

And I would like to know why you assert that it wouldn't. Your claim is that UA predicts something that conflicts with observations. I want to know what your basis is for making this prediction, given that observations contradict it. That is, you are not basing your prediction on observations, but on something else.

All I'm saying is that if their relative velocity increased over time, then more muons would be observed reaching the surface of the Earth over time, do you agree?

Obviously this is NOT whats being observed, so i would like to know how UA works?  how does an object accelerate in one direction and yet its velocity does NOT increase in the same direction over time?
Title: Re: The Math for universal Acceleration IS INCORRECT
Post by: xasop on May 09, 2020, 06:41:38 PM
That’s the way relative velocities work.  If the velocity of two objects, as measured within their own frame, changes, their relative velocity changes.

But we're not talking about two objects. We're talking about three (or more) objects—the Earth and at least two different muons, measured at different times. You are making an assertion about how the velocities of those two muons are related with no justification, and which contradicts observation. That justification is what I am asking for.

All I'm saying is that if their relative velocity increased over time, then more muons would be observed reaching the surface of the Earth over time, do you agree?

What do you mean by "their relative velocity"? The velocity of what relative to what?

Obviously this is NOT whats being observed, so i would like to know how UA works?  how does an object accelerate in one direction and yet its velocity does NOT increase in the same direction over time?

Nobody has made that claim. UA is explained quite well on the wiki (https://wiki.tfes.org/Universal_Acceleration).
Title: Re: The Math for universal Acceleration IS INCORRECT
Post by: pricelesspearl on May 09, 2020, 10:08:14 PM
Quote

But we're not talking about two objects. We're talking about three (or more) objects—the Earth and at least two different muons, measured at different times. You are making an assertion about how the velocities of those two muons are related with no justification, and which contradicts observation. That justification is what I am asking for.

The velocity of muons relative to one another has nothing to do with it.  It is the relative velocity of the earth and the muons that determines the amount of time dilation and how many muons are observed reaching the earth.

Go the link below.  Somewhere on that page is an option to “vary parameters” and you can enter whatever speed (as a % of c) that you want.  Try different speeds and see happens. Note that the speed of the muons is relative to the ground.

I’d show you what happens myself but I’m on my phone and it would be a little difficult.


http://hyperphysics.phy-astr.gsu.edu/hbase/Relativ/muon.html
Title: Re: The Math for universal Acceleration IS INCORRECT
Post by: xasop on May 09, 2020, 11:49:20 PM
The velocity of muons relative to one another has nothing to do with it.

Are you forgetting your own argument? You have been talking this whole time about what happens over the course of weeks as the Earth accelerates. The same muon does not last for weeks, so you must be talking about two different muons.

Observations show that those two different muons have roughly the same speed relative to the Earth. You claim it would be otherwise under UA. As we can calculate the Earth's change in velocity over a given time, this means that you are making a claim about the velocity of two muons relative to one another.

At the risk of sounding like a broken record, why?
Title: Re: The Math for universal Acceleration IS INCORRECT
Post by: BRrollin on May 10, 2020, 12:22:39 AM
I don’t see any use in talking about relative muon velocity either.

From what I gather, it seems like entire RE argument boils down to this:

1. Muons are made by cosmic ray interactions in the atmosphere. The reaction that makes them results in a tight muon energy distribution and so predictable muon speed.

2. In standard (non circular) UA, our speed on the surface increases as Earth undergoes constant acceleration.

3. Muon #1 has some proper speed, add that to ours and that is the relative speed we measure in our rest frame.

4. Muon #2 (a week later) has some similar proper speed, and that to our much faster speed than last week: change of speed would be 9.8*(86400*7) m/s.

So we would measure an increasing speed of incoming muons by about 864000 m/s each consecutive day.

5. We don’t measure that - muons have similar speeds.
Title: Re: The Math for universal Acceleration IS INCORRECT
Post by: pricelesspearl on May 10, 2020, 01:33:17 AM
Quote

Observations show that those two different muons have roughly the same speed relative to the Earth. You claim it would be otherwise under UA. As we can calculate the Earth's change in velocity over a given time, this means that you are making a claim about the velocity of two muons relative to one another.

At the risk of sounding like a broken record, why?

I’m not making any claim about muons relative to one another.  That has nothing to do with how many muons are observed to survive.  Only the relative velocity between the earth and muons is what matters.

You obviously didn’t go to site I suggested so I’ll tell what you would have found if you did so.

Out of 1 million muons:

7661 survive at .95c
49,312 survive at .98c
121,006 survive at .99c

More muons survive as the velocity increases.  But we don’t see an increase in the number of muons surviving over the many years this experiment has been done.  Consistently around 49k is what is observed. 

Therefore, the relative velocities between the earth and muons is not changing, as you would expect to see if the earth is accelerating and it’s velocity increasing.

If, over some period of time, the earth’s velocity went from .98c to .99c we would observe 121k muons surviving instead of 49k.

That’s as clear as I know how to make it.
Title: Re: The Math for universal Acceleration IS INCORRECT
Post by: xasop on May 10, 2020, 01:06:30 PM
I’m not making any claim about muons relative to one another.  That has nothing to do with how many muons are observed to survive.  Only the relative velocity between the earth and muons is what matters.

And that's observed to be constant. So we're all agreed that there is no problem with UA, then?

You obviously didn’t go to site I suggested so I’ll tell what you would have found if you did so.

Out of 1 million muons:

7661 survive at .95c
49,312 survive at .98c
121,006 survive at .99c

More muons survive as the velocity increases.  But we don’t see an increase in the number of muons surviving over the many years this experiment has been done.  Consistently around 49k is what is observed.

Why are you explaining this? Nobody asked for an explanation of how the experiment works. Indeed, I would hope that all involved understand it by this point.

Therefore, the relative velocities between the earth and muons is not changing, as you would expect to see if the earth is accelerating and it’s velocity increasing.

Yet again you are making this claim with zero justification. How many times do I need to ask why?

If, over some period of time, the earth’s velocity went from .98c to .99c we would observe 121k muons surviving instead of 49k.

Only if you make an assumption about the relative velocity of the muons today and the muons a week from now. You keep insisting that you are making no such assumption, and therefore UA does not present a problem for this experiment.
Title: Re: The Math for universal Acceleration IS INCORRECT
Post by: JSS on May 10, 2020, 03:12:47 PM
If, over some period of time, the earth’s velocity went from .98c to .99c we would observe 121k muons surviving instead of 49k.

Only if you make an assumption about the relative velocity of the muons today and the muons a week from now. You keep insisting that you are making no such assumption, and therefore UA does not present a problem for this experiment.

I am having trouble following both sides of the discussion as I have a number of assumptions I'm guessing at. May I list what I understand so far?

1. Under UA the Earth is constantly accelerating upwards at 9.8m/s

2. Cosmic rays are coming from somewhere outside Earth, and impacting the atmosphere.

3. The energy of the muon that we observe should be related the cosmic rays incoming velocity, plus the upward velocity of the Earth.

4. Since the Earth is constantly going faster, after a year we should see more energetic muons due to the higher collision speed.

I'm assuming that under UA, one or more of these is wrong? Could you tell me where I am diverging from UA theory and explain what is happening instead? Thanks.
Title: Re: The Math for universal Acceleration IS INCORRECT
Post by: xasop on May 10, 2020, 04:12:19 PM
4. Since the Earth is constantly going faster, after a year we should see more energetic muons due to the higher collision speed.

This is yet another instance of the claim that is being repeatedly made with no justification.

Why do ye say that there should be a higher collision speed under UA? Every response to this question seems to involve a re-explanation of how the muon experiment works followed by a re-assertion of the claim without explanation. Could someone please actually answer the question?
Title: Re: The Math for universal Acceleration IS INCORRECT
Post by: JSS on May 10, 2020, 05:10:27 PM
4. Since the Earth is constantly going faster, after a year we should see more energetic muons due to the higher collision speed.

This is yet another instance of the claim that is being repeatedly made with no justification.

Why do ye say that there should be a higher collision speed under UA? Every response to this question seems to involve a re-explanation of how the muon experiment works followed by a re-assertion of the claim without explanation. Could someone please actually answer the question?

I'm trying to break things out and be as precise as possible, sorry I'm not succeeding. Let me try and leave out all the extraneous detail.

1. Under UA the Earth is constantly accelerating upward.

2. Objects are colliding with the Earth/atmosphere from above.

3. Objects over time should be hitting faster and harder as the Earth's speed increases.

This is what I would expect with a body accelerating through space. UA clearly diverges from some of the rules of physics, but can't find what those are, so I am asking for clarification.
Title: Re: The Math for universal Acceleration IS INCORRECT
Post by: BRrollin on May 10, 2020, 05:38:34 PM
4. Since the Earth is constantly going faster, after a year we should see more energetic muons due to the higher collision speed.

This is yet another instance of the claim that is being repeatedly made with no justification.

Why do ye say that there should be a higher collision speed under UA? Every response to this question seems to involve a re-explanation of how the muon experiment works followed by a re-assertion of the claim without explanation. Could someone please actually answer the question?

I'm trying to break things out and be as precise as possible, sorry I'm not succeeding. Let me try and leave out all the extraneous detail.

1. Under UA the Earth is constantly accelerating upward.

2. Objects are colliding with the Earth/atmosphere from above.

3. Objects over time should be hitting faster and harder as the Earth's speed increases.

This is what I would expect with a body accelerating through space. UA clearly diverges from some of the rules of physics, but can't find what those are, so I am asking for clarification.

To simplify it further:

I am running at you at 10 m/s, and you throw a ball at me at 5 m/s. I see the ball come at me at 15 m/s from my rest frame.

If run at you faster, at say 20 m/s instead, then I see the ball come at me at 25 m/s.

In this analogy, I am the earth, and the ball is the muon. My speed is the instantaneous speed the earth has when the muon hits it.

So the muon’s speed relative to us should change if our speed keeps changing.
Title: Re: The Math for universal Acceleration IS INCORRECT
Post by: xasop on May 10, 2020, 05:49:53 PM
3. Objects over time should be hitting faster and harder as the Earth's speed increases.

This is, once again, an assumption. "Over time", we are talking about more than one different object. You are making an assumption about their velocity relative to each other by asserting this.

This is what I would expect with a body accelerating through space. UA clearly diverges from some of the rules of physics, but can't find what those are, so I am asking for clarification.

UA does not "diverge" from any established laws of physics. You are simply refusing to state your assumptions.
Title: Re: The Math for universal Acceleration IS INCORRECT
Post by: JSS on May 10, 2020, 06:23:17 PM
3. Objects over time should be hitting faster and harder as the Earth's speed increases.

This is, once again, an assumption. "Over time", we are talking about more than one different object. You are making an assumption about their velocity relative to each other by asserting this.

This is what I would expect with a body accelerating through space. UA clearly diverges from some of the rules of physics, but can't find what those are, so I am asking for clarification.

UA does not "diverge" from any established laws of physics. You are simply refusing to state your assumptions.

Ok, let me try this from a different angle to try and understand.  In current physics what is accepted is...

1. The Earth, Sun and stars of the milky way are all moving at approximately the same speed relative to each other, a very small fraction of the speed of light, about 0.001c.

2. Cosmic rays are protons that are moving at a very high rate of speed, about 0.98c.

3. So when a cosmic ray hits the earth, it's speed is largely determined by it's speed, the Earth is nearly a stationary target.

So here is my trying to understand UA.

So in UA, the Earth is accelerating forward at 9.8m/s.  It will fairly quickly approach and exceed the speed of the incoming cosmic rays as it moves through the galaxy.

My assumption is that under UA Earth is accelerating through space, and encountering objects, and these encounters would be more energetic as it continues to accelerate.

I am trying to understand which of my assumptions are wrong, and what the correct way is for UA. I know the way I explained it is not correct, but I don't have enough knowledge of the theory to be able to model it, and need some help.
Title: Re: The Math for universal Acceleration IS INCORRECT
Post by: xasop on May 10, 2020, 07:20:06 PM
So in UA, the Earth is accelerating forward at 9.8m/s.  It will fairly quickly approach and exceed the speed of the incoming cosmic rays as it moves through the galaxy.

The assumption that we are even located within a galaxy is unjustified in FET. That conclusion is based upon astronomical evidence coupled with the assumption that the Earth is round and orbits the Sun. Interpreting those same observations in the context of FET, the stars are instead located just a few thousand kilometres above the Earth.

This has nothing to do with UA specifically, this is just Flat Earth Theory, under which UA is one possibility. Of course, in the UA model, the stars accelerate along with the Earth.
Title: Re: The Math for universal Acceleration IS INCORRECT
Post by: JSS on May 10, 2020, 07:57:20 PM
So in UA, the Earth is accelerating forward at 9.8m/s.  It will fairly quickly approach and exceed the speed of the incoming cosmic rays as it moves through the galaxy.

The assumption that we are even located within a galaxy is unjustified in FET. That conclusion is based upon astronomical evidence coupled with the assumption that the Earth is round and orbits the Sun. Interpreting those same observations in the context of FET, the stars are instead located just a few thousand kilometres above the Earth.

This has nothing to do with UA specifically, this is just Flat Earth Theory, under which UA is one possibility. Of course, in the UA model, the stars accelerate along with the Earth.

Good to know, the stars accelerating along with the Earth is not in the Wiki anywhere I could find. Those are the kinds of details I meant when I was asking what about UA diverges from the standard model. It's hard to understand UA not knowing the context it's supposed to exist in, the Wiki is very brief on the subject.

Question, is there anything outside the Earth and a few thousand km above it? Is the rest of the universe an empty void?

I think what makes it hard to discuss the UA hypothesis is that it throws out everything currently accepted about the Earth and the universe, but doesn't give a definitive answer about what replaces it.

The question the recent discussion has been asking should have been "What are cosmic rays and where do they come from under UA" because until that is answered, we can't very well know what their behavior should be. If UA doesn't address that, then it's not a surprise the discussion has been going in circles for weeks. How can we explain how somethign should behave, if that something is undefined?
Title: Re: The Math for universal Acceleration IS INCORRECT
Post by: xasop on May 10, 2020, 08:17:56 PM
Good to know, the stars accelerating along with the Earth is not in the Wiki anywhere I could find. Those are the kinds of details I meant when I was asking what about UA diverges from the standard model. It's hard to understand UA not knowing the context it's supposed to exist in, the Wiki is very brief on the subject.

It is stated on the wiki (emphasis mine):

Quote from: https://wiki.tfes.org/Universal_Acceleration
Objects on the earth's surface have weight because all sufficiently massive celestial bodies are accelerating upward at the rate of 9.8 m/s^2 relative to a local observer immediately above said body.

Question, is there anything outside the Earth and a few thousand km above it? Is the rest of the universe an empty void?

This is an unsettled question in FET.

The question the recent discussion has been asking should have been "What are cosmic rays and where do they come from under UA" because until that is answered, we can't very well know what their behavior should be. If UA doesn't address that, then it's not a surprise the discussion has been going in circles for weeks. How can we explain how somethign should behave, if that something is undefined?

By starting with what we know from observations and developing a model based on that. This is why I have been asking why people have been making predictions based on assumptions which contradict observations, rather than starting with the observations and making predictions about unknowns.
Title: Re: The Math for universal Acceleration IS INCORRECT
Post by: JSS on May 10, 2020, 08:34:01 PM
Good to know, the stars accelerating along with the Earth is not in the Wiki anywhere I could find. Those are the kinds of details I meant when I was asking what about UA diverges from the standard model. It's hard to understand UA not knowing the context it's supposed to exist in, the Wiki is very brief on the subject.

It is stated on the wiki (emphasis mine):

Quote from: https://wiki.tfes.org/Universal_Acceleration
Objects on the earth's surface have weight because all sufficiently massive celestial bodies are accelerating upward at the rate of 9.8 m/s^2 relative to a local observer immediately above said body.

Ah thanks, I missed the significance of that. Is it known what 'sufficently massive' means or what the limits are?

I assume that there must be some kind of 'pusher layer' down below that the UA force acts on, and the rest of the Earth rides atop of.

But this wouldn't be the case for other planets as it would squash them flat as well, so other objects must be fully accelerated, including any atmosphere?

The question the recent discussion has been asking should have been "What are cosmic rays and where do they come from under UA" because until that is answered, we can't very well know what their behavior should be. If UA doesn't address that, then it's not a surprise the discussion has been going in circles for weeks. How can we explain how somethign should behave, if that something is undefined?

By starting with what we know from observations and developing a model based on that. This is why I have been asking why people have been making predictions based on assumptions which contradict observations, rather than starting with the observations and making predictions about unknowns.

Is there any hypothesis on what cosmic rays are under FET and where they come from?

Lastly, would it be appropriate for me to start a new thread with these questions, as the OP subject has been answered. (Yes, an object can accelerate forever and never go past c)
Title: Re: The Math for universal Acceleration IS INCORRECT
Post by: pricelesspearl on May 10, 2020, 09:56:10 PM


Quote

And that's observed to be constant. So we're all agreed that there is no problem with UA, then?

No it contradicts UA.  If the earth was accelerating it wouldn’t be consistent.

Quote

Why are you explaining this? Nobody asked for an explanation of how the experiment works. Indeed, I would hope that all involved understand it by this point.

You may understand how it works but you don’t understand the significance of the results



Quote
Yet again you are making this claim with zero justification. How many times do I need to ask why?

And yet again I have to explain because that is how relative velocities work.  Logic doesn’t need to be justified. If the proper velocities of moving objects change, then so do their relative velocities.  That’s either a valid conclusion or it’s not.  Perhaps you can explain why you think it is not a valid conclusion.

Quote
Only if you make an assumption about the relative velocity of the muons today and the muons a week from now. You keep insisting that you are making no such assumption, and therefore UA does not present a problem for this experiment.

No assumptions necessary because the velocity of muons has been directly measured and it is consistently measured at .98c.  If the earth were accelerating that wouldn’t be the case.

Why do you think otherwise?
Title: Re: The Math for universal Acceleration IS INCORRECT
Post by: xasop on May 10, 2020, 10:15:55 PM
No it contradicts UA.  If the earth was accelerating it wouldn’t be consistent.

I am just going to start typing "why?" every time you say this. It's going to save me some RSI.

You may understand how it works but you don’t understand the significance of the results

It is very difficult to understand when you don't explain yourself.

Quote
Yet again you are making this claim with zero justification. How many times do I need to ask why?

And yet again I have to explain because that is how relative velocities work.  Logic doesn’t need to be justified. If the proper velocities of moving objects change, then so do their relative velocities.  That’s either a valid conclusion or it’s not.  Perhaps you can explain why you think it is not a valid conclusion.

No, that's a diversion because that is not our point of contention. The point of contention is whether this scenario is applicable to the muon experiment.

Why don't we try this instead. Please identify, specifically, the two objects that you claim change relative velocity over the course of, say, a week, as it applies to the muon experiment.

No assumptions necessary because the velocity of muons has been directly measured and it is consistently measured at .98c.

I agree. Indeed, this is my entire position.

If the earth were accelerating that wouldn’t be the case.

Why do you think otherwise?

Because we observe otherwise. My conclusion is based directly on scientific observations, while yours is based on something that you are refusing to state for some reason.
Title: Re: The Math for universal Acceleration IS INCORRECT
Post by: pricelesspearl on May 11, 2020, 02:05:26 AM
Quote
Why don't we try this instead. Please identify, specifically, the two objects that you claim change relative velocity over the course of, say, a week, as it applies to the muon experiment.

The relative velocity between the earth and muons cascading at time "X" and the relative velocity between the earth and the muons cascading at time "Y". 

The relative velocity between muons and the earth is always .98c range, as evidenced by how many muons survive. It doesn't change between time "X" and time "Y". Basic logic tells us that if relative velocity doesn't change, then the proper velocities have not changed.   What does that tell you about earth's velocity?

[/quote]It is very difficult to understand when you don't explain yourself.
Quote

I did explain myself.  The calculations on that page clearly show that as the relative velocity between the earth and muons increase, the number of muons surviving to reach earth increases.  We don't see this.  From this we can conclude that the relative velocity between the earth and muons is not increasing and if that is the case, the proper velocities of the earth and muons are not changing. If the proper velocity of earth is not changing, its not accelerating.   



Title: Re: The Math for universal Acceleration IS INCORRECT
Post by: xasop on May 11, 2020, 09:55:34 AM
Quote
Why don't we try this instead. Please identify, specifically, the two objects that you claim change relative velocity over the course of, say, a week, as it applies to the muon experiment.

The relative velocity between the earth and muons cascading at time "X" and the relative velocity between the earth and the muons cascading at time "Y".

Nope, you've identified more than two objects there. That's not what I asked for.

Please either identify two specific objects whose relative velocities change, or concede that we are talking about more than two objects.
Title: Re: The Math for universal Acceleration IS INCORRECT
Post by: pricelesspearl on May 11, 2020, 11:37:31 AM
Quote
Nope, you've identified more than two objects there. That's not what I asked for.

Please either identify two specific objects whose relative velocities change, or concede that we are talking about more than two objects.

I am talking about the relative velocity between two objects at two separate times. I honestly have no idea what you are talking about.

Title: Re: The Math for universal Acceleration IS INCORRECT
Post by: xasop on May 11, 2020, 01:38:38 PM
I am talking about the relative velocity between two objects at two separate times. I honestly have no idea what you are talking about.

I'm not talking about anything. I am just asking you to identify for me what those two objects are. Your first attempt at responding to this involved identifying the Earth and "muons", plural. More than one muon plus the Earth is at least three objects.
Title: Re: The Math for universal Acceleration IS INCORRECT
Post by: pricelesspearl on May 11, 2020, 02:20:35 PM
Quote
I'm not talking about anything. I am just asking you to identify for me what those two objects are. Your first attempt at responding to this involved identifying the Earth and "muons", plural. More than one muon plus the Earth is at least three objects.

I did identify them. RV between the earth and muons that are cascading at time X (plural because there are millions cascading any given moment) at time x, and the earth and the muons that are cascading at time Y.

The RV between the earth and "the cascade of muons" at time X and the "cascade of muons" of muons at time "Y", if that makes it easier for you.  But it doesn't really matter...the RV between the earth and one single muon at time X and one single muon at time Y, the same principle applies.  If the RV between doesn't change between time X and Y...proper velocities haven't changed, therefore the earth isn't accelerating.

You seem to think that the RV between muons has some bearing on this. I don't believe it does, can you explain why you think otherwise"
Title: Re: The Math for universal Acceleration IS INCORRECT
Post by: xasop on May 11, 2020, 03:10:44 PM
But it doesn't really matter...the RV between the earth and one single muon at time X and one single muon at time Y, the same principle applies.

No it does not.

Suppose you are driving your car on a road. In front of you is a blue car travelling at 70 km/h, and in front of that one is a red car travelling at 90 km/h. You overtake the blue car at 80 km/h, and relative to you, it is moving backwards at 10 km/h. You then accelerate to 100 km/h and pass the red car, which is now moving backwards at 10 km/h relative to you.

Is it reasonable to assume that you were actually moving at the same speed the whole time just because you passed two other cars at the same relative speed?
Title: Re: The Math for universal Acceleration IS INCORRECT
Post by: BRrollin on May 11, 2020, 03:15:54 PM
But it doesn't really matter...the RV between the earth and one single muon at time X and one single muon at time Y, the same principle applies.

No it does not.

Suppose you are driving your car on a road. In front of you is a blue car travelling at 70 km/h, and in front of that one is a red car travelling at 90 km/h. You overtake the blue car at 80 km/h, and relative to you, it is moving backwards at 10 km/h. You then accelerate to 100 km/h and pass the red car, which is now moving backwards at 10 km/h relative to you.

Is it reasonable to assume that you were actually moving at the same speed the whole time just because you passed two other cars at the same relative speed?

That’s a fair point.

So in RE, the muons could be accelerating in between being created and reaching the Earth, and so this could hide many aspects of the motion in between.

I agree with that. Is there any evidence that the muons DO accelerate, if you are indeed making this claim?
Title: Re: The Math for universal Acceleration IS INCORRECT
Post by: Groit on May 11, 2020, 04:20:54 PM
Suppose you are driving your car on a road. In front of you is a blue car travelling at 70 km/h, and in front of that one is a red car travelling at 90 km/h. You overtake the blue car at 80 km/h, and relative to you, it is moving backwards at 10 km/h. You then accelerate to 100 km/h and pass the red car, which is now moving backwards at 10 km/h relative to you.

Is it reasonable to assume that you were actually moving at the same speed the whole time just because you passed two other cars at the same relative speed?

Those cars are all moving in the same direction, the Earth and the muons are travelling towards each other.
Title: Re: The Math for universal Acceleration IS INCORRECT
Post by: xasop on May 11, 2020, 04:43:36 PM
Those cars are all moving in the same direction, the Earth and the muons are travelling towards each other.

In which frame of reference?
Title: Re: The Math for universal Acceleration IS INCORRECT
Post by: Groit on May 11, 2020, 04:57:42 PM
Those cars are all moving in the same direction, the Earth and the muons are travelling towards each other.

In which frame of reference?

Ok, so you are saying that the Earth is accelerating and overtaking cosmic rays that are travelling in the same direction?
Title: Re: The Math for universal Acceleration IS INCORRECT
Post by: xasop on May 11, 2020, 06:43:41 PM
Ok, so you are saying that the Earth is accelerating and overtaking cosmic rays that are travelling in the same direction?

I'm saying that it depends on your frame of reference. Two objects moving at 10 m s-1 relative to each other are moving in the same direction at 5 m s-1 and 15 m s-1 from one possible frame of reference, in opposite directions at 5 m s-1 from another frame of reference, and at 5 m s-1 and 15 m s-1 in the same direction (but the opposite direction to the first case) from yet another.

There is no such thing as absolute velocity, only relative velocity. This is a fundamental concept in both Newtonian and Einsteinian mechanics.
Title: Re: The Math for universal Acceleration IS INCORRECT
Post by: BRrollin on May 11, 2020, 06:57:12 PM
Ok, so you are saying that the Earth is accelerating and overtaking cosmic rays that are travelling in the same direction?

I'm saying that it depends on your frame of reference. Two objects moving at 10 m s-1 relative to each other are moving in the same direction at 5 m s-1 and 15 m s-1 from one possible frame of reference, in opposite directions at 5 m s-1 from another frame of reference, and at 5 m s-1 and 15 m s-1 in the same direction (but the opposite direction to the first case) from yet another.

There is no such thing as absolute velocity, only relative velocity. This is a fundamental concept in both Newtonian and Einsteinian mechanics.

Then it is probably critical for a proper consideration by REers to understand what reference frame UA is relative to.

What is the 9.8 acceleration relative to?
Title: Re: The Math for universal Acceleration IS INCORRECT
Post by: pricelesspearl on May 11, 2020, 07:31:42 PM
Quote
No it does not.

Suppose you are driving your car on a road. In front of you is a blue car travelling at 70 km/h, and in front of that one is a red car travelling at 90 km/h. You overtake the blue car at 80 km/h, and relative to you, it is moving backwards at 10 km/h. You then accelerate to 100 km/h and pass the red car, which is now moving backwards at 10 km/h relative to you.

Is it reasonable to assume that you were actually moving at the same speed the whole time just because you passed two other cars at the same relative speed?

If you predict how long it would take you to overtake the cars without taking acceleration into account and that prediction turned out to be right…then yes, it would be a reasonable assumption.

If muons accelerate, the time dilation calculations that assume they don’t accelerate, wouldn’t be able to accurately predict how many survive to reach Earth’s surface.  But the calculations do accurately predict, indicating muons do not accelerate.

Quote
What is the 9.8 acceleration relative to?

Oy...here we go again.   ::)

Acceleration is not relative.  As an FE proponent...how do you know that the earth is accelerating?  and how do you know how fast it is accelerating?
Title: Re: The Math for universal Acceleration IS INCORRECT
Post by: xasop on May 11, 2020, 08:01:18 PM
If you predict how long it would take you to overtake the cars without taking acceleration into account and that prediction turned out to be right…then yes, it would be a reasonable assumption.

If muons accelerate, the time dilation calculations that assume they don’t accelerate, wouldn’t be able to accurately predict how many survive to reach Earth’s surface.  But the calculations do accurately predict, indicating muons do not accelerate.

I notice you completely avoided answering the question. Well done.
Title: Re: The Math for universal Acceleration IS INCORRECT
Post by: pricelesspearl on May 11, 2020, 10:59:07 PM
If you predict how long it would take you to overtake the cars without taking acceleration into account and that prediction turned out to be right…then yes, it would be a reasonable assumption.

If muons accelerate, the time dilation calculations that assume they don’t accelerate, wouldn’t be able to accurately predict how many survive to reach Earth’s surface.  But the calculations do accurately predict, indicating muons do not accelerate.

I notice you completely avoided answering the question. Well done.

I didn’t avoid.  I answered that under certain circumstances it would be reasonable to assume your hypothetical car was maintained a constant speed.
Title: Re: The Math for universal Acceleration IS INCORRECT
Post by: xasop on May 11, 2020, 11:43:42 PM
I didn’t avoid.  I answered that under certain circumstances it would be reasonable to assume your hypothetical car was maintained a constant speed.

The circumstances were given in the hypothetical. You chose to ignore the hypothetical and provide an answer to a different question. You did not answer the question I asked.
Title: Re: The Math for universal Acceleration IS INCORRECT
Post by: Groit on May 12, 2020, 08:10:47 AM
Ok, so you are saying that the Earth is accelerating and overtaking cosmic rays that are travelling in the same direction?

I'm saying that it depends on your frame of reference. Two objects moving at 10 m s-1 relative to each other are moving in the same direction at 5 m s-1 and 15 m s-1 from one possible frame of reference, in opposite directions at 5 m s-1 from another frame of reference, and at 5 m s-1 and 15 m s-1 in the same direction (but the opposite direction to the first case) from yet another.

There is no such thing as absolute velocity, only relative velocity. This is a fundamental concept in both Newtonian and Einsteinian mechanics.

Don't forget that cosmic rays travel in all directions, there will be some of them travelling towards the accelerating Earth as well as those in the opposite direction. how would we measure the relative velocities?
Title: Re: The Math for universal Acceleration IS INCORRECT
Post by: xasop on May 12, 2020, 10:51:43 AM
Don't forget that cosmic rays travel in all directions, there will be some of them travelling towards the accelerating Earth as well as those in the opposite direction. how would we measure the relative velocities?

What?
Title: Re: The Math for universal Acceleration IS INCORRECT
Post by: BRrollin on May 12, 2020, 04:30:00 PM
Don't forget that cosmic rays travel in all directions, there will be some of them travelling towards the accelerating Earth as well as those in the opposite direction. how would we measure the relative velocities?

What?

It may help understanding this question to note that there IS an absolute velocity in Einstein! The speed of light, which comprises one of the two fundamental postulates of special relativity.
Title: Re: The Math for universal Acceleration IS INCORRECT
Post by: Yobame on May 13, 2020, 08:54:35 PM
No. There is NO absolute velocity in Einstein's theory. This is why is called “Theory of relativity”, and that is the meaning of the word “relativity” in the theory.

The Speed of light in the theory of relativity IS the speed of the entropy. That’s mean the maximum speed of an object with mass will travel in the vacuum, and this speed is the same for every point of reference and it is RELATIVE to the observer only.
Title: Re: The Math for universal Acceleration IS INCORRECT
Post by: BRrollin on May 14, 2020, 01:19:42 PM
No. There is NO absolute velocity in Einstein's theory. This is why is called “Theory of relativity”, and that is the meaning of the word “relativity” in the theory.

The Speed of light in the theory of relativity IS the speed of the entropy. That’s mean the maximum speed of an object with mass will travel in the vacuum, and this speed is the same for every point of reference and it is RELATIVE to the observer only.

That does not mesh with what I have read at all. Just look up the postulates of special relativity and I think you’ll see right away that the constancy of the speed of light is fundamental to it.

In reading Einstein’s 1905 paper, I don’t see any mention of entropy at all.